Download as pdf or txt
Download as pdf or txt
You are on page 1of 203

1. Topography of the pelvic organs.

Vagina
• The vagina is a fibromusculomembranous sheath communicating the uterine cavity with
the exterior at the vulva. It constitutes the excretory channel for the uterine secretion
and menstrual blood. It is the organ of copulation and forms the birth canal of
parturition. The canal is directed upwards and backwards forming an angle of 45° with
the horizontal in erect posture.
• The long axis of the vagina almost lies parallel to the plane of the pelvic inlet and at right
angles to that of the uterus. The diameter of the canal is about 2.5 cm, being widest in
the upper part and narrowest at its introitus. It has got enough power of distensibility as
evident during childbirth.
• Walls: Vagina has got an anterior, a posterior, and two lateral walls. The anterior and
posterior walls are apposed together but the lateral walls are comparatively stiffer
especially at its middle, as such it looks ‘H’ shaped on transverse section. The length of
the anterior wall is about 7 cm and that of the posterior wall is about 9 cm The upper
end of vaginal is above the pelvic floor.
Cervix
• The cervix is the neck of the uterus, the lower, narrow portion where it joins with
the upper part of the vagina. It is cylindrical or conical in shape and protrudes
through the upper anterior vaginal wall. Approximately half its length is visible,
the remainder lies above the vagina beyond view. The vagina hasa thick layer
outside and it is the opening where the fetus emerges during delivery
• Epithelial Lining of the cervix
• Endocervical canal and glands
• There is a median ridge on both the anterior and posterior surface of the canal
from which transverse folds radiate. This arrangement is called arbor vitae uteri.
The canal is lined by single layer of tall columnar epithelium with basal nuclei.
Those placed over the top of the folds are ciliated.
• There are patches of cubical basal or reserve cells underneath the columnar
epithelium. These cells may undergo squamous metaplasia or may replace the
superficial cells.The glands which dip into the stroma are of complex racemose
type and are lined by secretory columnar epithelium.
• There is no stroma unlike the corpus and the lining epithelium rests on a thin
basement membrane. The change in the epithelium and the glands during
menstrual cycle and pregnancy are not so much as those in the endometrium.
• Portio vaginalis
• It is covered by stratified squamous epithelium and extends right up to the
external os where there is abrupt change to columnar type.

Uterus
• The uterus or womb is the major female reproductive organ. The uterus provides
mechanical protection, nutritional support, and waste removal for the developing
embryo (weeks 1 to 8) and fetus (from week 9 until the delivery). In addition,
contractions in the muscular wall of the uterus are important in pushing out the fetus at
the time of birth
• The uterus contains three suspensory ligaments that help stabilize the position of the
uterus and limits its range of movement. The uterosacral ligaments keep the body from
moving inferiorly and anteriorly. The round ligaments restrict posterior movement of
the uterus. The cardinal ligaments also prevent the inferior movement of the uterus.
• Тhe uterus is a pear-shaped muscular organ. Its major function is to accept a fertilized
ovum which becomes implanted into the endometrium, and derives nourishment from
blood vessels which develop exclusively for this purpose. The fertilized ovum becomes
an embryo, develops into a fetus and gestates until childbirth. If the egg does not
embed in the wall of the uterus, a female begins menstruation
Fallopian tube
• The uterine tubes are paired structures, measuring about 10 cm (4") and are situated in
the medial threefourth of the upper free margin of the broad ligaments. Each tube has
got two openings, one communicating with the lateral angle of the uterine cavity, called
uterine opening and measures 1 mm in diameter, the other is on the lateral end of the
tube, called pelvic opening or abdominal ostium and measures about 2 mm in diameter
• parts: There are four parts, from medial to lateral, they are—(1) intramural or
interstitial lying in the uterine wall and measures 1.25 cm (1/2") in length and 1 mm in
diameter; (2) isthmus almost straight and measures about 2.5 cm (1") in length and 2.5
mm in diameter; (3) ampulla—tortuous part and measures about 5 cm (2") in length
which ends in wide; (4) infundibulum measuring about 1.25 cm (1/2") long with a
maximum diameter of 6 mm.
• The abdominal ostium is surrounded by a number of radiating fimbriae, one of these is
longer than the rest and is attached to the outer pole of the ovary called ovarian
fimbria.
Structures—It consists of 3 layers:
1. Serous—consists of peritoneum on all sides except along the line of attachment of
mesosalpinx.
2. Muscular—arranged in two layers—outer longitudinal and inner circular.
3. Mucous membrane is thrown into longitudinal folds. It is lined by columnar epithelium, partly
ciliated, others secretory nonciliated and ‘Peg cells’. The epithelium rests on delicate vascular
reticulum of connective tissue. There is no submucous layer nor any glands. Changes occur in
the tubal epithelium during menstrual cycle but are less pronounced and there is no shedding

Ovaries
• Each gland is oval in shape and pinkish grey in color and the surface is scarred during
reproductive period. It measures about 3 cm in length, 2 cm in breadth and 1 cm in
thickness. Each ovary presents two ends—tubal and uterine, two borders— mesovarium
and free posterior and two surfaces—medial and lateral. The ovaries are intraperitoneal
structures. In nulliparae, the ovary lies in the ovarian fossa on the lateral pelvic wall.
• The ovary is attached to the posterior layer of the broad ligament by the mesovarium,
to the lateral pelvic wall by infundibulopelvic ligament and to the uterus by the ovarian
ligament.
Urinary bladder
The bladder is a hollow muscular organ with considerable power of distension. Its capacity is
about 450 mL (15 oz) but can retain as much as 3–4 liters of urine. When distended, it is ovoid
in shape. It has got: (1) an apex, (2) superior surface, (3) base,(4) two inferolateral surfaces and
(5) neck, which is continuous with the urethra. The base and the neck remain fixed even when
the bladder is distended.
Relations: The superior surface is related with the peritoneum of the uterovesical pouch.The
base is related with the supravaginal cervix and the anterior fornix. The ureters, after crossing
the pelvic floor at the sides of the cervix, enter the bladder on its lateral angles. In the interior
of bladder, the triangular area marked by three openings—two ureteric and one urethral, is
called the trigone. The inferolateral surfaces are related with the space of Retzius.
The neck rests on the superior layer of the urogenital diaphragm.
Pelvic ureter
The pelvic ureter extends from its crossing over the pelvic brim up to its opening into the
bladder. It measures about 13 cm in length and has a diameter of 5 mm.

2. Menstrual cycle. Regulation of a menstrual cycle. Cyclic changes in women’s reproductive


organs.

Phases Of Menstrual Cycle


1. Menstruation or Menses Phase

2. Folicullar Phase or Proliferative Phase

3. Ovulation

4. Luteal Phase

Menstruation Phase

• Menstruation occurs in response to drops in the


level of the hormone progesterone.

• The onset of menstruation, called a period,


monthly, menses or menstrual period, begins a
new menstrual cycle and is considered day one.

• This phase, which typically lasts from day 3 to day 7, is the time when the lining of the
uterus is actually shed out through the vagina if pregnancy has not occurred.
Follicular Phase

• This phase typically takes place from days 6 to 14.

• During this time, the level of the hormone estrogen rises, which causes the lining of the
uterus (called the endometrium) to grow and thicken in preparation for the
implantation of fertilized egg.

• The increase in levels of estrogens produced by the ovary causes the hypothalamus to
secrete a hormone GnRH which in turn causes the anterior pituitary gland to secrete
large amounts of the hormones FSH (Follicle Stimulating Hormone) and LH
(Leutinizeing Hormone).

• FSH causes follicles in the ovaries to grow.

• During days 10 to 14, only one of the developing follicles will form a fully mature egg
(ovum).

Ovulation

• This phase occurs at about day 14.

• When the follicle has matured, it secretes enough


estradiol to trigger the acute release of luteinizing
hormone (LH).

• A sudden increase in LH causes the ovary to release an


ovum, the largest cell of the body (with a diameter of
about 0.5 mm).

• It is released into the body cavity where it is (almost


always) caught by the fimbriae (finger-like projections) of the fallopian tube where it
travels toward the uterus awaiting fertilization by a sperm.

Luteal Phase

• This phase lasts from about day 15 to day 28.

• During this phase, the empty follicle, now called the corpus luteum, releases the
hormone progesterone which acts to keep the endometrial lining stable in case of
fertilization and implantation.
• It raises the body temperature by half- to one degree Fahrenheit (one-quarter to one-
half degree Celsius), thus women who record their temperature on a daily basis will
notice that they have entered the luteal phase.

• If the egg becomes fertilized by a sperm and attaches it self to the uterine wall,
progesterone levels remain high , the endometrium is not shed and the woman
becomes pregnant.
3. The modern points of view about the neurohumoral regulation of the menstrual
function.

a) At the beginning of each monthly menstrual cycle, levels of gonadal steroids are low and
have been decreasing since the end of the luteal phase of the previous cycle.
b) With the demise of the corpus luteum, FSH levels begin to rise and a cohort of growing
follicles is recruited. These follicles each secrete increasing levels of estrogen as they grow in
the follicular phase. This, in turn, is the stimulus for uterine endometrial proliferation.
c) Rising estrogen levels provide negative feedback on pituitary FSH secretion, which begins to
wane by the midpoint of the follicular phase. Conversely, LH initally decreases in response to
rising estradiol levels, but late in the follicular phase the LH level is increased dramatically
(biphasic response).
d) At the end of the follicular phase (just prior to ovulation), FSH-induced LH receptors are
present on granulosa cells and, with LH stimulation, modulate the secretion of progesterone.
e) After a sufficient degree of estrogenic stimulation, the pituitary LH surge is triggered, which is
the approximate cause of ovulation that occur 24 to 36 hours later. Ovulation heralds the
transition to the luteal-secretory phase.
f) The estrogen level decreases through the early luteal phase from just before ovulation until
the midluteal phase, when it begins to rise again as a result of corpus luteum secretion.
g) Progesterone levels rise precipitously after ovulation and can be used as a presumptive sign
that ovulation has occurred.
h) Both estrogen and progesterone levels remain elevated through the lifespan of the corpus
luteum and then wane with its demise, thereby setting the stage for the next cycle.

4. The cancer of the cervix of uterus. The role of women consultation in it’s prophylaxis
(prevention).

Primary prevention

(1) Identifying ‘high-risk’ female


− Women with high risk HPV infection
− Early sexual intercourse.
− Early age of first pregnancy.
− Too many births/too frequent birth.
− Low socioeconomic status.
− Poor maintenance of local hygiene

(2) Prophylactic HPV vaccine : is approved to all school girls (12–18 years) and women (16–25
years).
*Three doses are to be given :- Bivalent 0–2–6 month, Quadrivalent 0–1–6 month.

(3)Use of condom- to prevent from Std

Secondary prevention
The detection of the disease at an earlier stage when it is still curable.
Detection is done by nurses and other paramedical health workers using a simple speculum for
visual inspection of the cervix.
abnormal cervix has the following characters : Reddish, red or white area of patch, growth or
ulcer on the surface and bleeds on touch.

5. Dysfunctional uterine bleeding in reproductive and premenopausal age. Causes,


differential diagnosis. Treatment.
causes
Differential diagnosis:
• Adenomyosis
• Endometrial polyps
• Submucous leiomyomata (fibroids)
• Surface lesions of the genital tract
• Uterine sarcoma
Treatment
MEDICAL MANAGEMENT OF DUB
Hormones: With the introduction of potent orally active progestins, they became the mainstay
in the management of DUB in all age groups and practically replaced the isolated use of
estrogens and androgens.
Progestins: The common preparations used are norethisterone acetate and
medroxyprogesterone acetate

SURGICAL MANAGEMENT OF DUB


Uterine curettage
Endometrial ablation/resection
Hysterectomy
Uterine Curettage
It is done predominantly as a diagnostic tool for elderly women but at times, it has got
hemostatic and therapeutic effect by removing the necrosed and unhealthy endometrium. It
should be done following ultrasonography for detection of endometrial pathology. The
indication is an urgent one, if the bleeding is acyclic and where endometrial pathology is
suspected. Ideally hysteroscopy and directed biopsy should be considered both for the purpose
of diagnosis and therapy. Presently, dilatation and curettage should be used neither as a
diagnostic tool nor for the purpose of therapy.
Endometrial Ablation/Resection
Indications are: (a) Failed medical treatment, (b) Women who do not wish to preserve
menstrual or reproductive function, (c) Uterus—normal size or not bigger than 10 weeks
pregnancy size, (d) Small uterine fibroids (< 3 cm), (e) Women who want to avoid longer
surgery, (f) Woman who prefers to preserve her uterus
Hysterectomy is not recommended as a first line therapy for heavy menstrual bleeding (HMB)
or DUB. However, hysterectomy is justified when the conservative treatment fails or
contraindicated and the blood loss impairs the health and quality of life. Presence of
endometrial hyperplasia and atypia on endometrial histology is an indication for hysterectomy.
The decision can be made easily as the patient is approaching 40. Hysterectomy may be done
depending on the route by vaginal, abdominal, or laparoscopic assisted vaginal method. In this
regard, the factors to consider are: uterine size, mobility, descent, previous surgery, and
presence of comorbidities (obesity,diabetes, heart disease, or hypertension). Healthy ovaries
may be preserved at the time of hysterectomy especially those under 45 years of age.
6. The structure and functions of ovary. Sexual steroidal hormones. Ovarian cycle.
The structure and function of ovary.
The ovaries are paired sex glands or gonads in female which are concerned with (2 main
functions):
• Germ cell maturation, storage and its release.
• Steroidogenesis. (hormonal production)
The ovary is covered by a single layer of cubical cell known as germinal epithelium.
The substance of the gland consists of outer cortex and inner medulla.
• Cortex - the cortex is studded with numerous follicular structures, called the functional
units of the ovary in various phases of their development. The structures include
primordial follicles, maturing follicles, Graafian follicles and corpus luteum and corpus
albicans.
• Medulla - It consists of loose connective tissues, few unstriped muscles, blood vessels
and nerves. There are small collection of cells called “hilus cells” which are homologous
to the interstitial cells of the testes.

Sexual steroidal hormones


• Androgens – dehydroepiandrosterone, androstenedione and testosterone. Produced in
the ovary by all three types of cells—stroma, theca and granulose cells.
• Estrogens – estradiol(E2) and estrone. Produced in granulosa cells of the follicles.
• Progestogens – progestins like progesterone. Produced in luteinized theca granulosa
cells of the corpus luteum.

Ovarian cycle.
Definition - The development and maturation of a follicle, ovulation and formation of corpus
luteum and its degeneration constitute an ovarian cycle. All these events occur within 4 weeks.
Ovarian cycle consists of:
1. Recruitment of groups of follicles
2. Selection of dominant follicle and its maturation.
3. Ovulation
4. Corpus luteum formation
5. Demise of the corpus luteum

Has 2 phases.
1. Follicular phase
• This phase typically takes place from days 6 to 14.
• During this time, the level of the hormone estrogen rises, which causes the lining of the
uterus (called the endometrium) to grow and thicken in preparation for the
implantation of a fertilized egg.
• The increase in levels of estrogens produced by the ovary causes the hypothalamus to
secrete a hormone GnRH which in turn causes the anterior pituitary gland to secrete
large amounts of the hormones FSH (Follicle Stimulating Hormone) and LH (Leutinizeing
Hormone).
• FSH causes follicles in the ovaries to grow.
• During days 10 to 14, only one of the developing follicles will form a fully mature egg
(ovum).

2. Luteal phase
• This phase lasts from about day 15 to day 28.
• After the release of the oocyte, the remaining granulosa and theca cells on the ovary
form the corpus luteum (CL).
• corpus luteum, releases the hormone progesterone which acts to keep the endometrial
lining stable in case of fertilization and implantation.
• It raises the body temperature by half- to one degree Fahrenheit (one-quarter to one-
half degree Celsius), thus women who record their temperature on a daily basis will
notice that they have entered the luteal phase.
• If the egg becomes fertilized by a sperm and attaches itself to the uterine wall,
progesterone levels remain high , the endometrium is not shed and the woman
becomes pregnant.
• If there wasn’t fertilization, Demise of the corpus luteum and formation of corpus
albicans (white body).

7. Common principles of complex therapy of acute diseases of the upper part of the female
reproductive system.

The principles of therapy are:


1. To control the infection energetically.
Outpatient therapy: Apart from adequate rest and analgesic, antibiotics are to be prescribed
even before the microbiological report is available.
• Patient should have oral therapy for 14 days
• Regimen A – Levofloxacin 500 mg (or, ofloxacin 400 mg) PO once daily with or without –
Metronidazole 500 PO bid
• Regimen B – Ceftriaxone 250 mg IM single dose + Doxycycline 100 mg PO bid with or
without – Metronidazole 500 mg PO bid for 14 days.

Inpatient therapy -
• Bed rest is imposed.
• Oral feeding is restricted.
• Dehydration and acidosis are to be corrected by intravenous fluid.
• Intravenous antibiotic therapy is recommended for at least 48 hours but may be
extended to 4 days, if necessary.
• Regimen A - Cefoxitin 2 gm IV every 6 hours for 2-4 days PLUS „ Doxycycline
100 mg PO for 14 days

2. To prevent infertility and late sequelae.


3. To prevent reinfection.
• Educating the patient to avoid reinfection and the potential hazards of it.
• The patient should be warned against multiple sexual partners.
• To use condom.
• The sexual partner or partners are to be traced and properly investigated to find out the
organism(s) and treated effectively.

8. Adrenogenital syndrome. Etiology, clinical features, differential diagnosis, treatment.


Definitions of hypertrichosis, hirsutism, virilism.
Etiology
• It is an autosomal recessive disorder. It is due to inborn error of adrenal steroid
metabolism, commonly due to 21-hydroxylase (95%) and rarely due to 11-hydroxylase
or 3β hydroxy steroid dehydrogenase deficiency. There is lack of cortisol production
resulting in excess of ACTH production from the pituitary. ACTH in turn, stimulates the
adrenal to produce excess androgens with virilization of female offspring. Associated
aldosterone deficiency may lead to excess salt depletion. There is often history of
affection of sibling. The girls are potentially fertile.
Clinical presentation
• Ambiguity of sex at birth—Cases of ambiguity of sex detected at birth are due to
adrenogenital syndrome unless proved otherwise. Hirsutism and amenorrhea may be
the presenting features around puberty in milder form.
Diagnosis at birth
• The suspected anatomic abnormalities include:
• An enlarged clitoris
• Presence of penile urethra or hypospadius.
• Associated metabolic abnormality—salt wasting (hyponatremia, hyperkalemia)
and hypotension
• may be present.
• Fusion of the labia minora.
• The presence of any one or more of the above features necessitates further
investigations for
• confirmation of an early diagnosis.
Differential diagnosis
• 5-Alpha-Reductase Deficiency
• Adrenal Hypoplasia
• Androgen Insensitivity Syndrome
• Bilateral adrenal hemorrhage
• Congenital Adrenal Hyperplasia
• Defects in testosterone synthesis
• Denys-Drash Syndrome
• Differences (Disorders) of Sex Development (DSDs)
• Familial Glucocorticoid Deficiency
• Fluid, Electrolyte, and Nutrition Management of the Newborn
• Gender Identity
• Hyperkalemia in Emergency Medicine
• Hyponatremia in Emergency Medicine
• Mixed gonadal dysgenesis
• Nutritional Considerations in Failure to Thrive
• Obstructive uropathy

Treatment
• Supplying enough glucocorticoid to reduce hyperplasia and overproduction of
androgens or mineralocorticoids
• Providing replacement mineralocorticoid and extra salt if the person is deficient
• Providing replacement testosterone or estrogens at puberty if the person is deficient
• Additional treatments to optimize growth by delaying puberty or delaying bone
maturation
• If CAH is caused by the deficiency of the 21-hydroxylase enzyme, then treatment aims to
normalize levels of main substrate of the enzyme - 17α-hydroxyprogesterone.
Hirsutism: Hirsutism is the excessive growth of androgen dependent sexual hair (terminal hair)
in facial and central part of the body that worries the patient
Hypertrichosis: Hypertrichosis connotes excessive growth of non-sexual (fetal lanugo type) hair.
Virilism: Virilism is defined as the presence of any one or more of the following features—
deepening of the voice, temporal balding, amenorrhea, enlargement of clitoris (clitoromegaly)
and breast atrophy. It is a more severe form of androgen excess.Virilism may be due to adrenal
hyperplasia or tumors of adrenal or ovary.

9. The structure and the function of uterus.


The uterus is a hollow pyriform muscular organ situated in the pelvis between the bladder
in front and the rectum behind
position: Its normal position is one of the anteversion and anteflexion. The uterus usually
inclines to the right (dextrorotation) so that the cervix is directed to the left (levorotation) and
comes in close relation with the left ureter.
measurements and parts: The uterus measures about 8 cm long, 5 cm wide at the fundus and
its walls are about 1.25 cm thick. Its weight varies from 50–80 g. It has got the following parts
• Body or corpus • Isthmus • Cervix
Body or corpus: The body is further divided into fundus—the part which lies above the
openings of the uterine tubes. The body properly is triangular and lies between the openings of
the tubes and the isthmus.The superolateral angles of the body of the uterus project outwards
from the junction of the fundus and body and are called the cornua of the uterus. The uterine
tube, round ligament, and ligament of the ovary are attached to each cornu.
Isthmus: The isthmus is a constricted part measuring about 0.5 cm situated between the body
and the cervix. It is limited above by the anatomical internal os and below by the histological
internal os (Aschoff). Some consider isthmus as a part of the lower portion of the body of the
uterus.
Cervix: The cervix is the lowermost part of the uterus. It extends from the histological internal
os and ends at external os which opens into the vagina after perforating the anterior vaginal
wall. It is almost cylindrical in shape and measures about 2.5 cm in length and diameter. It is
divided into a supravaginal part—the part lying above the vagina and a vaginal
part which lies within the vagina, each measuring 1.25 cm. In nulliparous, the vaginal part of
the cervix is conical with the external os looking circular, whereas in parous, it is cylindrical with
the external os having bilateral slits. The slit is due to invariable tear of the circular muscles
surrounding the external os and gives rise to anterior and posterior lips of the cervix.
Cavity: The cavity of the uterine body is triangular on coronal section with the base above
and the apex below. It measures about 3.5 cm. There is no cavity in the fundus. The cervical
canal is fusiform and measures about 2.5 cm. Thus, the normal length of the
uterine cavity including the cervical canal is usually 6–7 cm
Relations
Anteriorly: Above the internal os, the body forms the posterior wall of the uterovesical
pouch. Below the internal os, it is separated from the base of the bladder by loose areolar
tissue.
Posteriorly: It is covered by peritoneum and forms the anterior wall of the pouch of Douglas
containing coils of intestine.
Laterally: The double folds of peritoneum of the broad ligament are attached laterally
between which the uterine artery ascends up. Attachment of the Mackenrodt’s ligament
extends from the internal os.
Structures
Body—The wall consists of 3 layers from outside
inwards:
♦ Perimetrium—It is the serous coat which invests the entire organ except on the lateral
borders.The peritoneum is intimately adherent to the underlying muscles.
♦ Myometrium—It consists of thick bundles of smooth muscle fibers held by connective tissues
and are arranged in various directions. During pregnancy, however, three distinct layers can be
identified—outer longitudinal, middle interlacing and the inner circular.
♦ Endometrium—The mucous lining of the cavity is called endometrium. As there is no
submucous layer, the endometrium is directly apposed to the muscle coat. It consists of lamina
propria and surface epithelium. The surface epithelium is a single layer of ciliated columnar
epithelium. The lamina propria contains stromal cells, endometrial glands, vessels and nerves.
The glands are simple tubular and lined by mucus secreting non-ciliated columnar epithelium
which penetrate the stroma and sometimes even enter the muscle coat. All the components
are changed during menstrual cycles.The endometrium is changed to decidua during pregnancy

Function of the uterus


• Тhe uterus is a pear-shaped muscular organ. Its major function is to accept a fertilized
ovum which becomes implanted into the endometrium, and derives nourishment from
blood vessels which develop exclusively for this purpose.
• The fertilized ovum becomes an embryo, develops into a fetus and gestates until
childbirth.
• If the egg does not embed in the wall of the uterus, a female begins menstruation
• The uterus provides mechanical protection, nutritional support, and waste removal for
the developing embryo (weeks 1 to 8) and fetus (from week 9 until the delivery). In
addition, contractions in the muscular wall of the uterus are important in pushing out
the fetus at the time of birth.

10. The role of women’s consultation and rehabilitation after acute inflammatory disease of
internal genital organs.

Repeat smears and cultures from the discharge are to be done after 7 days following the full
course of treatment.
The tests are to be repeated following each menstrual period until it becomes negative for
three consecutive reports when the patient is declared cured.
Until she is cured and her sexual partner(s) have been treated and cured, the patient must be
prohibited from intercourse.
The only unequivocal proof of successful treatment after salpingitis is an intrauterine
pregnancy.
And consult the patient to avoid the reinfection.
• Educating the patient to avoid reinfection and the potential hazards of it.
• The patient should be warned against multiple sexual partners.
• To use condom.
• The sexual partner or partners are to be traced and properly investigated to find out the
organism(s) and treated effectively.
• Ask not to douche. Douching upsets the balance of bacteria in vagina.
• Ask to Pay attention to hygiene habits. Wipe from front to back after urinating or
having a bowel movement to avoid introducing bacteria from colon into the vagina.

11. Trichomoniasis. Etiology, clinical features, diagnosis, treatment, prophylaxis

Etiology
Vaginal trichomoniasis is the most common and important cause of vaginitis in the childbearing
period. Causative Organism: It is caused by Trichomonas vaginalis, a pear-shaped unicellular
flagellate protozoa. It measures 20 μ long and 10 μ wide (larger than a WBC). It has got four
anterior flagellae and a spear-like protrusion at the other end with an undulating membrane
surrounding its anterior twothird. It is actively motile. The organism is predominantly transmitted
by sexual contact, the male harbors the infection in the urethra and prostate. The transmission
may also be possible by the toilet articles from one woman to the other or through examining
gloves. The incubation period is 3–28 days.
Clinical Features:
(a) There is sudden profuse and offensive vaginal discharge often dating from the last
menstruation.
(b) Irritation and itching of varying degrees within and around the introitus are common.
(c) There is presence of urinary symptoms such as dysuria and frequency of micturition.
(d) There may be history of previous similar attacks. Women with trichomoniasis should be
evaluated for other STDs including N. gonorrheae, C. trachomatis, and HIV.

On Examination
(a) There is thin, greenish-yellow and frothy offensive discharge per vaginum.
(b) The vulva is inflamed with evidences of pruritus.
(c)Vaginal examination may be painful. The vaginal walls become red and inflamed with multiple
punctate hemorrhagic spots. Similar spots are also found over the mucosa of the portio vaginalis
part of the cervix on speculum examination giving the appearance of ‘strawberry’.

Diagnosis:
(a) Identification of the trichomonas is done by hanging drop preparation. If found negative even
on repeat examination, the confirmation may be done by culture.
(b) Culture of the discharge collected by swabs in Diamond’s TYM or Feinberg Whittington
medium. In suspected cases, gonococcal or monilial infection should be excluded.

Treatment:
The treatment is very much effective with metronidazole. Metronidazole 200 mg thrice daily by
mouth is to be given for 1 week. A single dose regimen of 2 g is an alternative. Tinidazole single
2 gm dose PO is equally effective. The husband should be given the same treatment schedule for
1 week. Resistance to metronidazole is extremely rare. The husband should use condom during
coitus irrespective of contraceptive practice until the wife is cured.
Prophylaxis
• Use condoms while having sex
• Limited the sexual partners
• Vaccination against T. vaginalis is particularly interesting for high-risk individuals to
protect themselves and their partners

12. Premenstrual syndrome. Etiology, pathogenesis, clinic, diagnostic, therapy.

Def:
Premenstrual syndrome is a psychoneuro endocrine disorder with cyclic appearance of a
large number of symptoms just prior to menstruation.

Etiology:
Unknown etiology.
old eytiological theories are:
estrogen excess, estrogen withdrawal, progesterone deficiency, vit B6 deficiency,
electrolyte imbalances

pathogenesis:
The exact cause is not known but,
(a) Alteration estrogen: progesterone ratio or diminished progesterone level
(b)decreased synthesis of serotonin
(c)withdrawal of endorphins (neurotransmitters) from CNS
(d)Psychological and psychosocial changes
(e)alterations in TRH, prolactin, renin, aldosterone, prostaglandins and other hormone
levels

clinical symptoms:
breast tenderness
swelling of the extremities
Weight gain
neuropsychiatric symptoms (irritability, Depression, anxiety, forgetfulness)
fatigue
insomnia

Diagnostic:
History, symptoms and signs helps

Treatment:
Nonpharmacological-
Stress management, Diet manipulation (reduce salt caffeine and alcohol)

pharmacological-
Danazol
GnRH analogues
antidepressant drugs
Pyridoxine
Diuretics (frusemide, Spironolactone)
Anxiolytic agents
Serotonin Reuptake Inhibitors
Noradrenaline Reuptake Inhibitors
Progesterone

surgical- Oophorectomy

13. The methods of provocation of gonorrhea, criterion of reconvalescence, prevention of


gonorrhea.
Causes

Gonorrhea is caused by the bacterium Neisseria gonorrhoeae. The gonorrhea bacteria are most
often passed from one person to another during sexual contact, including oral, anal or vaginal
intercourse.

Risk factors

Sexually active women younger than 25 and men who have sex with men are at increased risk
of getting gonorrhea.

Other factors that can increase your risk include:


• Having a new sex partner

• Having a sex partner who has other partners

• Having more than one sex partner

• Having had gonorrhea or another sexually transmitted infection

criterion of reconvalescence

• Most patients recover fully after taking antibiotics.


• Most of them show improvement in the pain and urethral or vaginal discharge in a day
or two.
• Pain and discharge from rectum also eases in two to three days.
• Bleeding between periods usually improved before onset of next periods.
• Pain in the lower abdomen, testicles or epididymis start to improve quickly but could
take up to two weeks to go away.

prevention of gonorrhea

Preventive methods
Adequate therapy for gonococcal infection and meticulous follow up are to be done till the
patient is declared cured.
To treat adequately the male sexual partner simultaneously.
To avoid multiple sex partners.
To use condom till both the sexual partners are free from disease.
Curative: The specific treatment for gonorrhea is single dose regimen of any one of the
following drugs

14. Treatment of patients with premalignant pathology of endometrium. Indications for


surgical treatment.
Management, it has 2 types.
Preventive
Definitive treatment
Preventive
• To maintain ideal body weight.
• Estrogen use in non-hysterectomized women should be restricted. The combined
estrogen- progestogen preparations reduce the risks than estrogen alone.
• Screening of ‘at risk’ women mentioned earlier should be done by periodic
endometrial sampling
Definitive treatment—Treatment depends on:
♦ Age of the patient
♦ Histologic type of hyperplasia
Young patient with cystic or adenomatous hyperplasia:
• Cyclic progestogen therapy for 6–9 months may be helpful. Follow-up at interval of 6 months
by endometrial sampling is essential to note whether its regression is there or not.
• Induction of ovulation in cases of PCOS to improve the fertility in a young woman, is advised.
Atypical hyperplasia requires therapy with a progestin (provera 10 mg bid) continuously
or cyclically for 6–9 months. Periodic endometrial sampling at least every 6 months
is essential.
Perimenopausal and postmenopausal women:
• Hyperplasia without atypia: Continuous progestin therapy may be considered. However,
hysterectomy with bilateral salpingo-oophorectomy is done as an alternative as the risk of
carcinoma increases with age.
• Moderate to severe atypical hyperplasia and also those women who fail to respond with
progestin therapy should be considered for hysterectomy.
Indications for surgery
as the risk of carcinoma increases with age.
women who fail to respond with progestin therapy

15. Physiological mechanisms of normal biocenosis of the vagina.

16. The cancer of ovary. Etiology, clinic, diagnostic, therapy.


Etiology
Traditionally, ovarian cancer has been suggested to originate from cells in the serosa of the
ovary. However, some authors suggest a different cell of origin. The precise cause of ovarian
cancer is unknown, but several risk and contributing factors have been identified.
Reproductive factors
• Parity is an important risk factor. Women who have been pregnant have a 50%
decreased risk for developing ovarian cancer compared to nulliparous women.
• Multiple pregnancies offer an increasingly protective effect.
• Oral contraceptive use decreases the risk of ovarian cancer.
• These factors support the theory that risk for ovarian cancer is related to ovulation
and that conditions that suppress this ovulatory cycle play a protective role.
• Ovarian cancer may develop from an abnormal repair process of the surface of the
ovary, which is ruptured and repaired during each ovulatory cycle. Therefore, the
probability of ovarian cancer may be related to the number of ovulatory cycles
Genetic factors
• Family history plays an important role in the risk of developing ovarian cancer.
• The lifetime risk for developing ovarian cancer is 1.6% in the general population.
• This compares to a 4-5% risk when 1 first-degree family member is affected, rising to
7% when 2 relatives are affected.
• A history of breast cancer increases a woman's risk of developing ovarian cancer.
Hereditary ovarian cancer
• Families in which multiple members have ovarian cancer (alone or associated with
other tumors) are defined as having hereditary ovarian cancer
Clinical features
• sense of pelvic heaviness
• vaginal bleeding - rare
• weight gain or weight loss – rare and in late stages
• abnormal menstrual cycles -rare
• unexplained back pain that worsens over time
• increased abdominal girth
• USUALLY 1-2 STAGES ARE ASYMPTOMATIC
non specific gastrointestinal symptoms:
● vague lower abdominal discomfort
● increased gas
● indigestion
● lack of appetite
● nausea and vomiting
● Bloody stool
● inability to ingest usual volumes of food
● bloating
Additional symptoms that may be associated with this
disease:
● increased urinary frequency/urgency
● excessive hair growth
● Fluid buildup in the lining around the lungs (Pleural effusions)
● Positive pregnancy readings (in the absence of pregnancy. This
is for germ cell tumors only)
diagnostic
If ovarian cancer due to a pelvic or ovarian mass is suggested, minimize preoperative testing
and expedite a diagnostic and staging laparotomy.Routine preoperative tests include CBC
count, chemistry panel (including liver function tests), and a cancer antigen 125 assay (CA-125),
HE-4, CEA. Remember that CA-125 may be within normal limits in 50% of women with early
ovarian cancer.
CA 125
• Sustained elevation in 82% of women with advanced ovarian cancer, but fewer than 1%
of healthy women
• Poor sensitivity (elevated in only 50% of women with Stage I disease)
• Poor specificity (elevated in many gynecologic and non-gynecologic malignancies as well
as benign conditions)
Routine imaging is not required in all patients in whom ovarian cancer is highly suggested. If
diagnostic uncertainty is present, a pelvic ultrasound or CT scan of the abdomen and pelvis is
warranted.
Chest radiographs are common and considered routine.
CT scan of the chest is seldom indicated
Lysophosphatidic acid (LPA)
Tumor marker being investigated for screening Phospholipid with mitogenic and
growth factor-like actions
Other test
In patients with diffuse carcinomatosis and GI symptoms, GI tract workup may be indicated,
including one of the following:
Upper and/or lower endoscopy
Barium enema
Upper GI series
Biopsy
● A fine-needle aspiration (FNA) or percutaneous biopsy of an adnexal mass is not routinely
recommended. In most cases, taking this approach instead of performing a surgical staging
laparotomy may only serve to delay appropriate diagnosis and treatment of ovarian cancer.
● If a clinical suggestion of ovarian cancer is present, the patient should undergo a diagnostic
and surgical procedure.An FNA or diagnostic paracentesis should be performed in patients with
diffuse
carcinomatosis or ascites without an obvious ovarian mass.

TREATMENT
Medical Care: The standard treatment for ovarian cancer starts with staging and cytoreductive
surgery. Based on the surgical staging, patients are classified as having limited disease (stage I
and II) or advanced disease (stage III and IV).
Patients with limited disease are classified as having low or high risk for recurrence as follows:
Low risk for recurrence includes the following:
• Grade 1 or 2 disease
• No tumor on external surface of the ovary
• Negative peritoneal cytology
• No ascites
• Tumor growth confined to the ovaries
High risk for recurrence includes the following:
• Grade 3 disease
• Preoperative rupture of the capsule
• Tumor on the external surface of the ovary
• Positive peritoneal cytology
• Ascites
• Tumor growth outside of the ovary
• Clear cell tumors
• Surgical stage II
For postoperative treatment, chemotherapy is indicated in all patients with ovarian cancer
except those patients with surgical-pathological stage I disease with low-risk characteristics
• Surgical treatment (standart)
• Total or subtotal Hysterectomy with BOTH ovaries and subtotal resection of
omentus major. In young patients (below 35 in 1-st stage ) may be done unilateral
adnexectomia with byopsy of 2-nd ovary

Surgical Care: The standard care for ovarian cancer includes a primary staging and
cytoreductive or debulking surgical exploration.
Surgical staging
If the disease appears to be confined to the pelvis, comprehensive surgical staging is indicated.
The staging procedure should include (1) peritoneal
cytology, (2) multiple peritoneal biopsies, (3) omentectomy, and (4) pelvic and para-aortic
lymph node sampling.
Cytoreductive surgery - This should be performed by a gynecological oncologist at
the time of initial laparotomy. The volume of residual disease at the completion of surgery
represents one of the most powerful prognostic factors.
CONSERVATIVE TREAMENT
Chemotherapy ( individually drug, dosage and quantity of courses)
Large doses of progesterone in cases of sensitivity of tumour sells (determined in
histochemistry investigation)

17. Endometriosis. Surgical methods of treatment.


Definition: Presence of functioning endometrium (glands and stroma) in sites other than
uterine mucosa is called endometriosis. It is not a neoplastic condition, although malignant
transformation is possible. These ectopic endometrial tissues may be found in the
myometrium when it is called endometriosis interna or adenomyosis . More commonly,
however, these tissues are found at sites other than uterus and are called endometriosis
externa or generally referred to as endometriosis
SURGICAL MANAGEMENT OF ENDOMETRIOSIS
Indications
-Endometriosis with severe symptoms unresponsive to hormone therapy.
-Severe and deeply infiltrating endometriosis to correct the distortion of pelvic anatomy.
- Endometriomas of more than 1 cm.
Surgery may be conservative or definitive
-Conservative surgery is planned to destroy the endometriotic lesions in an attempt to
improve the symptoms (pain, subfertility) and at the same time to preserve the reproductive
function.Laparoscopy is commonly done to destroy endometriotic lesions by excision or
ablation by electrodiatherapy or by laser vaporization.Conservative surgical treatment in
minimal to mild endometriosis (ablation plus adhesiolysis) improves the fertility outcome.
Laparoscopic uterosacral nerve ablation (LUNA) is done when pain is very severe. The
advantage of laser is to cut the tissues precisely with least chance of damage to the underlying
vital structures. Great deal of technical expertise is essential to avoid injury to the ureters.
Surgical treatment improves fertility and symptoms in women with moderate and severe
endometriosis
- Definitive surgery: It is indicated in women withadvanced stage endometriosis where there is:
(i) No prospect for fertility improvement. (ii) Other forms of treatment have failed. (iii) Women
with completed family. Definitive surgery means hysterectomy with bilateral savlpingo-
oophorectomy along with resection of the endometrial tissues as complete as possible.

- Combined Medical and Surgical: Preoperative hormonal therapy aims at reduction of the size
and vascularity of the lesion which facilitate surgery. The idea of postoperative hormonal therapy
is to destroy the residual lesions left behind after surgery and to control the pain. But it does not
improve fertility. It is generally avoided. Duration of therapy is usually 3–6 months preoperatively
and 3–6 months postoperatively. The cumulative probability of pregnancy at 3 years following
laparoscopic surgery was 47% (51% for stage I, 45% of stage II, 46% of stage III and 44% for stage
IV). Overall risk of recurrence is 40% by 5 years time.
- Emperical treatment of pelvic pain with the presumptive diagnosis of pelvic endometriosis
may be given with combined oral contraceptives

18. Inner sexual woman’s organs, structure and function.


Vagina is a fibromuscular (made up of fibrous and muscular tissue) canal leading from the
outside of the body to the cervix of the uterus or womb. It is also referred to as the birth canal
in the context of pregnancy. The vagina accommodates the male penis during sexual
intercourse. Semen containing spermatazoa is ejaculated from the male at orgasm, into the
vagina potentially enabling fertilization of the egg cell (ovum) to take place.
The cervix is the neck of the uterus, the lower, narrow portion where it joins with the upper
part of the vagina. It is cylindrical or conical in shape and protrudes through the upper
anterior vaginal wall. Approximately half its length is visible, the remainder lies above the
vagina beyond view. The vagina has a thick layer outside and it is the opening where the fetus
emerges during delivery.
The uterus or womb is the major female reproductive organ. The uterus provides mechanical
protection, nutritional support, and waste removal for the developing embryo (weeks 1 to 8)
and fetus (from week 9 until the delivery). In addition, contractions in the muscular wall of
the uterus are important in pushing out the fetus at the time of birth.
Fallopian tube
The Fallopian tubes are two tubes leading from the ovaries into the uterus. On maturity of an
ovum, the follicle and the ovary's wall rupture, allowing the ovum to escape and enter the
Fallopian tube. There it travels toward the uterus, pushed along by movements of cilia on the
inner lining of the tubes. This trip takes hours or days. If the ovum is fertilized while in the
Fallopian tube, then it normally implants in the endometrium when it reaches the uterus,
which signals the beginning of pregnancy.
Ovaries
The ovaries are small, paired organs located near the lateral walls of the pelvic cavity. These
organs are responsible for the production of the egg cells (ova) and the secretion of
hormones. The process by which the egg cell (ovum) is released is called ovulation. The speed
of ovulation is periodic and impacts directly to the length of a menstrual cycle.
19.precancerous diseases of endometrium, etiology and and pathogenesis ,clinic ,diagnostic
therapy
Endometrial hyperplasia
Etioloogy
• unopposed estrogen
• Pre-menopausal persistent anovulation
• Peripheral conversion of androgens into estrogen
• polycystic ovarian syndrome
• drug therpay- tamoxifen
• nulliparity
• delayed menopause
• diabetes
• obesity
pathogenesis
• caused by increased amounts of estrogen unopposed by progesterone
clinical
• no classical symptoms
• abnormal perimenopausal uterine bleeding
diagnostics
• diagnosis is by uterine curettage and histology.
• Accidental diagnosis is made during investigation of infertility, DUB, PCOS or excised
specimen of removed uterus
• Vaginal pool smear
• endometrial aspiration (pipelle endometrial sampling)
• endometrial biopsy (curettage) and vaginal ultrasound are the different methods
available for screening.
Therapy
• Preventive
o To maintain ideal body weight.
o Estrogen use in non-hysterectomized women should be restricted. The
combined estrogen- progestogen preparations reduce the risks than estrogen
alone.
• Definitive treatment- Treatment depends on:
♦ Age of the patient
♦ Histologic type of hyperplasia
• Young patient with cystic or adenomatous
hyperplasia
o Cyclic progestogen therapy
o Induction of ovulation in cases of PCOS
o Atypical hyperplasia requires therapy with a progestin
• Perimenopausal and postmenopausal women
o Hyperplasia without atypia: Continuous progestin therapy may be considered
o hysterectomy with bilateral salpingo-oophorectomy

• endometrial polyps
o Are benign localized overgrowths that project from the endometrial surface into
the enometrial cavity
o Most occur during perimenopausal period
o Though to arise from endometrial foci that are hypersensitive to estrogen or do
not responsive to progesterone; therefore would not slough but would continue
to grow
• Clinical symptoms
o Usually present with bleeding due to ulcerated surface or hemorrhagic infarction
• Diagnosis
o Ultrasound
o Sagittal sonohysterogram
• Treatment
o Curettage and biopsy
o Hysterectomy in post menopausal women
20.septic abortion, sepsis, septic shock
Septic abortion refers to any abortion, spontaneous or induced, that is complicated by uterine
infection , including endometritis. Septic abortion typically refers to pregnancies of less than 20
weeks gestation while those ≥20 weeks gestation with intrauterine infection are described as
having intraamniotic infection.
Sepsis- Sepsis is the body's extreme response to an infection . It is a life-threatening medical
emergency. Sepsis happens when an infection you already have triggers a chain reaction
throughout your body.
Septic shock is a life-threatening condition that happens when your blood pressure drops to a
dangerously low level after an infection
etiology,
is commonly seen in menstruating women between 15 and 30 years of age following the use of
tampons (polyacrylate). Other condition associated with TSS is use of female barrier
contraceptives (diaphragm). It is characterized by the following
features of abrupt onset :
The most common cause is post-operative endometritis (85%)

pathogenesis,
The pathological features are due to liberation of exotoxin by Staphylococcus aureus . It may lead
to multiorgan system failure. Blood cultures are negative. Sepsis triggers systemic inflammatory
response
Endotoxin i.e. Complex lipopolysaccharide in the cell wall of Gram -ve bacteria and lipid A initiate
activation of coagulation, fibrinolysis, complement, prostaglandin and kinin systems
• Activation of coagulation and fibrinolysis lead to consumptive coagulopathy
Complement activation leads to release by mediators by leucocytes causing the damage of
vascular endothelium, platelet aggregation, intensification of the coagulation cascade, and
degranulation of mast cells with histamine release
Histamine cause increase capillary permeability, decreased plasma volume, vasodilation and
hypotension
Release of bradykinin and β-endorphins also causes systemic hypotension
Tumor necrosis factor may lead to depressed myocardial function during septic shock
Clinical signs and symptoms
Fever >38.9°C.
xx Diffuse macular rash, myalgia.
xx Gastrointestinal : Vomiting, diarrhea.
xx Cardiopulmonary : Hypotension, adult respiratory
distress syndrome.
xx Platelets : < 100,000/mm3 .
xx Renal : ↑ BUN (> twice normal).
xx Hepatic : Bilirubin, SGOT, SGPT rise twice the
normal level.
xx Mucous membrane (vaginal, oropharyngeal) :
Hyperemia

management
Principles of management: A rational approach to the management of endotoxic shock can be
formulated only on the basis
of the pathological changes produced by endotoxemia.
This includes administration of antibiotics, intravenous fluids, adjustment of acid base balance,
steroids, inotropes,
prevention and treatment of intravascular coagulation and toxic myocarditis, administration of
oxygen and elimination of the source of infection.
Antibiotics
Intravenous fluids and electrolytes
Correction of acidosis:
Maintenance of blood pressure
Vasodilator therapy
Diuretic therapy
Corticosteroids
Treatment of diffuse intravascular coagulation
Treatment of myocarditis
Elimination of source of infection
Intensive insulin therapy

21.gonadotrophic and ovarian hormones and their interaction


• Female gonadotrophic hormones are GnRH, FSH and LH Female ovarian hormones are
Estrogen and Progesteron
• GnRH, on reaching the anterior lobe of the pituitary,stimulates the synthesis and release
of both LH and FSH from the same cell (gonadotrophs) in the pituitary gland.
• Lower GnRH pulse frequencies favor FSH secretion and the higher frequencies favor LH
secretion. This response has been termed ‘self-priming’. Estrogen preferentially induces
more LH than FSH release.
• GnRH stimulate the release of stored FSH and LH and their subsequent
biosynthesis.However, if GnRH continues to be infused, gon-adotropin secretion is
inhibited, probably because there ceptors are saturated and are unable to stimulate the
release of second messenger. This is known as desensitization or down regulation
• Anterior pitutarysecretes FSH and LH in response to GnRH
• The function of FSH is predominantly morphogenic, related to the growth and
maturation of the Graafian follicle.
Morphological effects -FSH
• FSH rescues follicles from apoptosis.
• Stimulates formation of follicular vesicles (antral follicle).
• Stimulates proliferation of granulosa cells.
• Helps full maturation of the Graafian follicle (dominant follicle) as it converts the
follicular microenvironment from androgen dominated to estrogen dominated
Biochemical effects- FSH
• y Synthesizes its own receptors in the granulosa cells.
• y Synthesizes LH receptors in the theca cells.
• y Synthesizes LH receptors in the granulosa cells.
• y Induces aromatization to convert androgens to estrogens in granulosa cells (see p. 72).
• y Enhances autocrine and paracrine function (IGF-II, IGF-I) in the follicle.
• y Stimulates granulosa cells to produce activin and inhibin.
• y Stimulates plasminogen activator necessary for ovulation.
Biochemical effects-LH
• Activation of LH receptors in the theca cells whichstimulates the enzymes necessary for
androgen production → diffuse into the granulosa cells →estrogens
• Luteinization of the granulosa cells → to secrete progesterone.
• Synthesizes prostaglandins.
Morphological effects -FSH
• Stimulates resumption of meiosis with extrusion of first polar body (see p. 82).
• Helps in the physical act of ovulation (see p. 83).
• Formation and maintenance of corpus luteum
OvAriAn STerOidOGeneSiS
The principal hormones secreted from the ovaries are—(i) Estrogens; (ii) Progesterone;(iii)
Androgens; and (iv) Inhibin
Estrogen physiological action
Secondary sex characters- induce feminine characteristics. The hormone isresponsible for
feminine body configuration and feminine mental make up including shyness. hereis secretion
in apocrine glands, change in voice and deposition of fat on the breasts, thighs and hips.
Thegrowth of axillary and pubic hair is dependent predominantly on androgens of adrenal
origin.
Action on the genital organs—Under the action of estrogen, the genital organs not only
develop into maturity but induce cyclic changes for reproduction
Vulva and vagina-Estrogen induces thickening of the lining epithelium, cornification of the
superficial cells
Uterus-Changes the uterus from the infantile to adult form. Cyclic changes in the endometrium
includes regeneration and proliferation of the endometrium.
Cervix-hypertrophy of the cervix and increases the cervical gland secretion.

Progesterone physiological actions


Uterus—Progesterone produces myohyperplasia and diminishes the contractility of the
myometrium
Vagina—The maturation of the vaginal epithelium is hindered.
Fallopian tubes—The epithelial cells are stimulated to secrete clear mucus which helps in
migration of the ovum.
Breasts—Along with estrogen, it produces hypertrophy and growth of the acinar structures

22.TB of sexual organs featues of clinic diagnostic and therapy influence on reproductive
function

Fallopian tube: The commonest site of affection is the fallopian tubes (100%). Both the tubes
area affected simultaneously. The initial site of infection
Uterus: The endometrium is involved in 60 percent of cases. Endometrial ulceration may lead
to adhesion or synechiae formation (Asherman ’s syndrome).This may cause infertility,
secondary amenorrhea or recurrent abortion
Cervix: The cervical affection is not so uncommon(5–15%). May be ulcerative or may be bright
nodular in type. Both may bleed to touch, thereby causing confusion with carcinoma
Ovary: The ovaries are involved in about 30 per centof tubercular salpingitis. The manifestation
may be surface tubercles, adhesions, thickening of the capsule.

Pelvic peritonitis is present in about 40–50 percent of cases. Tuberculous peritonitis may be
‘wet’ (exudative type) or ‘dry’ (adhesive type

Symptoms
• Infertility
• Menstrual abnormality(menorrhagia, amenorrhea or oligomenorrhea)
• Chronic pelvic pain
• Vaginal discharge- post coital, blood stained discharge
• Health status- There may be constitutional symptoms like weakness, low grade fever,
anorexia, anemia and night sweats.
Investigations
• Blood- esr and Leukocytes increased
• Mantoux test- positive reaction
• Chest x-ray -healed or active pulmonary lesion
• Diagnostic uterine curettage
• Nucleic acid amplification (16S ribosomal DNA) techniques with Polymerase Chain
Reaction (PCR),can identify M. tuberculosis from endometrium or menstrual blood
• First day menstrual discharge
• Sputum and urine
• Lymph node biopsy
• Hysterosalphingography(HSG)
• Laporoscopy
Therapy- general, chemo, surgery
Chemotherapy-
• Initial phase:Four drugs are used for 2 months to reducethe bacterial population and to
prevent emergence of drug-resistance. The drugs used are isoniazid,
rifampicin,pyrazinamide and ethambutol. Ethambutol is essential to those who have
been treated previously or are immunocompromised (HIV positive individual).
• Continuation phase: Treatment is continued for a period of further 4 months with
isoniazid and rifampicin

After about a year of treatment, diagnostic endometrial curettage is to be done.


• A patient may be considered cured, if at least two reports including histological and
bacteriological examination become negative.
• The ideal surgery should be total hysterectomy with bilateral salpingo-oophorectomy. In
young women at least one ovary, if found apparently healthy, should be preserved.
23.Rupture of tubo-ovarian abscess. Clinical signs, diagnosis, treatment.
Pelvioperitonities. Peritonitis.

Clinical signs:
The signs and symptoms of tubo-ovarian abscess (TOA) are the same as with pelvic
inflammatory disease (PID) with the exception that it can create symptoms of acute-onset
pelvic pain. Tubo-ovarian abscess can mimic abdominal tumours.
But the localization of pus in the pouch of Douglas is evidenced by:
Symptoms
Spiky rise of high temperature with chills and rigor.
Rectal tenesmus—frequent passage of loose
mucoid stool.
Pain lower abdomen—variable degrees.
Urinary symptoms—difficulty or even retention of urine.
Signs:
General: The face is flushed with anxious look. Pulse rate is raised out of proportion to
temperature.
Per abdomen:
Tenderness and rigidity in lower abdomen.
A mass may be felt in the suprapubic region — tender, irregular, soft, and resonant on
percussion.
Per vaginam:
The vagina is hot and tender.
The uterus is pushed anteriorly; the movement of the cervix is painful.
A boggy, fluctuant, and tender mass is felt in the pouch of Douglas.
A separate mass may be felt through the lateral fornix.
Rectal examination defines precisely the mass in the pouch of Douglas.

Diagnosis:

Blood: There is high leukocytosis with increased polymorphs.


Bacteriological study: Swabs are taken from high vagina, endocervical canal and from the
pus. Culture is done for both aerobic and anaerobic microorganisms.
Sensitivity of the microorganisms to antibiotics is also to be detected.
Pelvic ultrasonography reveals accumulation of fluid in the pouch of Douglas.
Examination under anesthesia (EUA) and puncture of pouch of Douglas (culdocentesis) give
the correct diagnosis. Old blood comes out in the former and pus in the latter.

Treatment:
General: Systemic antibiotics should cover anaerobic as well as aerobic microorganisms
(broad spectrum):
Cefoxitin 1–2 gm IV every 6–8 hours and gentamicin 2 mg/kg IV per 24 hours and
metronidazole 500 mg IV 8 hourly are started.
Antibiotic regimen may have to be changed depending upon the sensitivity report.
Surgery: Posterior colpotomy is the definitive surgery to drain the pus through posterior
fornix. The loculi should be broken with finger.
Laparotomy is done when the patient’s condition deteriorates despite aggressive
management. In patients with recurrent infection and with loss of reproductive function
total abdominal hysterectomy with bilateral salpingo-oophorectomy is the preferred
treatment.

Pelvic peritonitis:
Tuberculous peritonitis may be ‘wet’ (Exudative type) or ‘dry’ (adhesive type).
In the ‘wet’ variety there is ascites with straw coloured fluid in the peritoneal cavity. The
parietal and visceral peritoneum are covered with numerous small tubercles
In the ‘dry’ variety there is dense adhesion with bowel loops. The adhesion is due to fibrosis
when the ‘wet’ variety heals.
24.Anatomical and physiological features of sexual organs of women (different age periods).

Female reproductive system- immature at birth


reproductive organs are developed from the intermediate mesoderm

pregnancy- physiologic activity increases


changes in uterus
1. Size - increase from 7.5 x 5 x 2.5 cm in nonpregnant states to 35 x 25 x 20 cm at term i.e.
the volume increase 1000 time
2. Weight - increases from 50 gm in nonpregnant state to 1000 gm at term
3. Shape - pyriform in the nonpregnant state , becomes globular at 8th week, then pyriform
by 16th week till term
4. Position - with ascent from the pelvis , the uterus usually undergoes rotation with tilting
to the right (dextrorotation) due to the presence of the rectosegmoid colon on the left
side.
5. Consistency : becomes progressively softer due to :
-Increased vascularity
-Presence of amniotic fluid
6. contractility from the 1st trimester onwards , the uterus undergoes irregular painless
contractions (Braxton hicks contractions0 they may cause some discomfort late in pregnancy and
may account for false labour pain
7. capacity – increases from 4ml in non pregnant state o 4000ml at term

Myometrium- hypertrophy (estrogen effect) rather than hyperplasia (progesterone effect) till
14th week then the fetus exerts a direct stretch
Formation of lower uterine segment from the isthmus and lower half inch of the body
Cervix-
1.It becomes hypertrophied, soft and bluish in color due to edema and increased vascularity.
2. Soon after conception , a thick cervical secretion obstructs the cervical canal forming a mucous
plug .
3.The endocervical epithelium proliferates and or everted forming cervical ectopy (previously
called erosion)
Changes in fallopian tubes and ligaments (round and broad)- Inactive , elongated , marked
increase in vascularity
There may be broad ligament varicose veins
Vagina- becomes soft , warm , moist with increased secretion and violet in colour (Chadwick's
sign) due to increased vascularity
Vulva-It becomes soft, violet in color
Edema and varicosities may develop
Ovaries- Both ovaries are enlarged due to increased vascularity and edema particularly the ovary
which contains the corpus luteum .
Corpus luteum continues to grow till 7 - 8 weeks , then it stops growing, it becomes inactive and
starts degeneration at 12 weeks (degeneration is completed after labor)
Ovulation ceases during pregnancy due to pituitary inhibition by the high levels of estrogen and
progesterone
25.The main types of gynecological surgeries

In elective operations, the patient should be made fit for surgery prior-hand. Even in minor
surgery, examination of the cardiovascular system, complete hemogram and complete urine
examination should at least be done. Preoperative risk assessment is essential to minimize
surgical morbidity and other complications

Preoperative work up in the operation table

-IV infusion—An infusion of Ringer’s solution


drip is started.

- Anesthesia—Local, regional or general anes


thesia is administered with sole discretion of the
anesthetist.

- Position of the patient—In abdominal operation,


the position is dorsal, whereas in vaginal operation, it is lithotomy.

- Antiseptic dressings: Bladder preparations: For minor operations the patient voids before
being taken to the operating room. For major operations, soft rubber catheter or Foley’s
catheter is inser-ted in operating table. In vaginal operation, metal catheter is used after
draping and Foley’s catheter is introduced at the end of the operation and (see
Draping—Proper draping is done prior to surgery
using sterile linen, towel and leggings (in vaginal
operation). Towel clips are used

List of Gynaecological Operations


1-Minor Operations:  Diagnostic curettage including Polypectomy, Endometrial Ablation 
Therapeutic abortions (suction evacuation).

2-Major Operations:  Total hysterectomy +/- bilateral/unilateral salpingo- oophorectomy. 


Subtotal hysterectomy.  Myomectomy.  Ovarian cystectomy.  Oophorectomy/salpingo-
oophorectomy  Salpingectomy.

Dilatation & Curettage D & C:  It is the most common minor gynecologic surgical procedure,
used as diagnostic or therapeutic.
Indications: 1. Abnormal uterine bleeding. 2. Postmenopausal bleeding. 3-Endometrial
hyperplasia with heavy bleeding . 4. Removal of endometrial polyps or small pedunculated
myomas. 5. Dilatation & evacuation in inevitable and missed abortion. 4. Removal of missed
intrauterine IUCD.

Technique  1.Evacuate the bladder.  2.Anesthesia.  3.Vaginal speculum & grasp the cervix.
 4.Sounding.  5.Dilate the cervix.  6.Curette

Hysterectomy can be abdominal, vaginal, laparoscopic or laparoscopic assisted vaginal.

ABDOMINAL HYSTERECTOMY

Hysterectomy is the operation of removal of uterus. When the uterus is removed abdominally,
it is called abdominal hysterectomy.

Types: Depending upon the extent of removal of the uterus and adjacent structures, the
following types are described:

⚫ Total hysterectomy—Removal of the entire uterus.


Subtotal—Removal of the body or corpus leaving behind the cervix.
⚫ Panhysterectomy—Removal of the uterus along with removal of tubes and ovaries of both
sides. The term ‘hysterectomy with bilateral salpingooophorectomy’ is preferred.
⚫ Extended hysterectomy—Panhysterectomy with removal of cuff of vagina.
⚫ Radical hysterectomy—Removal of the uterus,tubes and ovaries of both the sides, upper
one- third of vagina, adjacent parametrium and the draining lymph nodes of the cervix
indcations
Total- • DUB • Uterine fibroid . TO mass • Endometriosis
Subtotal-• Difficult TO mass • Endometriosis (rectovaginal septum)• Obstetric causes

Radical hysterectomy 

. With this procedure only a selective group of lymph nodes (e.g. enlarged and palpable) and
only the medial half of the cardinal and uterosacral ligaments were removed.

The classic radical hysterectomy (Type IV) is performed in most centers these days. Tissues
removed in this operation include wide resection of the parametrium, periureteral tissue,
superior vesical artery, cardinal and uterosacral ligaments, upper three-fourth of vagina and
thorough pelvic lymphadenectomy .
indicatons
A. Carcinoma cervix: Stage IAI (lymphovascular space
involvement): IA2, IBI, IB2, IIA (selected)
B. Endometrial carcinoma : Stage IIB
C. Vaginal carcinoma : Stage I–II (Limited to upper one-third vagina)
D. Recurrence of cervical cancer after radiotherapy :
Growth limited to cervix and upper vagina

SALPINGECTOMY
one pair of long hemostatic forceps is placed on the medial end of the fallopian tube including
the mesosalpinx as close to the uterus. A second pair of clamp is placed from the lateral aspect
on the mesosalpinx. The clamp tips are to be approximated. The tube is excised and the clamps
are replaced by ligatures. The excised tube is to be sent for histology.

Ovarian cystectomy
Removal of the ovarian tumor leaving behind the healthy ovarian tissue is called ovarian
cystectomy.
It is the operation of choice especially when both the ovaries are involved with benign
neoplasm in young women
Steps of ovarian cystectomy
(1) Line of incision; (2) Enucleation of the tumor; (3) Closure of the ovarian incision
Ovariotomy
: Removal of the tumor along with healthy ovarian tissue is called ovariotomy. The term is
better replaced by oophorectomy. This is indicated when the tumor is big or complicated by
torsion or hemorrhage and the other ovary is healthy.
Steps of ovariotomy—1. Clamps are placed on either side of the ovarian pedicle; 2. Removal of
tumor with the clamps placed over the pedicle; 3. The clamps are replaced by sutures, the
lateral one by transfixing; (4) Look at the operation site

Wedge resection: A wedge of ovarian tissue with the base on the surface and the apex
extending to medulla is removed in PCOS when medical treatment fails to induce ovulation.
About one-third of the ovarian tissue is removed by the wedge method. This operation is not
commonly done these days.
ABDOMINAL MYOMECTOMY

Uterine incision — a single incision (linear or elliptical) in the midline on the anterior wall of the
uterus is preferred.
This has the following advantages:

− Avoids any injury to the tube and ovary


− Lateral fibroids are removed by tunnelling without any additional scar on the uterine surface.
− Fibroids from the cavity or posterior wall can be removed by transcavity approach. This avoids
scar
on the posterior wall and adhesion formation.

Actual steps

Incision is deepened through the myometrium and the


(pseudo) capsule, till the myoma is reached.

The myoma is grasped with a single toothed vulsellum and dissection is continued in the plane
between the myoma and the capsule (to minimize blood loss). Myoma is enucleated
(intracapsular) from its bed by sharp (scissors) and blunt (knife handle) dissection.

The myoma bed (deep space) is obliterated by interrupted mattress or figure-of-eight sutures.
Sometimes layers of sutures (tire stitch) may be required to approximate the myometrium

-Laparoscopic ovarian drilling is more commonly done compared to wedge resection in the
management of PCOS cases Operations for chronic inversion of the uterus may be abdominal
(Haultain) or vaginal (Kustner or Spinelli)

Vagina-
Birth upto10-14 days –epithelium is stratified squamous under the influence of maternal
estrogen
Up to pre puberty to post menopause- epithelium become thin; consisting few layers
Stratified squamous epithelium

uterus – at birth uterus lies in false pelvis


-cervix is longer than the body cervix : body 2:1

Puberty-body growing faster under action of estrogen 1:2


Child bearing – 1:3

After menopause- uterus atrophied , overall length reduce, walls become thinner, less muscular
more fibrous
26. Hormonal methods of contraception and mode of their action. Use of hormonal
contraceptives for treatment of the gynecological pathology(chapter 29 ,page 485)
Steroidal contraceptions
• Estrogen to a minimum of 20 µg or even 15 µg in the tablet
combined oral contraceptives (PIllS)
• The combined oral steroidal contraceptives is the most effective reversible method of
contraception. In the combination pill, the commonly used progestins are either
levonorgestrel or norethisterone or desogestrel and the estrogens are principally
confined to either ethinyl-estradiol or menstranol (3 methylether of ethinyl-estradiol).
Currently ‘lipid friendly’,

• Third generation progestins, namely desogestrel, gestodene,norgestimate are available.

• 4th generation: Drospirenone which is an analogue of spironolactone is used as


progestin. It has antiandrogenic and antimineralocorticoid action. It causes retention of
K+.

Mode of action
The probable mechanism of contraception are:

x Inhibition of ovulation — Both the hormones synergistically act on the


hypothalamopituitary axis. The release of gonadotropin releasing hormones from the
hypothalamus is prevented through a negative feedback mechanism. There
is thus no peak release of FSH and LH from the anterior pituitary. So follicular growth is
either not initiated or if initiated, recruitment does not occur.

x Producing static endometrial hypoplasia—There is stromal edema, decidual reaction


and regression of the glands making endometrium nonreceptive to the embryo.

x Alteration of the character of the cervical


mucus (thick, viscid and scanty) so as to prevent sperm penetration.

x Probably interferes with tubal motility and alters


tubal transport. Thus, even though accidental breakthrough ovulation occurs, the other
mechanisms prevent conception.

X Estrogen inhibits FSH rise and prevents follicular growth. It is also useful for better
cycle control and to prevent breakthrough bleeding.

X Progestin: Anovulatory effect is primarily by inhibiting LH surge. It is also helpful to


counteract the adverse effects of estrogen on the endometrium (endometrial
hyperplasia and heavy withdrawal bleeding). It is also responsible for changes in the
cervical mucus (vide supra)

Use of hormonal contraceptives for treatment of the gynecological pathology

Improvement of menstrual abnormalities—


(1) Regulation of menstrual cycle
(2) Reduction of dysmenorrhea (40%)
(3) Reduction of menorrhagia (50%)
(4) Reduction of premenstrual tension syndrome (PMS)
(5) Reduction of Mittelschmerz’s syndrome
(6) Protection against iron-deficiency anemia.

Protection against health disorders —


(7) Pelvic inflammatory disease (thick cervical mucus)
(8) Ectopic pregnancy
(9) Endometriosis
(10) Fibroid uterus
(11) Hirsutism and acne
(12) Functional ovarian cysts
(13) Benign breast disease

Prevention of malignancies—
(14) Endometrial cancer (50%)
(15) Epithelial ovarian cancer (50%)
(16) Colorectal cancer (40%)

27.Endoscopic methods in gynecology. Indications.


Gynaecological endoscopy: laparoscopy
Hysteroscopy
Salpingoscopy
Falloposcopy
Cyctoscopy
Culdoscopy
Sigmoidoscopy & proctoscopy
method Indications diagnostic Advantages of diadvantag Complications
laparoscopic es
surgery
Laparoscopy- Diagnostic Infertility work up -Rapid post -operative 1.extraperitoneal
visualization of Therapeutic - Peritubal operative time may insufflation
peritoneal caity by adhesions recovery be longer -surgical
means of fiberoptic - Minimal -Less post -risk of emphysema
endoscope endometriosis operative pain, iatrogenic -omental
introduced into the - Ovulation reduce need of complicatio emphysema
abdominal wall, prior stigma of the post operative ns -cardiac
pneumoperitoneum ovary anaelgesia -high initial arrhythmia
is achieved Chronic pelvic pain -Shorter expenditur 2. injury to blood
Nature of pelvic mass: hospital stay e vessels
fibroid , ovarian cyst -quicker -surgeons 3.injury to bowel
To diagnose acute resumption into need 4.eletro surgical
pelvic lesion day to day specialized complications-
- Ectopic activity training causing thermal
pregnancy -less adhesion and injury
- Acute formation expertise 5. gas embolism –
appendicitis - resulting
- Acute -reduced blood complicatio hypotension,
salphingitis loss ns are cardiac arrhytmias
Follow up of pelvic -minimal specific to
surgery abdominal scar laparoscop
- Tuboplasty y and may
- Ovarian be fatal
malignancy
Uterine perforation
Investigation of
protocol of
amenorrhea
Hysteroscopy Diagnostic Unresponsive irregular - - Preoperative:
Operative uterine bleeding to Distension media
exclude uterine polyp, -fluid overload
submucous fibroid or -pulmonary edema
product of -hyponatraemia
contraception -neurological
-congenital uterine symptoms
septum in recurrent Operative
abortion procedure
-missing thread if IUD -uterine
-intrauterine adhesions perforaton
-hemorrhages
-injury to intra
abdominal organs
Late
complications:
-abnormal uterine
bleeding
-haematometra
and pyometra
salpingoscopy Study - - - -
physiology and
anatomy of
tubal
epithelium and
permits more
accurate
selection of
patient for IVF
rather than
tubal surgery
Falloscopy To study the - Helps to direct - -
entire length of visualization of
tubal lumen tubal ostia,
Is performed mucosal
through pattern , intra
uterine cavity , tubal polyps &
using debris
hysteroscopy

Cystoscopy To evaluate - - - -
cervical cancer
prior to staging
& investigate
urinary
symptoms
including
hematuria ,
incontinence,
fistula

culdoscopy To visualize the - - - -


pelvic
structures
through the
incision in the
pouch of
douglas
Proctoscopy & For rectal - - - -
sigmoidoscopy involvement of
genital
malignancy

28. uterine bleeding of adolescent period. Etiology, pathophysiology, clinic, diagnostic,


principal of therapy
Types of uterine bleeding

1.Metrorrhagia
Irregular, frequent bleeding,But not excessive in amount
2.Menorrhagia
Prolonged or excessive bleeding ,At regular interval
3.Meno-metrorrhagia
Prolonged or excessive,At irregular interval
4.Polymenorrhea
At regular interval but more than normal frequency,Less than 21days per cycle
5. Oligomenorrhea
➢ At regular interval but decreased in frequency
➢ More than 35 days per cycle
6. Hypermenorrhea
➢ Menstrual bleeding more than 80 ml
7. Hypomenorrhea
➢ Menstrual bleeding less than 30 ml
8. Inter-menstruation
➢ Bleeding between regular menstrual cycle
9.Postmenopausal bleeding
➢ Uterine bleeding that occurs more than 1 year after the last menses in a woman with
ovarian failure
10. Amenorrhea
➢ Absent of menstruation for more than 6 months

Etiology of abnormal uterine bleeding

1. Complication of pregnancy
Ectopic pregnancy
Spontaneous abortion (threatened, incomplete, missed)
Placental pathology ( placenta previa, placenta abruption
Gestational trophoblastic disease
➢ Placental polyp
➢ Subinvolution of the placental site

2. Infection
Cervicitis
Endometritis

3. Trauma
Laceration, perforation, abrasion
Foreign body
4.Malignant neoplasm
Cervical
Endometrial
Vaginal
5.Benign pelvic pathology
Myoma
Polyps
6.Puberty / Perimenopausal for systemic disease
Leukemia
Renal
Coagulopathy
7.Medication / Iatrogenic
Operative injuries
Pathophysiology
Pathophysiology is defended on etiology of the bleeding; all causes of bleeding may be

Organic type

Eg. Fibroid, tumour, polyps, myoma

Produce symptoms like hypermenorrhea, menorrhagia


Pathogenesis :-
1.Fibroid must grow towards uterine cavity

2.Resulting protrusion and distortion of cavity

3.Compression and congestive alteration

4.Result excessive bleeding


Functional type
Due to disturbed hypothalamo-pituitary-ovarian- endometrial axis.
1.Corpus luteum fails to form
2.Failure of normal cyclic progesterone secretion
3.Continuous unopposed estradiol production
4.Overgrowth of endometrium
5.Without pregnancy, endometrium proliferates
6.Excessive outgrow of blood supply
7.Necrosis
8.Overproduction of uterine blood flow

Investigations
A.Laboratory

perform a CBC with platelets


prothrombin time ( PT )
activated partial thromboplastin time ( aPTT )
liver function tests ( if other signs indicate liver disease
Pregnancy must be ruled out by urine and/or serum human chorionic gonadotropin, before
consideration of any imaging studies
Consider thyroid function tests. FSH, TSH, DHEAS, and prolactin levels
Tests to determine ovulatory status : Because hormonal irregularities can contribute to
abnormal bleeding, ovulatory testing may be recommended

B. Instrumental
Endometrial biopsy
Tests that assess the endometrium (lining of the uterus) to rule out endometrial cancer and
structural abnormalities such as uterine fibroids or polyps.
Perform endometrial biopsy for the following patients:
All patients older than 35 years
Obese patients
Patients with diabetes mellitus
Patients with hypertension Patients with suspected polycystic ovarian disease
Transvaginal ultrasound
Consider if the patient may be pregnant or may have anatomic problems or polycystic ovarian
syndrome
An ultrasound uses sound waves to measure an organ's shape and structure.
Ultrasound cannot distinguish between different types of abnormalities ( eg, polyps versus
cancer ) and further testing may be necessary.

A computed tomography (CT) scan or magnetic resonance image (MRI)


• are non-invasive tests that are sometimes used to determine if fibroids or other
structural abnormalities of the uterus are present.
Hysteroscopy
During hysteroscopy, a small scope is inserted through the cervix and into the uterus. Tissue
samples may be taken. In most cases, hysteroscopy is performed along with a D&C
Dilation and curettage (D&C)
It can be both therapeutic and diagnostic. It may be the treatment of choice when bleeding is
severe, and it allows more extensive sampling of the uterine cavity and also has a higher
sensitivity than endometrial biopsy.
It can sometimes be used as a treatment for prolonged or excessive bleeding that is due to
hormonal changes and that is unresponsive to other treatments.
Treatment
The treatment of abnormal bleeding is based upon the underlying cause.
Birth control pill
Birth control pills are often used to treat uterine bleeding that is due to hormonal changes or
hormonal irregularities.
Birth control pills may be used in women who do not ovulate regularly to establish regular
bleeding cycles and prevent excessive growth of the endometrium.
In women who do ovulate, they may be used to treat excessive menstrual bleeding.
During perimenopause, birth control pills or other hormonal therapy may be used to regulate
the menstrual cycle and prevent excessive growth of the endometrium.
Estrogen

o To stop acute bleeding,

o intravenous After estrogen, give progestational agent to stabilise endometrium


and allow withdrawal bleeding

o Eg. Conjugated estrogen 25 – 40 mg IV

o Then add medroxyprogesterone acetate 10mg qd

• Progesterone
o Progesterone is a hormone made by the ovary that is effective in preventing
excessive bleeding in women who do not ovulate regularly.
o A synthetic form of progesterone, called progestin, may be recommended to
treat abnormal bleeding.
o Progestins are usually given as pills (eg, medroxyprogesterone acetate,
norethindrone ), and are taken one or more times daily for two to three weeks.
When the progestin is stopped, the woman should expect to have uterine
bleeding within 14 days.
o In some cases, the progestin is given on a regular basis ( eg, every few months )
to prevent excessive growth of the uterine lining and heavy menstrual bleeding.
Intrauterine device

o An intrauterine contraceptive device ( IUD ) that secretes progestin ( eg, Mirena ) may
be recommended for women who do not ovulate regularly.
o IUDs are inserted by a healthcare provider through the vagina and cervix into the
uterus. Most are made of molded plastic and include an attached plastic string that
projects through the cervix, enabling the woman to check that the device remains in
place
o Progestin-releasing IUDs decrease menstrual blood loss by 40 to 50 percent and
decrease pain associated with periods.

Nonsteroidal anti-inflammatory drugs


Nonsteroidal anti-inflammatory drugs ( NSAIDS, eg ibuprofen, naproxen sodium ) may also be
helpful in reducing blood loss and cramping in these women.
Ibuprofen 600-800mg PO 6-8 hours
Naproxen 250-500mg 12hours
Surgery
• Surgery may be necessary to remove abnormal uterine structures ( eg, fibroids, polyps ).
Women who have completed childbearing and have heavy menstrual bleeding can
consider a surgical procedure such as endometrial ablation.
• Women with fibroids can have surgical treatment of their fibroids, either by removing
the fibroid(s) ( eg, myomectomy ) or by reducing the blood supply of the fibroids ( eg,
uterine artery embolization ).
• D & C for immediate relief but no long term effect
• Hysterectomy for definite treatment

29. ruptured ovarian cyst. Types clinical features, diagnosis and treatment
Clinical presentation
The patient often presents with an acute onset of abdominal pain, typically during strenuous
physical activity, such as exercise or sexual intercourse. Given that follicular cyst rupture is
more common than corpus luteal cyst rupture, the onset tends to be midcycle. Other
associated symptoms include the following:
• Abdominal pain that is sudden and severe
• Vaginal bleeding
• Fever
• Feeling cold with clammy skin
• Nausea and/or vomiting
• Weakness
• Syncope
• Shoulder tenderness
• Circulatory collapse

Diagnostic methods

Physical Examination
• Vital signs are usually within normal range. Physical findings can range from mild
unilateral low abdominal tenderness to those of an acute abdomen with severe
tenderness, guarding, rebound, and peritoneal signs.
• Low-grade fever is sometimes observed, and an adnexal mass may be palpable,
although absence of such findings on examination has no diagnostic value as many cysts
decompress after rupture. Orthostatic changes are consistent with a sizable
hemorrhage.

Laboratory tests
• Serum or urine pregnancy testing should be performed. In the case of a positive result,
the patient should be evaluated for ectopic pregnancy.
• Complete blood count – monitor hematocrit to ensure there is no continued bleeding
• Blood group and cross- match are indicated in patient with significant peritoneal signs or
hemodynamic instability, because such patients may require surgical intervention or
blood transfusion.
Ultrasound diagnosis
• The diagnosis of a ruptured ovarian cyst usually starts with an ultrasound.If the cyst has
ruptured, the ultrasound will show fluid around the ovary and may even reveal an
empty, sac-like ulcer.
CT scan of the pelvis

Treatment
• Conservative management – recommend for stable patients
o Analgesics - oral or intravenous analgesics
o Follow up ultrasound tests
o For the patient with multiple episodes of ruptured physiologic cysts or following
a single severe episode, it is reasonable to consider suppression of ovulation
with oral hormonal contraception, as this may help reduce the risk of
recurrence of ovarian cysts.
• Surgical management
o Surgical care may entail laparoscopy or laparotomy, depending on clinical
presentation, amount of blood in the abdomen, patient stability, and operator
skill.
o Most bleeding can be stopped with suturing, cautery, cystectomy, or wedge
resection.
o Occasionally salpingo-oophorectomy
30. progressive fallopian gestation. Clinic, diagnosis, and treatment.Clinical symptoms and
signs
• No specific signs, patient think she is pregnant
• Presence of delayed period or spotting with features suggestive of pregnancy.
• Breast engorgement
• Change the taste, the smell and other sensational characteristic of pregnancy
• Loss of appetite
• Nausea , vomiting
• May be lower abdominal pain
• Uneasiness on one side of the flank which is continuous or at times colicky in nature.
Diagnosis
➢ Bimanual examination: ( most of time 6th to 8th week)
(i) Uterus is usually soft showing evidence of early pregnancy
(ii) Uterine size smaller than the expected term of pregnancy
(iii) A pulsatile small, well circumscribed tender mass may be felt through
one fornix separated from the uterus.
(iv) The palpation should be gentle, else rupture may precipitate and massive
intraperitoneal hemorrhage when shock and collapse may occur
dramatically.
➢ Ultrasound investigation
• Main method
• Transvaginal sonography (TVS) is more informative.
o The diagnostic features are:
(1) Absence of intrauterine pregnancy with a positive pregnancy test.
(2) Fluid (echogenic) in the pouch of Douglas.
(3) Adnexal mass clearly separated from the ovary.
(4) Rarely cardiac motion may be seen in an unruptured tubal ectopic
pregnancy.
o Color Doppler Sonography: (TV–CDS)—can identify the placental shape (ring-
of-fire pattern) and enhanced blood flow pattern outside the uterine cavity.
➢ Laparoscopy
• offers benefit in cases of confusion with other pelvic lesions.
• It should be employed only when the patient is hemodynamically stable.
• Advantages are: (i) Confirmation of diagnosis (ii) Removal of the ectopic mass
using operative procedures at the same time (iii) Direct injection of
chemotherapeutic agents into the ectopic mass—when medical management is
decided . However, laparoscopy runs the risk of false positive or false negative
diagnosis in 2–5% of cases.
➢ Laparotomy offers benefit when in doubt.
➢ Estimation of β hCG: Urine pregnancy test—ELISA is sensitive to 10-50 mIU/ml and are
positive in 95% of ectopic pregnancies. A single estimation of β hCG level either in the
serum or in urine confirms pregnancy but cannot determine its location. The
suspicious findings are:
(1) Lower concentration of β hCG compared to normal intrauterine pregnancy
(2) Doubling time in plasma fails to occur in 2 days.

➢ Combination of quantitative β hCG values and sonography: TVS provides visualization


of a well formed intrauterine gestational sac as early as 4-5 weeks from the last
menstrual period. The lowest level of serum β hCG at which a gestational sac is
consistently visible using TVS (discriminatory zone) is 1500 IU/L. The corresponding
value of serum β hCG for TAS is 6000 IU/L.
(1) When the β hCG value is greater than 1500 IU/L and there is an empty
uterine cavity, ectopic pregnancy is more likely.
(2) Failure to double the value of β hCG by 48 hours along with an empty uterus
is very much suggestive.
Management
Management may be expectant, medical or surgical
Expectant management:
Where only observation is done hoping spontaneous resolution.
Indications are:
(i) Initial serum hCG level < 1000 IU/L and the subsequent levels are falling
(ii) Gestation sac size < 4 cm
(iii) No fetal heart beat on TVS.
(iv) No evidence of bleeding or rupture.
Medical management:
Number of chemotherapeutic agents have been used either systemic or direct local
(under sonographic or laparoscopic guidance) as medical management of ectopic pregnancy.
The drugs commonly used for salpingocentesis are: methotrexate, potassium chloride,
prostaglandin (PGF2α), hyperosmolar glucose or actinomycin.
The patient must be
(i) hemodynamically stable
(ii) Serum hCG level should be < 3000 IU/L
(iii) tubal diameter should be less than 4 cm without any fetal cardiac activity
(iv) There should be no intra-abdominal hemorrhage. For systemic therapy, a
single dose of methotrexate (MTX) 50 mg/M2 is given intramuscularly.

Monitoring is done by measuring serum β hCG on D4 and D7. When the decline in hCG between
(i) D4 and D7 is > 15%, patient is followed up weekly with serum hCG until hCG < 10 mIU/ml. If
the decline is < 15%, a second dose of MTX 50 mg/M2 is given on D7. Variable dose
methotrexate (MTX) includes: MTX – 1 mg/kg IM on
D1,3,5,7 and Leukovorin 0.1 mg/kg IM on D2,4,6&8. Serum β hCG is monitored weekly until <
5.0 mIU/ml.

Surgery:
The procedure can be done either laparoscopically or by microsurgical laparotomy.
Options
➢ Salpingotomy ( incision into the tube to remove an ectopic)
➢ Salpingectomy (removal of the tube )

Salpingectomy is done when (i) whole of the affected tube is damaged (ii) contralateral tube is
normal or (iii) future fertility is not desired.
Following conservative surgery or medical treatment, estimation of β-hCG should be done
weekly till the value becomes < 5.0 mlU/ml. Additional monitoring by TVS is preferred.
Following laparoscopic salpingostomy, persistent ectopic pregnancy ranges between 4-20%.

31. Cancer of the cervix of uterus. Classification, clinic, methods of diagnostic.

HPV is the main cause of cervical neoplasia.


HPV DNA is found in
i) 95% of all squamous cell carcinomas,
ii) 90% of all adenocarcinomas and
iii) Adenosquamous carcinomas

staging

a) Stage 0: Carcinoma in situ, intraepithelial carcinoma


b) Stage I: The carcinoma is strictly confined to the cervix
i) Stage IA:
Invasive cancer identified only microscopically.
All gross lesions, even with superficial invasion, are stage 1B cancers.
Invasion is limited to measured stromal invasion with maximum depth of 5mm and width of
7mm.
ii) Stage IA1 – Measured invasion of stroma no greater than 3mm in depth and no wider than
7mm.
iii) Stage IA2 – Measured invasion of stroma greater than 3mm and no greater than 5mm
depth, and no wider than 7mm.
iv) Stage IB – Clinical lesions confined to the cervix of preclinical lesions higher than stage IA.
v) Stage IB1: Clinical lesions no larger than 4cm.
vi) Stage IB2: Clinical lesions larger than 4cm.

c) Stage II:
The carcinoma extends beyond the cervix but has not extended to the pelvic wall.
The carcinoma involves the vagina but not as far as the lower one-third.

ii) Stage IIA – No obvious parametrial involvement


iii) Stage IIB – Obvious parametrial involvement.

d) Stage III:
The carcinoma has extended to the pelvic wall.
On rectal examination, there’s no cancer free space between the tumour and the pelvic wall.
i) Stage IIIA – No extension to the pelvic wall
ii) Stage IIIB – Extension to the pelvic wall and /or hydronephrosis or non-functioning kidney.

e) Stage IV:
The carcinoma has extended beyond the true pelvis or has clinically involved the mucosa of the
bladder or rectum.

A bullous edema as such does not permit a case to be allotted to stage IV.
i) Stage IVA – Spread of the growth to adjacent organs
ii) Stage IVB – Spread to distant organs.

Diagnosis
Speculum examination - red granular area which looks like an ectopy (erosion) extending from
the external os or a nodular growth or an ulcer
The lesion bleeds on friction
Bimanual examination reveals the lesion is indurated, friable and bleeds to touch. Cervix is
freely mobile.
Rectal examination reveals the parametrium absolutely free

Ancillary aids for confirmation of staging:


Cystoscopy
X-ray chest
Intravenous pyelography
Proctoscopy.
32.Chlamydial infections and ureaplasmosis. Etiology, clinical features, diagnosis, treatment,
prophylaxis.
Chlamydial Infection epidemiology: The causative organism is Chlamydia trachomatis (of D-K
serotypes), an obligatory intracellular Gram-negative bacterium.
Chlamydia has longer incubation period (6-14 days) compared to gonorrhea (3–7 days). The
organisms affect the columnar and transitional epithelium of the genitourinary tract. The lesion
is limited superficially. As there is no deeper penetration, the pathological changes to produce
symptoms may not be apparent. The infection is mostly localized in the urethra, Bartholin’s gland,
and cervix. It can ascend upwards like gonococcal infection to produce acute PID.
Too often (20–40%), it is associated with gonococcal infection.
Clinical Features are non-specific and asymptomatic in most cases (75%). Dysuria, dyspareunia,
postcoital bleeding, and intermenstrual bleeding are the presenting symptoms. Findings include
mucopurulent cervical discharge, cervical edema, cervical ectopy, and cervical friability. bilateral
lower abdominal pain

Complications: Urethritis and bartholinitis are manifested by dysuria and purulent vaginal
discharge. Chlamydial cervicitis spreads upwards to produce endometritis and salpingitis.
Chlamydial salpingitis is asymptomatic in majority of the cases. It causes tubal scarring resulting
in infertility and ectopic pregnancy. It is the more common cause of perihepatitis (Fitz-Hugh-
Curtis syndrome) than gonococcus. The spread to the liver from the pelvic organs is via
lymphatics and the peritoneal cavity.

Diagnosis: Chlamydia: Diagnosis


Samples are taken from endocervix (endocervical swab).
common bllod test- wbs , ESR increased
Chlamydia culture
New tests include:
Direct immunofluorescence assays (DFA)
Enzyme immunoassay (EIA)
detection by polymerase chain reaction (PCR) is a very sensitive and specific test (95%).
sonography - has limited value . can detect dilated tubes fluid in douglas pouch .
cludocentesis- wbc more than 30 000 per ml is inflammation
laparoscopy or laparotomy- violin strings adhesrsions inthe pelvic and around the liver

Management of Chamydial infection:


preventive methods- barrier method condom, diaphragm
medical treatment
azithromycin — 1 g orally single dose or
doxycycline — 100 mg orally bid × 7 days or
ofloxacin — 200 mg orally bid × 7 days or
erythromycin — 500 mg orally bid × 7 days.

The sexual partner should also be treated with the same drug regimen. Treatment failure with
the above strict guidelines are indicative of either lack of patient compliance or reinfection.
ureaplasmosis

Spread of Ureaplasma Infections Major infections are  produced by M.hominis and


M.urealyticum Commonly spread by Sexaul contact
Ureaplasma can be opportunistic pathogen in pregnancy
Ureaplasma urealyticum T Strains of mycoplasma isolated from the urogenital tract of human
beings

clinical features -causes Urethritis  Pyelonephritis  Pelvic inflammatory disease 


Endometritis or chorioamnionitis  Infectious arthritis  Surgical wound infections  Neonatal
pneumonia  Neonatal meningitis
Men - Nonspecific urethritis, proctitis, balanoposthitis & Reiter’s syndrome
Women – acute salpingitis, PID, cervicitis, vaginitis Also associated with infertility, abortion,
postpartum fever, chorioamnionitis & low birth weight infants

diagnosis -Diagnosis of Urogenital Infections Material from urethra, cervical, or vaginal


cases are Diagnosed with Serological Tests
Cold Agglutination Test
PCR method

U. urealyticum Treatment - erythromycin U. urealyticum is resistant to tetracycline


Prevention- Abstinence or barrier protection No vaccine

33.Age periods of woman, their characteristics

preadolesence Not developed secondary sexual characteristics


According to tanner scale
Stage 1 -<10yr-no pubic hair
In breast – no glandular tissues, elevation of papilla only, areola
follows chest contour
Stage 2 (10-11 yr)pubic hair- small amount of , downy hair
around labia major with slight pigmentation
In breast- breast buds form with small area of surrounding
glandular tissue , areolar
Stage 3 (12-14yr)-
Pubic hair – hair become more coarse and curly, and begin to
extend laterally
Breast- breast begins to become more elevated , and extends
beyond the borders of the areola, which continue to widen but
remains in contour with surrounding breast
Menarch Stage 4- age 13-15yr
Pubic hair – adult like hair quality, extending across pubis but
sparing medial thigh
Breast – increased breast size and elevation , areola and papilla
form a secondary mound projecting from the contour of the
surrounding breast
Stage 5 16+
Pubic hair – adult like to medial surface of the thighs
Breast – reaches final adult size , areola returns to contour of
the surrounding breast with a projecting central papila
menopause
a) Vagina:

i) Atrophic changes of the epithelium


ii) The rugae progressively flatten

iii) As the epithelium thins, the capillary bed shines through as a


diffuse or patchy reddening.

iv) Rupture of surface capillaries produce irregular scattered


petechiae, and a brownish discharge may be noted.

v) Further atrophy renders its capp. Bed increasingly sparse, so


hyperemic appearance becomes smooth, shiny and pale.
b) Cervix:

i) Decrease in size
ii) Reduction of cervical mucus (excessive vaginal dryness which
may cause dyspareunia)

c) Uterus

i) Shrinkage of endo/myometrium (sometime beneficial to


those who has myoma and prevent necessity of surgery)

d) Oviducts and ovaries

i) Decrease in size and becomes impalpable

ii) Supporting structures


iii) Loss of tone

iv) May lead to progressive pelvic relaxation

34. Benign tumors of mammary gland.


Breast cysts
• These occur most commonly in the last decade of reproductive life as a result of a non-
integrated involution of stroma and epithelium.
• They are often multiple, may be bilateral
• can mimic malignancy but almost always benign
• filled with green-yellow fluid, even black
• occasionally painful
Diagnosis
• USS
• Mammography
• FNAC/ aspiration
• Core biopsy/ local excision – in residual mass/ recurrent cyst/ bloody aspirate
Treatment
• A solitary cyst or small collection of cysts can be aspirated
• If they resolve completely, and if the fluid is not blood-stained, no further treatment is
required.
• However, 30% will recur and require reaspiration.
• Cytological examination of cyst fluid is no longer practiced routinely.
• If there is a residual lump or if the fluid is blood-stained, a core biopsy or local excision for
histological diagnosis is advisable, which is also the case if the cyst reforms repeatedly.
• Hormone manipulation
• This will exclude cystadenocarcinoma, which is more common in elderly women.
Galactocele
Galactocele, which is rare, usually presents as a solitary, subareolar cyst and always dates from
lactation. It contains milk and in longstanding cases its walls tend to calcify.
Fibroadenoma
• usually arise in the fully developed breast between the ages of 15 and 25 years, although
occasionally they occur in much older women
• arise from hyperplasia of a single lobule and usually grow up to 2–3 cm in size.
• Estrogen dependent – therefore increase in size with menstruation, involutes after
menopause
• They are surrounded by a well-marked capsule and can thus be enucleated through a
cosmetically appropriate incision.
Diagnosis
• USS
• FNAC
Treatment
• Reassurance and allow to regress spontaneously
• Surgical excision – in doubtful diagnosis, cosmesis, patient’s preference
• Cryoablation
• Heating with high frequency US
• Removal with large core biopsy vacuum system
Phyllodes tumour
• These benign tumours, previously sometimes known as sero-cystic disease of Brodie or
cystosarcoma phyllodes, usually occur in women over the age of 40 years but can appear in
younger women.
• They present as a large, sometimes massive, tumour with an unevenly bosselated surface.
• Occasionally, ulceration of overlying skin occurs because of pressure necrosis.
• Despite their size they remain mobile on the chest wall.
• Histologically, there is a wide variation in their appearance, with some of low malignant
potential resembling a fibroadenoma and others having a higher mitotic index, which are
histologically worrying.
• The latter may recur locally but, despite the name of cystosarcoma phyllodes, they are rarely
cystic and only very rarely develop features of a sarcomatous tumour.
• These may metastasize via the bloodstream.
Diagnosis
• USS
• FNAC
• Mammography
Treatment
• Treatment for the benign type is enucleation in young women or wide local excision.
• Massive tumours, recurrent tumours and those of the malignant type will require
mastectomy

35. Amenorrhea. Etiology. Classification.

CRYPTOMENORRHEA: In cryptomenorrhea, there is periodic shedding of the endometrium and


bleeding but the menstrual blood fails to come out from the genital tract due to
obstruction in the passage
Causes: „ Congenital „ Acquired
Congenital
. Imperforate hymen
. Transverse vaginal septum
. Atresia of upper-third of vagina and cervix.
Acquired
. Stenosis of the cervix following amputation, deep auterization and conization.
. Secondary vaginal atresia following neglected and difficult vaginal delivery.

Primary amenorrhea: A young girl who has not yet menstruated by her 16 years of age is
having primary amenorrhea rather than delayed menarche. The normal upper age limit for
menarche is 15 years.
Causes: The causes of primary amenorrhea are grouped as follows:
A. Hypogonadotropic hypogonadism
(i) Delayed puberty — delayed GnRH pulse reactivation.
(ii) Hypothalamic and pituitary dysfunction — Gonadotropin deficiency due to stress, weight loss,
excessive exercise, anorexia nervosa, chronic disease (tuberculosis).
(iii) Kallmann’s syndrome — inadequate GnRH pulse secretion — reduced FSH and LH.
(iv) Central nervous system tumors — craniopharyngioma→ reduced GnRH secretion →
reduced FSH and LH.
B. Hypergonadotropic hypogonadism
(i) Primary ovarian failure.
(ii) Resistant ovarian syndrome.
(iii) Galactosemia: Due to premature ovarian failure.
(iv) Enzyme deficiency (17 α hydroxylase deficiency) — characterized by ↓ cortisol and ↑
ACTH, ↑ mineralocorticoids production. There is hypertension with hypernatremia and
hypokalemia. The individual may be 46 XX or 46 XY with primary amenorrhea and no secondary
sexual characters.
(v) Others — Gonadotropin receptor mutations – rarely FSH and/or LH levels are high as the
respective receptor may be absent or mutated.
C. Abnormal chromosomal pattern
• Turner’s syndrome (45 X).
• Various mosaic states 45 X/46 XX.
• Pure gonadal dysgenesis (46 XX or 46 XY) —Phenotypically female with streak gonads.
Stature is average with some secondary sexual characters.
• Androgen insensitivity syndrome (Testicular feminization syndrome), 46 XY.

• Partial deletions of the X chromosome (46 XX). When part of one X chromosome is missing
— deletion of long arm of X chromosome (Xq–) leads to streak gonads and amenorrhea
but no somatic abnormalities. Deletion of short arm of X chromosome (Xp–) usually leads
to somatic features similar to Turner’s syndrome.
D. Developmental defect of genital tract
yy Imperforate hymen.
yy Transverse vaginal septum.
yy Atresia upper-third of vagina and cervix.
yy Complete absence of vagina.
yy Absence of uterus in MRKH syndrome
E. Dysfunction of thyroid and adrenal cortex
yy Adrenogenital syndrome.
yy Cretinism.
F. Metabolic disorders
yy Juvenile diabetes.
G. Systemic illness
yy Malnutrition, anemia
yy Weight loss
yy Tuberculosis.
H. Unresponsive endometrium
yy Congenital
−− Uterine synechiae (tubercular).
36- the organization of gynecological assistance of russia

37. Ovarian cancer. Classification, clinic, diagnostic.


1. Primary
a. Epithelial
i. Serous
ii. Mucinous
iii. Endometroid
iv. Brenner
b. Nonepithelial
i. Germ cell
1) Mature cystic teratoma
2) Dysgerminoma
3) Choriocarcinoma
4) Embryonal carcinoma
5) Yolk sac tumour
ii. Sex cord stromal
1) Granulosa cell tumour
2) Theca cell tumour
3) Fibroma
4) Thecoma
5) Sertoli cell tumour
6) Ledig cell tumour
2. Secondary
a. Typical
b. Atypical (krukenberg)

Clinical features

LOA with a sense of bloating after meals


In pre-existing tumor
• Appearance of dull aching pain and tenderness over one area
• Rapid enlargement of the tumor
Gradually, more pronounced symptoms appear. These are
• Abdominal swelling which may be rapid
• Dull abdominal pain
• Sudden LOW
• Respiratory distress — may be mechanical due to ascites or due to pleural effusion
• Menstrual abnormality is conspicuously absent except in functioning ovarian tumors

General Examination reveals


Cachexia and pallor of varying degree
Jaundice may be evident in late cases.
Left supraclavicular lymph gland (Virchow’s) may be enlarged
Edema leg or vulva is characteristic of malignant and not of benign neoplasm
Per abdomen
Liver may be enlarged, firm and nodular
A mass is felt in the hypogastrium; too often it may be bilateral. It has got the following
features:
− Feel — solid or heterogenous
− Mobility — mobile or restricted
− Tenderness — usually present
− Surfaces — irregular
− Margins — well-defined but the lower pole is usually not reached
− Percussion — usually dull over the tumor may be resonant due to overlying intestinal
adhesions
Per vaginum
The uterus may be separated from the mass felt per abdomen
Nodules may be felt through the posterior fornix

Diagnosis
• FBC- Hb/ Plt
• Renal function test
• Chest x-ray-chest to exclude pleural effusion and chest metastasis
• Tumor markers-
o Epithelial tumors- CA125 (normal value- 35 U/mL)
o Granulosa cell tumors- inhibin/ estrogen
o Teratoma- alpha fetoprotein
• USS-
o Bilateral involvement
o Solid component
o Multilocular cyst
o Increased vascularity
• Barium enema- to detect any lower bowel malignancy
• Cytologic examination of thoracocentesis fluid
• Computed Tomography (CT) is helpful for retroperitoneal lymph node assessment and
detection of metastasis (liver, omentum). It helps in staging of ovarian carcinoma
• Magnetic Resonance Imaging (MRI) is helpful to determine the nature of ovarian neoplasm
and also for the retroperitoneal lymph nodes and detection of metastasis. It can also detect
relapse of the tumor following initial treatment
• Positron Emission Tomography (PET) can differentiate normal tissues from cancerous
tissues. It is more sensitive than CT or MRI
• Intravenous pyelography
• Examination under anesthesia
• Diagnostic uterine curettage
38. Dysmenorrhoea. Clinical features. Treatment.

(dutta)
Clinical features: The pain begins a few hours before or just with the onset of menstruation.
The severity of pain usually lasts for few hours, may extend to 24 hours but seldom persists
beyond 48 hours. The pain
is spasmodic and confined to lower abdomen; may radiate to the back and medial aspect of
thighs. Systemic discomforts like nausea, vomiting, fatigue, diarrhea, headache and tachycardia
may be associated. It may be accompanied by vasomotor changes causing pallor, cold sweats and
occasional fainting. Rarely, syncope and collapse in severe cases may be associated. Abdominal
or pelvic examination does not reveal any abnormal findings. For detection of any pelvic
abnormalities, ultrasound is very useful and it is not invasive.

(lecture)
HISTORY
• A complete history should include the following : • Age at menarche • Menstrual frequency,
length of period, estimated menstrual flow, and presence or absence of intermenstrual bleeding
• Associated symptoms • Onset, duration, type, and severity of pain, as well as its relation to the
menstrual cycle • External factors affecting the pain • Impact of dysmenorrhea on physical and
social activity • Progression of symptom severity • Sexual and obstetric history

CHARACTER OF PAIN IN PRIMARY DYSMENORRHOEA • Onset shortly after menarche (typically


within • 6 months) • Usual duration of 48-72 hours (often • starting several hours before or just
after • the menstrual flow) • Cramping or laborlike pain • Background of constant lower
abdominal pain • , radiating to the back or the anterior or • medial thigh • Often unremarkable
pelvic examination findings (including rectal)

CHARACTER OF PAIN IN SECONDARY DYSMENORRHOEA • A different pattern of pain is


observed with secondary dysmenorrhea that is not limited to the onset of menses; this is
usually associated with abdominal bloating, pelvic heaviness, and back pain. Typically, the pain
progressively increases during the luteal phase until it peaks around the onset of menstruation.
• The following may indicate secondary dysmenorrhea[1, 2] : • Dysmenorrhea beginning in the
20s or 30s, after relatively painless menstrual cycles in the past • Heavy menstrual flow or
irregular bleeding • Dysmenorrhea occurring during the first or second cycles after menarche,
which may indicate congenital outflow obstruction
• Pelvic abnormality with physical examination (consider endometriosis, pelvic inflammatory
disease [PID], pelvic adhesions, and adenomyosis) • Little or no response to nonsteroidal anti-
inflammatory drugs (NSAIDs) or OCs • Infertility • Dyspareunia • Vaginal discharge

SEVERITY OF PAIN • Do you need to take pain killer for this pain? • Have you needed to take any
time off work/school due to pain?
•EXAMINATION
• ABDMINAL EXAMINATION • For any mass
PELVIC EXAMINATION • Inspection of the external genitalia for rashes, swelling, or discoloration
• Inspection of the vaginal vault for discharge, blood, or foreign bodies • Inspection of the cervix
for the above, plus any masses or signs of infection
• BIMANUAL EXAMINATION • To assess cervical motion tenderness, uterine or adnexal
tenderness, or any masses in the pelvis • Fixed uterus • Endometriotic nodules

Treatment: General measures include improvement of general health and simple psychotherapy
in terms of explanation and assurance. Usual activities including sports are to be continued.
During menses, bowel should be kept empty; mild analgesics and antispasmodics may be
prescribed. Habit forming drugs such as pethidine or morphine must not be prescribed. With
these simple measures, the pain is relieved in majority.
Severe cases:
™ Drugs
™ Surgery Drugs: The drugs used are —  Prostaglandin synthetase inhibitors.  Oral
contraceptives (combined estrogen and progestogen). Prostaglandin synthetase inhibitors (PSI)
These drugs not only reduce the prostaglandin synthesis (by inhibition of cyclo-oxygenase
enzyme) but also have a direct analgesic effect. Intrauterine pressure is reduced significantly. Any
of the preparations listed in the table can be used orally for 2–3 days starting with the onset of
period. The drug should be continued for 3–6 cycles.

COMMONLY USED NSAIDs


™ Fenamate group — mefanamic acid 250–500 mg 8 hourly or flufenamic acid 100–200 mg 8
hourly. ™ Propionic acid derivatives — ibuprofen 400 mg 8 hourly or naproxen 250 mg 6 hourly.
™ Indomethacin 25 mg 8 hourly
Newer drugs NSAIDs inhibit two different isoforms of the enzyme cyclo-oxygenase:
COX–1 and COX–2. Selective inhibitors of the enzyme COX-2 may have similar analgesic efficacy
but fewer side effects. Transdermal use of smooth muscle relaxant glyceryl trinitrate is also used
currently.
The suitable cases are—comparatively young age and having contraindications to ‘pill’. The
contraindications of its use include allergy to aspirin, gastric ulceration and history of asthma.

Oral contraceptive pills: The suitable candidates are patients (i) wanting contraceptive
precaution, (ii) with heavy periods and (iii) unresponsive or contraindications to anti-
prostaglandin drugs. The pill should be used for 3–6 cycles.

Dydrogesterone: It does not inhibit ovulation but probably interferes with ovarian
steroidogenesis. The drug should be taken from day 5 of a cycle for 20 days. It should be
continued for 3–6 cycles
If the above protocol fails, laparoscopy is indicated to find out any pelvic pathology to account
for pain, the important one being endometriosis.

Surgery: Transcutaneous electrical nerve stimulation (TENS) has been used to relieve
dysmenorrhea. Results are not better than that of analgesics. Surgical procedures: Laparoscopic
uterine nerve ablation (LUNA) for primary dysmenorrhea has not been found beneficial.
Laparoscopic presacral neurectomy is done to cut down the sensory pathways (via T11–T12) from
the uterus. It is not helpful for adnexal pain (T9 –T10) as it is carried out by thoracic autonomic
nerves along the ovarian vessels. As such its role in true dysmenorrhea is questionable. Dilatation
of cervical canal: It is done under anesthesia for slow dilatation of the cervix to relieve pain by
damaging the sensory nerve endings. It is not commonly done. Late sequela may be cervical
incompetence.

SECONDARY DYSMENORRHEA (Congestive)


Clinical features: The pain is dull, situated in the back and in front without any radiation. It usually
appears 3–5 days prior to the period and relieves with the start of bleeding. The onset and
duration of pain depends on the pathology producing the pain. There is no systemic discomfort
unlike primary dysmenorrhea. The patients may have got some discomfort even in between
periods. There are symptoms of associated pelvic pathology. Abdominal and vaginal
examinations usually reveal the offending lesion. At times, the lesion is revealed by laparoscopy,
hysteroscopy or laparotomy
Treatment: The treatment aims at the cause rather than the symptom. The type of treatment
depends on the severity, age and parity of the patient

39. The common symptoms for gynecological diseases.


Abnormal vaginal bleeding:
- STDs (sexually transmitted diseases).
- Bleeding even not to the menstrual
cycle.
- Infection of the cervix or uterine lining.
Common in all types of gynecological ca
- Bleeding between menstruation, after
- Fibroids or cervical polyps, lumen polyps. vulvar cancer).
sex, or after menopause.
Polycystic ovary syndrome (PCOS).
- The amount of blood is too much or
much less than the previous cycle.
Feeling of gas, nausea, stomach pain or
- Common manifestations in the
back pain, fatigue, increased body Ovarian Cancer.
menstrual period.
temperature.
- Endometrial cancer. - Ovarian Cancer.
Frequent back pain, lower abdominal
pain or pelvic pressure, constant - Endometrial optimism. - Cancer of the uterus.
urination.
- Fibroids.
- Endometrial cancer.
- Urinate a lot.
- Endometrial optimism.
- Constipation frequently. - Ovarian Cancer.
- Fibroids.
- Acute diarrhea. - Vaginal cancer.
- Prolapse.

- Urinary tract infections.


- The vulva itching, burning, or pain.
-Vaginitis.
- Vulvar pain accompanied by a rash,
- Vulvodynia. - Vulvar cancer.
sores, and warts.
- STDs (sexually transmitted diseases).
- Pain during sex.
- Vaginal infections.
- Increased vaginal discharge.
- Vaginitis caused by fungus.
- Cervical cancer.
- Vaginal discharge has an unusual
- Chlamydia or STDs.
odor.
- Endometrial cancer
Pelvic inflammatory disease (PID).
- Bloody mucus secretion.
- Trichomonas.
- Pain after sex - Vaginitis, cervicitis. - Cervical cancer
Signal Common gynecological díease Gynecological Cancer

- Irregular menstruation.
- Fibroids.
- Frequent low back pain.
Abnormal vaginal bleeding:
- STDs (sexually transmitted diseases).
- Bleeding even not to the menstrual
cycle.
- Infection of the cervix or uterine lining.
Common in all types of gynecological ca
- Bleeding between menstruation, after
- Fibroids or cervical polyps, lumen polyps. vulvar cancer).
sex, or after menopause.
Polycystic ovary syndrome (PCOS).
- The amount of blood is too much or
much less than the previous cycle.
40. Amenorrhea, etiology, principals of diagnostic and therapy.

CRYPTOMENORRHEA: In cryptomenorrhea, there is periodic shedding of the endometrium and


bleeding but the menstrual blood fails to come out from the genital tract due to
obstruction in the passage.
Causes: „ Congenital „ Acquired
Congenital
. Imperforate hymen
. Transverse vaginal septum
. Atresia of upper-third of vagina and cervix.
Acquired
. Stenosis of the cervix following amputation, deep auterization and conization.
. Secondary vaginal atresia following neglected and difficult vaginal delivery.

Primary amenorrhea: A young girl who has not yet menstruated by her 16 years of age is
having primary amenorrhea rather than delayed menarche. The normal upper age limit for
menarche is 15 years.
Causes: The causes of primary amenorrhea are grouped as follows:
A. Hypogonadotropic hypogonadism
(i) Delayed puberty — delayed GnRH pulse reactivation.
(ii) Hypothalamic and pituitary dysfunction — Gonadotropin deficiency due to stress, weight loss,
excessive exercise, anorexia nervosa, chronic disease (tuberculosis).
(iii) Kallmann’s syndrome — inadequate GnRH pulse secretion — reduced FSH and LH.
(iv) Central nervous system tumors — craniopharyngioma→ reduced GnRH secretion →
reduced FSH and LH.
B. Hypergonadotropic hypogonadism
(i) Primary ovarian failure.
(ii) Resistant ovarian syndrome.
(iii) Galactosemia: Due to premature ovarian failure.
(iv) Enzyme deficiency (17 α hydroxylase deficiency) — characterized by ↓ cortisol and ↑
ACTH, ↑ mineralocorticoids production. There is hypertension with hypernatremia and
hypokalemia. The individual may be 46 XX or 46 XY with primary amenorrhea and no secondary
sexual characters.
(v) Others — Gonadotropin receptor mutations – rarely FSH and/or LH levels are high as the
respective receptor may be absent or mutated.
C. Abnormal chromosomal pattern
• Turner’s syndrome (45 X).
• Various mosaic states 45 X/46 XX.
• Pure gonadal dysgenesis (46 XX or 46 XY) —Phenotypically female with streak gonads.
Stature is average with some secondary sexual characters.
• Androgen insensitivity syndrome (Testicular feminization syndrome), 46 XY.
• Partial deletions of the X chromosome (46 XX). When part of one X chromosome is missing
— deletion of long arm of X chromosome (Xq–) leads to streak gonads and amenorrhea
but no somatic abnormalities. Deletion of short arm of X chromosome (Xp–) usually leads
to somatic features similar to Turner’s syndrome.
D. Developmental defect of genital tract
yy Imperforate hymen.
yy Transverse vaginal septum.
yy Atresia upper-third of vagina and cervix.
yy Complete absence of vagina.
yy Absence of uterus in MRKH syndrome
E. Dysfunction of thyroid and adrenal cortex
yy Adrenogenital syndrome.
yy Cretinism.
F. Metabolic disorders
yy Juvenile diabetes.
G. Systemic illness
yy Malnutrition, anemia
yy Weight loss
yy Tuberculosis.
H. Unresponsive endometrium
yy Congenital
−− Uterine synechiae (tubercular).

Investigations:
History:Certain types of primary amenorrhea are of heredofamilial in nature. Delayed menarche
or androgen insensitivity syndrome often runs in family, the later one is often found in multiple
sibs of the same family and their maternal aunts.
Medical diseases: Genital tuberculosis or diabetes though rare, may be responsible for primary
amenorrhea. Such type of amenorrhea is usually associated with hypogonadism.
Other features: Abnormal loss or gain in weight within short span of time is suggestive of
some metabolic disorder
The scope of therapeutic success in the management of primary amenorrhea is very limited.
Development Anomalies

Complete agenesis of vagina - Vaginal reconstruction is the accepted form of treatment. The
principle of vaginal reconstruction is to create an avascular space between the bladder and
rectum. The patency is to be maintained by a mould and graft. The commonly used materials for
graft are skin or amniotic membranes. The result are quite satisfactory so far as the coital act is
concerned. The ideal time of operation is prior to or soon after marriage.

Chromosomal Abnormalities - In Turner or other types of gonadal dysgenesis, shortterm use of


combination of estrogen and progestogen is indicated at least for development of breasts. The
gonads of XY gonadal dysgenesis should be removed for its increased development of seminoma
or dysgerminoma. In androgen insensitivity syndrome, the ectopic gonads are to be removed
after the secondary sex characters are well-developed, because they may turn to malignancy.
Substitution therapy after gonadectomy is indicated to maintain the secondary sex characters.
Hormone replacement therapy with conjugated equine estrogen (premarin 0.625 mg daily) is
adequate.

Hypothalamopituitary ovarian axis defect - Patients with delayed puberty, following exclusion of
other causes, should be counselled and reassured. Otherwise puberty may be induced using oral
estrogen and progestin therapy when there is severe delay. Gross defects in the form of
adiposogenital dystrophy or pituitary dwarfism are not amenable to any form of therapy. In mild
disorders, it is possible to induce ovulation and menstruation either by treatment with
gonadotropins or with GnRH analogs. Individuals with isolated gonadotropin deficiency
(Kallmann’s syndrome) can be treated for induction of menstruation or ovulation. Pulsatile
administration of GnRH is used for induction of ovulation. Estrogen and progestin therapy is given
for menstruation. Hypothalamic-pituitary tumors (craniopharyngioma) may need surgical
excision or radiotherapy. Team approach involving a gynecologist, an endocrinologist, a
neurosurgeon and a radiotherapist is ideal.

Thyroid and adrenal dysfunction - Gross thyroid hypoplasia (cretinism) does not respond to
thyroid replacement therapy. However, mild hypothyroidism may have good result with
replacement therapy. Adrenogenital syndrome with enlarged clitoris should be treated by
surgical removal of clitoris (clitoroplasty) as early as possible to avoid psychological problems.
Corticosteroid therapy should be continued for a prolonged period. Corticosteroid replacement
therapy is given for 17 α hydroxylase deficiency state. Prolactinomas need to be treated with
dopamine agonists.

Metabolic and Nutritional - Diabetes and tuberculosis are to be treated by antidiabetic and
antitubercular drug respectively. Correction of anemia and improvement of nutrition status may
resume menstruation. Correction of malabsorption, weight loss stress and chronic diseases are
to be done when indicated.

Unresponsive Endometrium - Uterine synechiae of tubercular origin should be treated by


antitubercular drugs supplemented by adhesiolysis and intrauterine contraceptive device (IUCD)
insertion.
Hysteroscopic release of adhesions using scissors or electrocautery can be done. To prevent
recurrence of adhesion formation, high dose estrogen and progestin therapy is given monthly for
withdrawal bleed. There is no known treatment as yet for congenital unresponsive endometrium
(receptor defect)
Secondary Amenorrhea: It is the absence of menstruation for 6 months or more in a woman in
whom normal menstruation has been established.
Ethiology
Investigation:
In secondary amenorrhea, there is altered coordinated function of the hypothalamopituitary
ovarian axis by some pathology. As such, it is not easy in most cases to pinpoint the diagnosis
only by clinical examination.
It should be emphasized that pregnancy must be excluded prior hand irrespective of the status
of the women — married, unmarried, widow, divorced or separated.
Detailed History
Enquiry should be made about:
(i) Mode of onset — whether sudden or gradual preceded by hypomenorrhea or oligomenorrhea.
(ii) Sudden change in environment, emotional stress, psychogenic shock, or eating disorder
(anorexia nervosa).
(iii) Sudden change in weight—loss or gain.
(iv) Intake of psychotropic or antihypertensive drugs like reserpine or methyldopa. Intake of oral
‘pills’ or its recent withdrawal. History of radiotherapy and chemotherapy or surgery.
(v) Appearance of abnormal manifestations either coinciding or preceeding the amenorrhea,
such as:
(a) Acne, hirsutism (excessive growth of hair in normal and abnormal sites in female) or change
in voice.
(b) Inappropriate lactation (galactorrhea) — abnormal secretion of milk unrelated to pregnancy
and lactation.
(c) Headache or visual disturbances.
(d) Hot flushes and vaginal dryness.
(vi) Obstetric history — overzealous curettage leading to synechiae.
(a) Cesarean section may be extended to hysterectomy of which the patient may be unaware.
(b) Severe postpartum hemorrhage, or shock or infection.
(c) Postpartum or postabortal uterine curettage.
(d) Prolonged lactation — The patient may be amenorrheic since childbirth or she may have
one or two periods, followed by amenorrhea. Even though the patient states that she is not
breastfeeding her baby, a patient enquiry may reveal that she is putting the baby to the breast
at night. This is sufficient to make the patient remain amenorrheic.
(vii) Medical history of tuberculosis (pulmonary or extrapulmonary), diabetes, chronic nephritis
or
overt hypothyroid state should be enquired.
( viii) Family history — Premature menopause often runs in the family (mother or sisters).
General examination
The following features are to be noted:
i. Nutritional status.
ii. Extreme emaciation or marked obesity.
iii. Presence of acne or hirsutism.
iv. Discharge of milk from the breasts.
Abdominal examination
i. Presence of striae associated with obesity may be related to Cushing disease.
ii. A mass in the lower abdomen
Pelvic examination
i. Enlargement of clitoris.
ii. Adnexal mass suggestive of tubercular tuboovarian mass or ovarian tumor.

Even though there is no inappropriate galactorrhea, serum prolactin, TSH estimations and X-ray
sella turcica are mandatory. If these are normal, the following protocols are followed:
Step – I
Progesterone challenge test is employed. If withdrawal bleeding occurs, it proves— (i) The intact
hypothalamopituitary ovarian axis and (ii) There is adequate endogenous estrogens (serum E2
level more than 40 pg/ml) to promote progesterone receptors in the endometrium, (iii)
Anatomically patent outflow tract and (iv) Endometrium is responsive. Estimation of serum
testosterone, prolactin TSH, oral GTT and fasting lipid profile should be done in a case of PCOS. If
withdrawal bleeding fails to occur, it signifies — (i)
lack of progesterone receptors in the endometrium or (ii) diseased endometrium. To
differentiate between the two, one is to proceed to step II.
Step – II
Estrogen–progesterone challenge test — Ethinyl estradiol 0.02 mg or conjugated equine estrogen
1.25 mg is to be taken daily for 25 days. Medroxyprogesterone acetate 10 mg daily is added from
day 15–25. Alternately, one course of oral contraceptive pill is given and to observe whether
withdrawal bleeding occurs or not. If there is no bleeding, it signifies local endometrial lesion
such as uterine synechiae. This is to be confirmed by HSG or hysteroscopy. If withdrawal bleeding
occurs, it indicates the presence of responsive endometrium but the endogenous estrogen
production is inadequate. As such, to determine whether the underlying defect lies in the ovary
or in the pituitary, one is to proceed to step III.
Step – III
Estimation of serum gonadotropins is to be done. If the level of serum FSH is more than 40
mIU/ml, the case is one of premature ovarian failure or resistant ovarian syndrome. Ovarian
biopsy is not recommended to confirm the diagnosis or to differentiate the two entities. If,
however, the level of FSH is either normal or low, it signifies pituitary dysfunction. Whether the
disturbed pituitary function is primary or secondary to hypothalamus, one should proceed to step
IV.
Step – IV
GnRH dynamic test — If with GnRH administration, there is rise of pituitary gonadotropins, it is
probably a case of hypothalamic dysfunction. In cases of primary pituitary disorder, there will be
no rise of gonadotropins. The result is however inconclusive. If possible, pituitary tumor have to
be excluded by X-ray sella turcica, CT or MRI, scan even though the prolactin level is normal.
Conservative Therapy:
 Premature ovarian failure is managed as for menopause i.e. HRT
 Anovulation may be treated by Progestgens or oral contraceptives. In patients desiring
pregnancy, ovulation induction agents such as Clomiphene citrate (150-250 mg daily for 5 days)
or gonadotropins (human menopausal gonadotropin) may be used
 Hyperprolactinemia responds well to dopamine agonist such as Bromocriptine
 Pituitary insufficiency can be managed by replacing target organ hormones as well as HRT as
for a menopausal patient
 Hypogonadotrophic hypogonadism is also managed with cyclic HRT. Patients are generally
responsive to pulsatile GnRH therapy when pregnancy is required

Surgical therapy:
 Outflow tract disorders:  For Vaginal agenesis – formation of a functional neovagina  For
transverse vaginal septum or imperforate hymen – excision
 Dysgenetic gonads:  Removal of these gonads because of risk of malignancy
 For Pituitary macroadenomas – require excision if It does not respond to conservative therapy

41. Female infertility. Causes, diagnosis, methods of conservative and operative treatment.
Causes of female infertility
o Ovulation factor
o Tubal factor
o Uterine factor
o Vaginal factor

Ovulation factor
1. Problem in the H-P-O axis

Hypogonadotrophic hypogonadism
o Stress
o Sudden weight loss/ gain
o Infections
o Radiation
o Tumor
o Anorexia nervosa
o Bulimia nervosa
o Kallmann XD

Hypergonadotrophic hypogonadism
o Pituitary tumor
o TB
o Sheehan XD
o Turner mosaic

Normogonadotrophic normogonadism
o PCOS- commonest cause for annovulation
o Premature menopause
2. Prolactinemia (prolactine)
3. Hypo/ hyperthyroidism (thyroxine)
4. DM (insulin)

Diagnosis
Mid luteal progesterone level (> 30 ng/dl suggestive of ovulation)
Basal body temperature method
Cervical mucus thickness
Endometrium biopsy (whether it’s secretory endometrium)
Treatment
Treat the underlying cause

Normogonadotrophic normogonadism → ovulation induction


o Ant-estrogen- Clomiphene citrate
o Aromatase inhibitor- Letrazole (only stimulate single follicle)

Complications-
• Risk of multiple pregnancies
• Ovarian hyperstimulation XD (OHSS)-life threatening, can cause thromboembolism, pulmonary/
cerebral edema
• Long term- ovarian cancer

Tubal factor
PID
STD
Endometriosis
Congenital tubal anomaly
Diagnosis
1. Saline infusion sonogram
2. Hysterosalpoingo gram
• Analyze the endometrial cavity
• Tubal patency
• Localization of blockage
• Uterine anomalies and fibroid/ polyps
• Out-patient procedure
• Pain and vaso-vagal attack
• Risk of PID
• Results might be false bcz muscle spasms can be shown as blockage
3. Lap and Dye
• Gold standard
• Diagnostic and therapeutic
• Done under anesthesia
• Cannot find the location of blockage

Treatment
o Tubal reconstruction (tuboplasty)

(Risk of tubal stenosis and ectopic pregnancy)


o IVF

Uterine factor
Fibroids/ polyp
Uterine anomalies
Diagnosis
USS
Hysteroscopy
Treatment
Manage the cause
For ex:
Fibroids
Medical Mx
Non-hormonal- Tranexemic acid/ Mefenamic acid
Hormonal- OCP/ progesterone (reduce bleeding)
o GnRH analogue (reduce bleeding and fibroid size) - therefore can be given in pre-
operative stage (3-6 months)
o Selective progesterone receptor modulator (SPRM)- ulipristal acetate
o Anti-progesterone- gastrinone
o Danazol

Surgical Mx
Minimally invasive- uterine artery embolization (risk of infection, risk of hysterectomy)
Definitive- myomectomy (laparotomy/ laparoscopy- high side effects / hysteroscopy) - recurrent
risk is high
Hysterectomy- (total abdominal-TLA/ total laparoscopic-TLH/ vaginal hysterectomy)

42.adolscence . environment influence on reproductive system.

Adolscence
The period of life beginning with the appearanceof secondary sex characters and terminating
with cessation of somatic growth is described asadolescence. The problems during the period
are:
„ Menstrual disorders.
„ Delayed puberty
„ Delayed manifestations of intersex.
„ Hirsutism.
„ leucorrhea.
„ Neoplasm
The period of life beginning with the appearance of secondary sex characters and terminating
with cessation of somatic growth is described as adolescence.
This period is from 11- 18 years.

The problems during the period are:


• Menstrual disorders.
• Delayed puberty (see p. 54).
• Delayed manifestations of intersex.
• Hirsutism.
• Leucorrhea.
• Neoplasm.

Environmental influences on the reproductive system are,

Endogenous environmental factors-


• emotional stress
• psychogenic shock
• eating disorder (anorexia nervosa)
• hormonal and metabolic factors
Exogenous environmental factors-
• Mechanical (vibration, noise, overheating, etc.)
• chemical (alcohol, nicotine, drugs, medicines)
• biological (pollution of the atmosphere, soil, water)

43. Polycystic ovarian disease

A-definition, Polycystic ovarian syndrome (PCOS) was originally described in 1935 by Stein and
Leventhal as a syndrome manifested by amenorrhea, hirsutism and obesity associated with
enlarged polycystic ovaries. This heterogenous disorder is characterized by excessive androgen
production by the ovaries mainly. PCOS is a multifactorial and polygenic condition.
HA-IR-AN syndrome
Hyper Androgenism
Insulin Resistance
Acanthosis Nigricans

B-changes in ovaries,
Typically, the ovaries are enlarged. Ovarian volume is increased > 10 cm3. Stroma is increased.
The capsule is thickened and pearly white in color. Presence of multiple (> 12) follicular cysts
measuring about 2–9 mm in diameter are crowded around the cortex. There is thickening of
tunica albuginea. The cysts are follicles at varying stages of maturation and atresia. There is
theca cell hypertrophy (stromal Hyperthecosis).

C-clinics,
The patient complains of increasing obesity (abdominal – 50%), menstrual abnormalities (70%)
in the form of oligomenorrhea, amenorrhea or DUB and infertility. Presence of hirsutism and
acne are the important features (70%). Virilism is rare.

D-diagnosis,

Sonography — Transvaginal sonography is specially useful in obese patient. Ovaries are


enlarged in volume (> 10 cm3). Increased number (> 12) of peripherally arranged cysts (2–9
mm) are seen.

LH level is elevated and/or the ratio LH: FSH is > 2:1.

◼ Raised level of estradiol and estrone — The estrone level is markedly elevated.

◼ SHBG level is reduced.

◼ Hyperandrogenism—mainly from the ovary but less from the adrenals. Andro-
stenedione is raised.

◼ Raised serum testosterone (> 150 ng/dl) and DHEA–S may be marginally elevated.

◼ Insulin Resistance (IR): Raised fasting insulin levels > 25 μIU/ml and fasting
glucose/insulin ratio < 4.5 suggests IR (50%). Levels of serum insulin response > 300
μIU/ml at 2 hours postglucose (75 gm) load, suggests severe IR.

Laparoscopy — Bilateral polycystic ovaries are characteristic of PCOS

D-treatment
Management of PCOS needs individualization of the patient. It depends on her presenting
symptoms, like menstrual disorder, infertility, obesity, hirsutism or combined symptoms.

Weight reduction in obese patients is the first line of treatment. Body mass index (BMI) < 25
improves menstrual disorders, infertility, impaired glucose intolerance (insulin resistance),
hyperandrogenemia (hirsutism, acne) and obesity. Weight reduction (2–5%) improves the
metabolic syndrome and reproductive function.

Irregular bleeding: progestin and oral contraception

Hirsutism: oral contraceptive and spironolactone

Infertility: clomiphene and metformin

Laparoscopic ovarian drilling (LOD) is done for cases found resistant to medical therapy

44.endometriosis,clinics and diagnosis

Clinical picture
• Depend on the location of the endometriosis
• Reproductive systeme-heavy menstrual bleeding,dysmennorrhea,deep
dyspareunea,infertility
• Urinary tract-cyclic haematurea,obstruction,dysurea
• GIT-haematochezia,dyschesia,obstruction .
• Nasal cavity-cyclic epistaxis
• Surgical scar/umbilicus-cyclic pain and bleeding

Diagnosis
Ultrasound scan-can see chocolate cyst(ground glass appearence ),enlarged uterus in
adenomyosis.
MRI-deep tissue deposits (rectovaginal septum)
Laparascopy-gold standard ,show red /blackish matchstick like appearence ,white fibrotic
depostions.it has both diagnostic and therapeutic value.

45. Genital endometriosis. Etiology, classification, the most common symptoms, the
methods of diagnostic.
Presence of functioning endometrium (glands and stroma) in sites other than uterine
mucosa is called endometriosis.
Etiology- There are risk factors which increase the likelihood of endometriosis.
• Women who are having retrograde menstruation
• Heredity. The role of hereditary predisposition to the development of
endometriosis and its transmission from mother to daughter is very high.
• Surgical interventions on the uterus: surgical termination of pregnancy ,
cauterization of erosions, cesarean section, etc.
• Immunosuppression
• Metabolic disorders, obesity, overweight.
• Use of intrauterine contraceptives.
• Age after 30-35 years.
• Increased estrogen levels.
• Smoking.
Classification- Genital endometriosis can be classified as:
A. Internal (endometriosis of the body of the uterus or adenomyosis), endometriosis of
the intramural fallopian tubes);
B. External (ovarian endometriosis, retrocervical endometriosis, vaginal endometriosis,
endometriosis of the fallopian tubes, endometriosis of the vaginal part of the cervix,
round ligaments of the uterus, sacro-uterine ligaments, peritoneum, vesico-uterine
space and retardial space, external genital organs, perineum).
The most common symptoms-
• Dysmenorrhea (70%)
There is progressively increasing secondary dysmenorrhea. The pain starts a few days prior to
menstruation; gets worsened during menstruation and takes time, even after cessation of
period, to get relief of pain, (co-menstrual dysmenorrhea). Pain usually begins after few years
pain-free menses. The site of pain is usually deep seated and on the back or rectum. Increased
secretion of PGF 2α, thromboxane β2 from endometriotic tissue is the cause of pain.
• Abnormal menstruation (20%)
Menorrhagia is the predominant abnormality. If the ovaries are also involved, polymenorrhea
or epimenorrhagia may be pronounced. There may be premenstrual spotting.
• Infertility (40–60%)
Endometriosis is found in 20–40 percent of infertile women, where as in about 40–50 percent
patients with endometriosis suffer from infertility.
• Dyspareunia (20–40%)
The dyspareunia is usually deep. It may be due to stretching of the structures of the pouch of
Douglas or direct contact tenderness. As such, it is mostly found in endometriosis of the
rectovaginal septum or pouch of Douglas and with fixed retroverted uterus.
• Chronic Pelvic Pain
The pain varies from pelvic discomfort, lower abdominal pain or backache. The cause may be
multifactorial. These include—(i) Inflammation in the peritoneal implants and release of PGF,
and also due to adhesions and ovarian cysts. (ii) Action of inflammatory cytokines released by
the macrophages. (iii) Invasion of nerves or involvement of bladder and bowel. The pain
aggravates during period.
Methods of diagnosis-
• Clinical diagnosis is by the classic symptoms of progressively increasing secondary
dysmenorrhea, dyspareunia and infertility. This is corroborated by the pelvic findings of
nodules in the pouch of Douglas, nodular feel of the uterosacral ligaments, fixed
retroverted uterus and unilateral or bilateral adnexal mass. However, physical
examination has poor sensitivity and specificity.
• Serum marker CA 125—A moderate elevation of serum CA 125 is noticed in patients
with severe endometriosis. But it is not specific for endometriosis. Also monocyte
Chemotactic Protein (MCP-1)level is increased in the peritoneal fluid.
• Ultrasonography is not much helpful to the diagnosis. TVS (trans-vaginal scan) can
detect ovarian endometriomas.
• CT and MRI
• Laparoscopy is the gold standard.Confirmation is done by double puncture laparoscopy
or by laparotomy.
• Biopsy confirmation of excised lesion is ideal but negative histology does not exclude it.

46. Ovarian cyst torsion. Clinical features. Diagnosis. Treatment. Steps of operation.

Ovarian torsion is a condition that occurs when an ovary twists around the ligaments that
hold it in place, mainly due to a cyst.
Clinical features-
Torsion of the ovarian cyst provokes different symptoms. The most serious
symptomatology is characteristic of complete torsion. It manifests itself as:

Sudden onset ( commonly during exercises or other agitating movement) of severe,


unilateral lower abdominal pain that worsens intermittently over many hours. Pain can
be radiated to the back, pelvis or thigh.
Nausea and Vomiting.
Tender abdomen
Fever
Tachycardia.

In addition, the manifestation of a number of other symptoms is characteristic of the


complication:

Urinary incontinence.
Problems with urinary frequency.
Pain and cramps during urination.
Pale skin.
Hypotension.

Diagnosis-
Physical Examination
The physical examination, like the history, is typically nonspecific and is highly variable. A
unilateral, tender adnexal mass may be present and Tenderness to palpation is common;
however, it is mild.
Ultrasound-Ovarian enlargement secondary to impaired venous and lymphatic drainage is the
most common sonographic finding in ovarian torsion. A coexistent mass is often seen. The ovary
usually contains several cysts along its periphery; these are follicles that have likely been
displaced peripherally because of ovarian edema and venous congestion. In addition, there may
be irregular echogenic areas within the ovary corresponding to stromal edema and/or
hemorrhage.
On color Doppler sonograms, little or no intraovarian venous flow is present; this finding is
followed by a lack of intraovarian arterial flow.
Computed Tomography
Multiple CT findings have been described in ovarian torsion
• Enlarged adnexal structure (>5cm)
• Thick, straight blood vessels draped around the lesion
• Complete absence of enhancement
• Hemorrhage or gas in the torsed lesion
• Misplacement of the torsed structure (to the midline or contralateral side)
• Deviation of the uterus to the involved side
• Infiltration of the periadnexal fat
• Tubal thickening
• Thickened vascular pedical with engorged vessels
• Ascites
Magnetic Resonance Imaging
MRI may demonstrate ovarian enlargement and intraperitoneal fluid. In a case report, MRI
demonstrated a twisted pedicle.

Treatments
Pharmacological;
Analgesics- Narcotics- morphine
Antiemetics- Metoclopramide
Main surgical treatment- Laparoscopic surgery

Steps of operation- ovarian cyst laparoscopy is performed under general anesthesia.


Before laparoscopy, the doctor performs premedication, which consists of tranquilizers that have
sedative and hypnotic effects.
On the operating table, the patient is in the Trendelenburg position.
After processing the operating field, a puncture is made in the abdominal wall, through which
carbon dioxide enters the cavity.
Then a laparoscope is inserted into another hole. After moving the instrument to the ovaries, 2
more incisions are made in the lateral parts of the abdomen, which are intended for the
introduction of additional instruments.
If there is a need for wide access, then all the instruments are removed and the surgeons begin
the laparotomy.
Depending on the type and location of the cyst, removal can be done in different ways. Upon
completion, a thorough inspection is carried out, after which the instruments are removed and
the carbon dioxide is sucked off. Skin sutures and an aseptic dressing are applied.
The patient is transferred to the ward if there is no breathing disorder or other complications.
47. Contraception. Classification of methods and means. The criteria for ideal
contraceptive method (chapter 29,page 475)

Contraception.
• Methods of preventing pregnancy
Classification of methods and means

Temporary
• Temporary methods are commonly used to postpone or to space births
1.Barrier Methods
These methods prevent sperm deposition in the vagina or prevent sperm penetration
through the cervical canal. The objective is achieved by mechanical devices or by
chemical means which produce sperm immobilization, or by combined means.
• The following are used
1.Mechanical
Male — condom
female — condom, diaphragm, cervical cap
2.chemical (Vaginal contraceptives)
creams — delfen (nonoxynol-9, 12.5 %)
Jelly — Koromex, Volpar paste
foam tablets—Aerosol foams, chlorimin t or contab, sponge (today)
3.combination
combined use of mechanical and chemical

2.Natural
1.Fertility Awareness Method
Fertility Awareness Method requires partner’s cooperation.
The woman should know the fertile time of her menstrual cycle.

2.Rhythm Method:
The method is based on identification of the fertile period of a cycle and to abstain
from sexual intercourse during that period.This requires partner’s cooperation. The
methods to
determine the approximate time of ovulation and the
fertile period include —
(a) recording of previous menstrual cycles (calendar rhythm)
(b) noting the basal body temperature chart (temperature rhythm)and
(c) noting excessive mucoid vaginal discharge(mucus rhythm)

3.Coitus Interruptus (withdrawal)


It necessitates withdrawal of penis shortly before ejaculation

4.Breastfeeding, Lactational amenorrhea (LAM):


Prolonged and sustained breastfeeding offers a natural protection of pregnancy. This
is more effective in women who are amenorrheic than those who are menstruating.

3.IUCDs
The device may be nonmedicated as Lippes loop or medicated (bioactive).

4.Steroidal contraceptions
• Estrogen to a minimum of 20 µg or even 15 µg in the tablet
combined oral contraceptives (PIllS)
• The combined oral steroidal contraceptives is the most effective reversible method of
contraception. In the combination pill, the commonly used progestins are either
levonorgestrel or norethisterone or desogestrel and the estrogens are principally
confined to either ethinyl-estradiol or menstranol (3 methylether of ethinyl-estradiol).
Currently ‘lipid friendly’,

• Third generation progestins, namely desogestrel, gestodene,norgestimate are available.

• 4th generation: Drospirenone which is an analogue of spironolactone is used as


progestin. It has antiandrogenic and antimineralocorticoid action. It causes retention of
K+.

Permanent
1.Vasecotmy
It is a permanent sterilization operation done in the male where a segment of vas
deferens of both the sides are resected and the cut ends are ligated.

2. Tubectomy
It is an operation where resection of a segment of both the fallopian tubes is done to
achieve permanent sterilization.

The criteria for ideal contraceptive method


• widely acceptable
• inexpensive
• simple to use safe
• highly effective and requiring minimal motivation,maintenance and supervision

48. Regulation of childbirth. Prevention of spontaneous pregnancy


If the couple is expecting to have a child in the future, until then temporary methods of
contraception can be used.
They can be barrier methods, Natural contraception, IUDs and steroidal contraception.
Barrier Methods-These methods prevent sperm deposition in the vagina or prevent sperm
penetration through the cervical canal. The objective is achieved by mechanical devices or by
chemical means which produce sperm immobilization, or by combined means.
• Mechanical
Male — Condom
Female — Condom,diaphragm, cervical cap
• Chemical
(Vaginal contraceptives)
Creams — Delfen (nonoxynol-9, 12.5 %)
Jelly — Koromex, Volpar paste
Foam tablets—Aerosol foams, Chlorimin T or
Contab, Sponge (Today)
• Combination
Combined use of mechanical and chemical

Condom (male): The method is suitable for couples who want to space their families and who
have contraindications to the use of oral contraceptive or IUD. These are also suitable to those
who have infrequent sexual intercourse. Protection against sexually transmitted disease is an
additional advantage.

Female condom (Femidom) : It is a pouch made of polyurethane which lines the vagina and also
the external genitalia.

Diaphragm (Table 29.4 and Fig.29.1B) It is an intravaginal device made of latex with flexible
metal or spring ring at the margin. The device is introduced up to 3 hours before intercourse
and is to be kept for at least 6 hours after the last coital act.

Vaginal Contraceptives -1.Spermicides: Spermicides are available as vaginal foams, gels, creams,
tablets and suppositories.
These agents mostly cause sperm immobilization. The cream or jelly is introduced high in the
vagina with the help of the applicator soon before coitus. Foam tablets (1–2) are to be
introduced high in the vagina at least 5 minutes prior to intercourse.
2) Vaginal contraceptive sponge (Today): It releases spermicide during coitus, absorbs ejaculate
and blocks the entrance to the cervical canal. The sponge should not be removed for 6 hours
after intercourse.

Fertility Awareness Method: Fertility Awareness Method requires partner’s cooperation. The
woman should know the fertile time of her menstrual cycle.

Rhythm Method: The method is based on identification of the fertile period of a cycle and to
abstain from sexual intercourse during that period. This requires partner’s cooperation. The
methods to determine the approximate time of ovulation and the fertile period include — (a)
recording of previous menstrual cycles (calendar rhythm) (b) noting the basal body
temperature chart (temperature rhythm) and (c) noting excessive mucoid vaginal discharge
(mucus rhythm).
Coitus Interruptus (withdrawal) : It is the oldest and probably the most widely accepted
contraceptive method used by man. It necessitates withdrawal of penis shortly before
ejaculation.

IUD-They act predominantly in the uterine cavity and do not inhibit ovulation. Probable factors
are:
Biochemical and histological changes in the endometrium — There is a nonspecific
inflammatory reaction along with biochemical changes in the endometrium which have got
gametotoxic and spermicidal property. Lysosomal disintegration from the macrophages
attached to the device liberates prostaglandins, which are toxic to spermatozoa. Macrophages
cause phagocytosis of spermatozoa.
There may be increased tubal motility which prevent fertilization of the ovum. Endometrial
inflammatory response decreases sperm transport and impedes the ability of sperm to fertilize
the ovum.

The combined oral steroidal contraceptives - is the most effective reversible method of
contraception. In the combination pill, the commonly used progestins are either levonorgestrel
or norethisterone or desogestrel and the estrogens are principally confined to either ethinyl-
estradiol or menstranol.
Mode of action: The probable mechanism of contraception are: Inhibition of ovulation.
Producing static endometrial hypoplasia.
Alteration of the character of the cervical mucus (thick, viscid and scanty) so as to prevent
sperm penetration.
Probably interferes with tubal motility and alters tubal transport.

IMPLANT - Implanon is a progestin only delivery system containing 3 Ketodesogestrel


(etonorgestrel). It is a long-term (up to 3 years) reversible contraception. Mechanism of action:
It inhibits ovulation in 90 percent of the cycles for the first year. It has got its supplementary
effect on endometrium (atrophy) and cervical mucus (thick) as well.
If the couple is not expecting to have a child in the future and permanat sterilization can be
done. Vasectomy for males and tubal occlusion for females can be done.

49.Acute abdomen” in gynecology. Etiology, clinical symptoms, principals of therapy.

Acute pain is intense and characterized by the sudden onset, sharp rise, and short course.
Complication of pregnancy
1. Ectopic pregnancy
2. Abortion, threatened or incomplete
Acute infection
. Endometritis
. PID or salpingo-oophoritis
. Tubo-ovarian abscess
Adnexal disorders
. Hemorrhagic functional ovarian cyst
. Torsion of adnexa
. Rupture of functional, neoplastic, or inflammatory
ovarian cyst

Gastrointestinal
1. Gastroenteritis
2. Appendicitis
3. Bowel obstruction
4. Diverticulitis
5. Inflammatory bowel disease
6. Irritable bowel syndrome
Genitourinary
1. Cystitis
2. Pyelonephritis
3. Ureteral lithiasis
3. Aortic aneurysm
4. Abdominal angina
Clinical features-

Rapid onset of pain: perforation of hollow viscus or ischemia


- Colic or severe cramping pain: muscular contraction or obstruction of hollow viscus Pain
perceived over the entire
-abdomen: generalized reaction to irritating fluid in peritoneal cavity (blood, purulent fluid,
content of ovarian cyst)
-Autonomic reflex responses
1. nausea
2. emesis
3. diaphoresis
4. apprehension
- Sign of inflammation or infection
1. fever
2. leukocytosis

Diagnosis- Careful history- Menses history obstetric history, gynecological history


Chief Complaint onset of the pain , localization Frequency, periodicity , Aggravating / relieving
factors
Appetite ,Vomiting ,Medications used Allergies
physical examination - heart rate , respiratory rate , pule temperature ,skin color
abdominal examination- palpation , abdominal distension , peritoneal signs Bowel
sounds,Tenderness, guarding and rigidity
Pelvic examination - uterus size, tenderness, external genital examination , speculum
examination

Special Investigations
CBC and Urine
Blood Chemistry and HCG
Chest X-ray
CT, US, MRI of the abdomen
Endoscopy
Culdocentesis
Laparoscopy

Treatment- treatment for the underlying course mostly urgent laparotomy or laparoscopic
surgery follwing antibiotic therapy
Anti-shock treatment: Anti-shock measures are to be taken energetically with simultaneous
preparation
for urgent laparotomy.
• Ringer’s solution (crystalloid) is started, if necessary with venesection.
• Arrangement is made for blood transfusion.

50. Uterine myoma. Indications for surgical treatment.


Indications for myomectomy- planned
Persistent uterine bleeding despite medical therapy.
excessive pain or pressure symptoms.
size >2 weeks, woman desirous to have a baby.
Unexplained infertility with distortion of the uterine cavity.
recurrent pregnancy wastage due to fibroid.
rapidly growing myoma during follow-up.
™subserous pedunculated fibroid

inDIcatIOns OF emeRgency suRgeRy In a FIBROID


torsion of a subserous pedunculated fibroid.
Massive intraperitoneal hemorrhage following rupture of veins over subserous fibroid.
Uncontrolled infected fibroid.
Uncontrolled bleeding fibroid

51. The modern methods of contraception, their efficiency mark (Perlya’s index)(
https://www.who.int/news-room/fact-sheets/detail/family-planning-contraception)

Mechanisms of action and effectiveness of contraceptive methods [6]

Effectiveness: Effectiveness:
pregnancies per 100 pregnancies per
Method How it works women per year 100 women per
with consistent and year as commonly
correct use used
Combined oral
Prevents the release of eggs 0.3
contraceptives (COCs) 7
from the ovaries (ovulation)
or “the pill”
Thickens cervical mucous to
Progestogen-only pills
block sperm and egg from
(POPs) or "the 0.3 7
meeting and prevents
minipill"
ovulation
Thickens cervical mucous to
blocks sperm and egg from
Implants 0.1 0.1
meeting and prevents
ovulation
Thickens cervical mucous to
Progestogen only block sperm and egg from 0.2
4
injectables meeting and prevents
ovulation
Monthly injectables or
Prevents the release of eggs
combined injectable 0.05 3
from the ovaries (ovulation)
contraceptives (CIC)
Combined 7 (for patch)
contraceptive patch 0.3 (for patch)
Prevents the release of eggs
and combined 7 (for
from the ovaries (ovulation)
contraceptive vaginal 0.3 (for vaginal ring) contraceptive
ring (CVR) vaginal ring)
Effectiveness: Effectiveness:
pregnancies per 100 pregnancies per
Method How it works women per year 100 women per
with consistent and year as commonly
correct use used
Intrauterine device Copper component damages
(IUD): copper sperm and prevents it from 0.6 0.8
containing meeting the egg
Thickens cervical mucous to
Intrauterine device
block sperm and egg from 0.5 0.7
(IUD) levonorgestrel
meeting
Forms a barrier to prevent
Male condoms 2 13
sperm and egg from meeting
Forms a barrier to prevent 5
Female condoms 21
sperm and egg from meeting
Male sterilization Keeps sperm out of
0.1 0.15
(Vasectomy) ejaculated semen
Female sterilization Eggs are blocked from
0.5 0.5
(tubal ligation) meeting sperm
Lactational
Prevents the release of eggs
amenorrhea method 0.9 (in six months) 2 (in six months)
from the ovaries (ovulation)
(LAM)
Prevents pregnancy by
Standard Days
avoiding unprotected vaginal 5 12
Method or SDM
sex during most fertile days.
Reliable
Basal Body Prevents pregnancy by
effectiveness rates
Temperature (BBT) avoiding unprotected vaginal
are not available
Method sex during fertile days

Prevents pregnancy by
4
TwoDay Method avoiding unprotected vaginal 14
sex during most fertile days,
Prevents pregnancy by
Sympto-thermal
avoiding unprotected vaginal <1 2
Method
sex during most fertile
Emergency Prevents or delays the < 1 for ulipristal
contraception pills release of eggs from the acetate ECPs
(ulipristal acetate 30 ovaries. Pills taken to 1 for progestin-only
mg or levonorgestrel prevent pregnancy up to 5 ECPs
1.5 mg) days after unprotected sex 2 for combined
Effectiveness: Effectiveness:
pregnancies per 100 pregnancies per
Method How it works women per year 100 women per
with consistent and year as commonly
correct use used
estrogen and
progestin ECPs
The couple prevents
pregnancy by avoiding
unprotected vaginal sex Reliable
Calendar method or
during the 1st and last effectiveness rates 15
rhythm method
estimated fertile days, by are not available
abstaining or using a
condom.
Tries to keep sperm out of
Withdrawal (coitus 4
the woman's body, 20
interruptus)
preventing fertilization

52 . trophoblastic disease – refer q 75


53. Germ cell tumor of ovary. Classification. Treatment.

Treatment: young patients ( when preservation of fertility is desire) – laparotomy for staging
&unilateral salphingo- oopherecctomy
If there is a suspicion of involvement of other ovary- bisection of contralateral ovary &
excisional biopsy

Treatment of choice-Systemic chemotherapy ( fertility Is preserved even in presence of


metastatic disease)
Radiotherapy- loss of fertility is a problem with this ; not used in young patients

54) The meaning of cytological and histological tests in diagnostic of gynaecological diseases.

cytological and histological tests main indications are screening procedures,


cytohormonal studies, exfoliative cell cytology, Sex chromatin studies
and other tests

cyto and histo examination helps to reduce the incidence and mortality from cervical cancer
and other female reproductive system related malignancies also helps to diagnose infections
other cell abnormalities and diseases

Cytological examination example - pap smear examination


In cytological examinations, cell samples can be taken from different places of female
reproductive system
such as cervix vaginal wall fornix endometrium ... the place depends on the signs and
symptoms , other diagnostic methods and doctors experience. to take cell samples special
instruments are used
ex;for screening procedure Ayre’s(cervix) cytobrush are used for cytohormonal studies
wooden spatula used
then collected samples carefully taken to the lab and further examination take place.
for study the sample Fixation and Staining need.The principle of the staining is to achieve clear
nuclear definition and to define cytoplasmic coloration. staining done either with
Papanicolaou’s or Sorr’s method and
examined by a trained cytologist.

In cytological and histological examinations-


Morphological Abnormalities of the Nucleus
nucleus—in number, size and shape
Hyperchromasia
Condensation of chromatin material
Multinucleation
Abnormal cells mild, moderate, sever dyskaryotic cells
Carcinoma in situ
Invasive carcinomas
tissue hyperplasia
and other tissue and cellular pathologies can be diagnosed by these methods and are highly
accurate and fast
In cytological examinations these classifications are using for put final diagnose

Papanicolaou’s grading system

Group – I Normal
Group – II Presence of borderline atypical cells—probably due to infection. No evidence
of malignancy
Group – III Cells suspicious of malignancy
Group – IV Presence of few malignant cells
Group – V Presence of large number of malignant cells

The Bethesda classification describe abnormalities of squamous glandular and other cells

For Cytohormonal status -Maturation index is calculated by taking count of parabasal cell to
intermediate cell to superficial cell ratio

Liquid-based Cytology: Cervical smear


is taken using a plastic spatula. The spatula is rinsed in
a liquid media. Cells are separated by centrifugation.
Thin layer smears are made.

55. Alternative methods of treatment of uterine myoma.


Drugs used to mInImIze
Blood loss
Antiprogesterones (Mifepristone)
Danazol
gnrh analogs: ™ Agonists ™ Antagonists
Lng–iUs
Prostaglandin synthetase inhibitors

Surgery
Myomectomy

Endoscopic Surgery
Hysteroscopy- fibroid of 3–4 cm in diameter or a polyp is resected with a hysteroscope.
Laparoscopy: Subserous and intramural fibroids could be removed laparoscopically
Embolotherapy: Uterine artery embolization (UAE) causes avascular necrosis followed by
shrinkage of fibroid.
Hysterectomy: Hysterectomy in fact, is the operation of choice in symptomatic fibroid when
there is no valid reason for myomectomy.
56) Ectopic pregnancy. Etiology, pathogenesis, Classification. Clinical features of tubal
rupture. Treatment.
Definition - the fertilized ovum is implanted and develops outside the normal endometrial
cavity.
Etiology
A) Salpingitis and pelvic inflammatory disease (PID)
B) Iatrogenic causes
1. Contraception failure
(a) IUD—It prevents intrauterine pregnancy effectively, tubal implantation to a lesser extent
and the ovarian pregnancy not at all. There is relative increase in tubal pregnancy (7 times
more) should pregnancy occur with IUD in situ.
(b) Sterilization operation—There is 15–50% chance of being ectopic if pregnancy occurs.
(c) Use of progestin only pill or postcoital estrogen preparations increases the chance of tubal
pregnancy probably by impaired tubal motility.
2. Tubal surgery—Tubal reconstructive surgery to improve fertility, increases the risk of
tubal pregnancy significantly.
3. Intrapelvic adhesions following pelvic surgery
4. ART—Tubal pregnancy is increased following ovulation induction and IVF-ET and GIFT
procedures.
5. Others
• Previous ectopic pregnancy
• Prior induced abortion
• Developmental defects of the tube: (a) Elongation. (b) Diverticulum. (c) Accessory ostia.
• Transperitoneal migration of the ovum—contralateral presence of corpus luteum is
noticed in tubal pregnancy in about 10% cases
Risk Factors of Ectopic Pregnancy
• History of PID
• History of tubal ligation
• Contraception failure
• Previous ectopic pregnancy
• Tubal reconstructive surgery
• History of infertility
• ART particularly if the tubes are patent but damaged
• IUD use
• Previous induced abortion
• Tubal endometriosis
Pathogenesis
The mechanisms responsible for ectopic implantation are unknown.
The four main possibilities are
• an anatomic obstruction to the passage of the zygote,
• an abnormal conceptus,
• abnormalities in the mechanisms responsible for tubal motility,
• transperitoneal migration of the zygote.
Classification
According to the site of implantation
A) Extrauterine
1. Tubal – Ampulla, isthmus, infundibulum, interstitial
2. Ovarian
3. Abdominal – Primary (rare)
Secondary – intraperitoneal, Extraperitoneal
B) Uterine
1. Cervical
2. Angular
3. Cornual
4. Caesarean scar
Clinical features of tubal rupture
• Sudden onset of severe abdominal pain
• Vaginal bleeding
• Very quickly worsening of condition – Haemorrhagic shock
Signs -
• Pale colour
• Features of shock – rapid pulse, hypotension, cold, clammy extremities
• Abdominal examination: Abdomen (lower abdomen)—tense, tumid, tender. No mass is
usually felt, shifting dullness present, bowels may be distended. Muscle guard—usually
absent
• Pelvic examination is less informative due to extreme tenderness and it may precipitate
more intraperitoneal haemorrhage due to manipulation. The findings are: (i) Vaginal
mucosa—blanched white. (ii) Uterus seems normal in size or slightly bulky. (iii) Extreme
tenderness on fornix palpation or on movement of the cervix. (75%) (iv) No mass is
usually felt through the fornix. (v) The uterus floats as if in water.
Treatment
In case of tubal rupture emergency surgery is required – laparoscopy/laparotomy
Indications for laparotomy
• Haemorrhagic shock
• Severe intra-abdominal adhesions
• Non diagnosed conditions
Salpingectomy is the definitive surgery. The excised tube should be sent for histological
examination
The ipsilateral ovary and its vascular supply is preserved. Oophorectomy is done only if the
ovary is damaged beyond salvage or is pathological.
Keeping the tube with only removing the pregnancy – Milking
Serum hCG levels should normalize after 1 month of treatment. (Max 3 months) If not need
medical advice. -Repeat surgery and tube removal or chemotherapy

Conservative therapy – Chemotherapy ( in unruptured tubal pregnancy)


• In progressive pregnancy
• Results will be negative mostly
• Tube will become non-functional and can lead to tubal abscess

57. Trichomonas colpitis. Clinical symptoms. Diagnostics. Therapy


Causative Organism: It is caused by Trichomonas vaginalis, a pear-shaped unicellular flagellate
protozoon
Mode of Transmission – sexual contact
Clinical symptoms
• sudden profuse and offensive vaginal discharge often dating from the last
menstruation.
• Irritation and itching of varying degrees within and around the introitus are common.
• presence of urinary symptoms such as dysuria and frequency of micturition.
Signs
• thin, greenish-yellow and frothy offensive discharge per vaginum.
• vulva is inflamed with evidences of pruritus.
• Vaginal examination may be painful. The vaginal walls become red and inflamed with
multiple punctate hemorrhagic spots. Similar spots are also found over the mucosa of
the portio vaginalis part of the cervix on speculum examination giving the appearance of
‘strawberry’
Diagnostics
(a) Identification of the trichomonas is done by hanging drop preparation. If found negative
even on repeat examination, the confirmation may be done by culture.
(b) Culture of the discharge collected by swabsin Diamond’s TYM or Feinberg Whittington
medium
Hanging drop preparation
The small amount vaginal discharge is dropped over a slide and then mixed with one drop of
normal saline. It is then covered with a coverslip. Actively motile trichomonads can be seen
under microscope easily. It can be effectively visualized after staining with 1 percent brilliant
cresyl violet; leukocytes and other bacteria will not take up the dye

Treatment
Metronidazole 200 mg thrice daily by mouth is to be given for 1 week. (Most effective)
Or
Tinidazole single 2 gm dose PO
Partner also should be given the same treatment schedule for 1 week
The husband should use condom during coitus irrespective of contraceptive practice until the
wife is cured

58. Prevention of endometrial cancer. Groups of risk


Prevention -
• Prevention from inflammatory diseases
• Prevention from somatic diseases – like diabetes
• Maintain healthy body weight – absence of obesity
• Breast feeding following normal pregnancy
• Hormonal contraception use – progesterone is most effective
• Appropriate treatment for benign and premalignant pathologies of the uterus
• Regular examination
Groups of risk
Any incidence which cause abnormal ratio of oestrogen and progesterone (relative increase of
oestrogen) increases risk of endometrial CA.
• Women with abnormal menstrual cycle
• Nulliparity
• Late menopause
• Absence of breastfeeding
• Somatic diseases – diabetes, obesity
• Peri / post-menopausal age
• Unopposed oestrogen stimulation - functioning ovarian tumours, PCOS, Unopposed
estrogen replacement therapy in postmenopausal women

59.Ectopic pregnancy. Surgical and conservative methods of treatment.

Conservative management may be either medical or surgical. Otherwise salpingectomy is done.

The advantages of conservative management are:


(1) Significant reduction in operative morbidity, hospital stay as well as cost
(2) Improved chance of subsequent intrauterine pregnancy
(3) Less risk of recurrence.
Medical management: Number of chemotherapeutic agents have been used either
systemic or direct local (under sonographic or laparoscopic guidance)
as medical management of ectopic pregnancy. The drugs commonly used for
salpingocentesis are: methotrexate, potassium chloride, prostaglandin (PGF2α), hyperosmolar
glucose or actinomycin.
The patient must be (i) hemodynamically stable (ii) Serum hCG level
should be < 3000 IU/L (iii) tubal diameter should be less than 4 cm without any fetal cardiac
activity (iv) There should be no intra-abdominal hemorrhage. For systemic therapy, a single
dose of methotrexate (MTX) 50 mg/M2 is given intramuscularly.
Monitoring is done by measuring serum β hCG
. When the decline in hCG between

Conservative Surgery: The procedure can be done either laparoscopically or by


microsurgical
laparotomy.
1. Linear Salpingostomy: A longitudinal incision is made on the antimesenteric border directly
over the site of ectopic pregnancy. After removing the products (by fingers, scalpel handle or by
suction), the incision line is kept open to be healed
later on by secondary intention. Hemostasis is achieved by electrocautery or laser
2. Linear Salpingotomy: The procedures are the same as those of salpingostomy. But the
incision line is closed in 2 layers
with 7-0 interrupted vicryl sutures. This is not commonly done.
3. Segmental Resection: This is of choice in isthmic pregnancy. End to end anastomosis can be
done immediately or at
a later date after appropriate counselling of the patient.
4. Fimbrial Expression: This is ideal in cases of distal ampullary (fimbrial) pregnancy and is done
digitally.

Salpingectomy is done when (i) whole of the affected tube is damaged (ii) contralateral tube is
normal or (iii) future fertility is not desired.

abdominal pregnancy
Laparotomy: The ideal surgery is to remove the entire sac-fetus, the placenta and the
membranes. This
may be achieved if the placenta is attached to a removable organ like uterus or broad ligament.
If however,
the placenta is attached to vital organs, it is better to take out the fetus and leave behind the
placenta and the sac, after tying and cutting the cord flushed with its placental attachment. In
such a situation, placental activity is to be monitored by quantitative serum β-hCG level and
ultrasound. Complete absorption of the left behind placenta occurs through aseptic autolysis
60) womens infertlitiy. Reasons, diagnostics, principles of therapy.- refer q 41

61. Bacterial vaginosis. Etiology, clinic, diagnostic and therapy.


Etiology: For unknown reasons, the vaginal flora’s symbiotic relationship shifts to one in
which there is overgrowth of anaerobic species including Gardnerella vaginalis, Ureaplasma
urealyticum, Mobiluncus species, Mycoplasma hominis, and Prevotella species. Bacterial
vaginosis (bv) (bacterial vaginitis) is also associated with a signifi cant reduction or absence
of the normal hydrogen peroxide-producing Lactobacillus species. Whether an altered
ecosystem leads to lactobacilli disappearance or whether its disappearance results in the
changes observed with BV is unclear. Clinically, it is characterized by creamy vaginal discharge
with fishy smell without extensive evidence of inflammation.

Clinical Features: Bacterial Vaginosis (BV) is characterized by malodorous vaginal discharge.


The term vaginosis is preferred as there is no vaginal inflammation. The discharge is
homogeneous, greyish-white and adherent to the vaginal wall. Clinical implications of BV, in
pregnancy are, preterm rupture of membranes, preterm labor, and chorioamnionitis.

Diagnosis:
(A) Amsel’s four diagnostic criteria are:

• Homogeneous vaginal discharge.


• Vaginal pH > 4.5 (litmus paper test).
• Positive whiff tests (see below).
• Presence of clue cells (> 20% of cells).

(B) Gram stained vaginal smear (Hay/Ison): Presence of more Gardnerella or mobiluncus
morphotypes with few or absent lactobacilli.

Whiff Test: Fishy (amine) odor when a drop of discharge is mixed with 10 percent potassium
hydroxide solution.

Clue cells: A smear of vaginal discharge is prepared with drops of normal saline on a glass slide
and is seen under a microscope. Vaginal epithelial cells are seen covered with these
coccobacilli and the cells appear as stippled or granular. At times, the cells are so heavily
stippled that the cell borders are obscured. These stippled epithelial cells are called “clue
cells”. Presence of clue cells ( >20% of cells) are diagnostic of BV.

Treatment: is highly effective with metronidazole — 200 mg orally thrice daily for 7 days.
Clindamycin cream (2%) and metronidazole (0.75%) gel are recommended for vaginal
application daily for 5 days to prevent obstetric complications. The patient’s sexual partner
should be treated simultaneously. Cure rate is 80%.

62. Management of patients with benign cervical pathology. Methods of conservative and
surgical treatment.
Benign cervical pathologies are,
• Cervical ectopy or erosion
• Chronic cervicitis
• Cervical polyp
• leukoplakia
• Cervical tear and eversion
• Cervical cyst
• Elongation of the cervix
All cases should be subjected to cytological examination from the cervical smear to exclude
dysplasia or malignancy.
Watchful regular observation is a must.
Conservative treatment is indicated in younger women and in mild degree.
Surgical procedures that preserve the uterus and may permit future childbearing only when the
disease process is much disturbing the lifestyle.
In cases such as premalignant, surgery includes cryosurgery (freezing), laser surgery, loop
electrosurgical excision procedure (LEEP) or cold-knife conization. Cryosurgery, laser surgery, and
LEEP can be performed in the out-patient office or short procedure facility, often with local
anesthesia. A cold-knife conization is a more extensive operation that involves removal of part of
the cervix under general anesthesia. Not all patients can be adequately treated with cryosurgery,
laser surgery or LEEP. This decision depends on the extent and appearance of the disease upon
examination.
In addition to that symptomatic management with pain medications and broad spectrum
antibiotics should be given.
Women treated with conservative surgery require lifelong visits to their doctor to ensure that
recurrence of cervical disease can be detected in the precancerous state or early while the cancer
is still curable.

63) Diagnostic methods of hyperplastic processes of endometrium.


There is no classic symptom of premalignant lesions. But the constant feature is abnormal
perimenopausal uterine bleeding and ultimate diagnosis is by uterine curettage and histology.
Accidental diagnosis is made during investigation of infertility, DUB, PCOS or excised specimen
of
removed uterus.
Diagnosis by screening procedures extended to ‘at risk’ women is not as effective like that of
CIN.
Vaginal pool smear, endometrial aspiration (pipelle endometrial sampling), endometrial biopsy
(curettage) and vaginal ultrasound are the different methods available for screening.
Histology
Simple hyperplasia: Endometrium is thick. The
glands are dilated and have outpouching and invaginations.
They are crowded and have irregular
outlines. The stroma is denser and more cellular.

Complex hyperplasia: Endometrium is thicker. The gland are crowded and arranged back to
back with reduced stroma. Most glands have irregular outlines. There are papillary processes
and intraluminal bridges within the glands. Epithelial pseudostratification is present.
Atypical hyperplasia: The endometrial glands have cytologic atypia. The gland outlines are of
complex hyperplasia in type. The nuclei of the glands show enlargement, irregular size and
shape, hyperchromasia and coarse chromatin.

Carcinoma-in-situ: Commonly describes a lesion with severe cytologic as well as architectural


abnormalities
of the glands.

64. Acute Endometritis


Clinics
• Lower abdominal pain
• Fever with shivering
• Purulent vaginal discharge
• Lower abdominal tenderness
• PV: enlarged tender uterus, tenderness on movement of the cervix and uterus
Diagnostics
• Lab. Findings - increase leukocytosis, increase ESR
• Imaging- Ultrasonography
• History of termination of pregnancy, intrauterine procedure, using IUCD
Treatment
Outpatient therapy: (i) Ofloxacin 400 mg PO twice daily for 14 days plus metronidazole 500 mg
PO twice daily for 14 days (see Table 10.4) are given. Patient is admitted for inpatient therapy if
there is no response by 72 hours.
Inpatient therapy (Temp >39°C, toxic look, lower abdominal guarding, and rebound
tenderness). Clindamycin 900 mg IV 8 hourly, plus gentamicin 2 mg/kg IV, then 1.5 mg/kg IV
every 8 hours are given. This is followed by doxycycline 100 mg twice daily orally for 14 days. IV
fluids to correct dehydration and nasogastric suction in the presence of abdominal distension or
ileus are maintained. Laparotomy is done if there is clinical suggestion of abscess rupture.
Chronic Endometritis
Clinics
• Women often presents with purulent or seropurulent vaginal discharge.
• Lower abdominal pain

• Fever with shivering


• Lower abdominal tenderness
• PV: enlarged tender uterus, tenderness on movement of the cervix and uterus
Diagnosis
• is made by cervical smear, culture of the discharge, transvaginal ultrasonography and
histology, of the endometrium.
Treatment:
• The offending cause is to be removed or eradicated. Levofloxacin 500 mg PO daily for 14
days with Metronidazole 400 mg PO twice daily for 14 days are given.

65. Characteristics of normal menstrual cycle. (refer question 2)

66. Myoma of uterus. Modern ideas of etiopathogenesis. Classification, clinic, diagnostics.


Etiology
The etiology of this common tumor is not known. Leiomyomas are not
detectebale before puberty and, being hormonally responsive,
normally grow during the reproductive year.

Dysregulation of these hormones cause myoma


➢ Vascular endothelial growth factor
► Heparin-binding epidermal growth
factor
► Platelet-derived growth factor
► Transforming growth factor ,
► Parathyroid hormone-related protein
► Prolactin
► Fibroblast growth factor
Pathogenesis
Myomas contains estrogen receptors in higher concentrations than in the surrounding
myometrium, but in lower concentrations than in endometrium. Data concerning progesterone
receptors in myoma are inconsistent. Myoma generally increase in size with estrogen therapy
and during pregnancy but decrease in size and even disappear following menopause.
Classification –
Typical –
1. Subserosal - outside surface on the of the uteru
2. Intramural - within the muscular wall
of the uterus
3. Submucous - bulging in to the uterine cavity

Atypical –
Intra ligament, intra peritoneal
abdominal, vaginal-
cervical, pedunculated
Complaints
► Submucosal„ dysmenorrhoea, sever pain
► Subserous„ without pain (only if large)
► Interstitial „ pain in abdominal cavity
A. Common symptoms
1. Pressure effects
2. Abnormal Uterine Bleeding
Menorrhagia (prolonged or heavy menstrual flow)
3. Pain
B. Less common symptoms
1. Infertility
2. Pregnancy complications
a. Recurrent Miscarriage
b. Premature labor
c. Fetal Malpresentation
d. Labor complications
e. spontaneous abortion

Diagnostics –
Abdominal exam -Uterus palpable above symphysis pubis
Bimanual examination -Enlarged, mobile and irregular uterine contou
The swelling is dull on percussion.
Feel is firm, more toward hard; may be cystic in
cystic degeneration.
y Margins are well-defined y Surface is nodular; may be uniformly enlarged in a single fibroid.
y Mobility is restricted from above downwards but can be moved from side to side.

Ultrasound and Color Doppler (TVS)- assess the myoma location, dimensions volume.
Saline Infusion Sonography (SIS)- detect any submucous fibroid or polyp
Magnetic resonance imaging (MRI)- differentiate adenomyosis from fibroids
Laparoscopy- pelvic endometriosis and tubal pathology can be revealed.
Hysteroscopy is of help to detect submucous fibroid

67. The role of human papillomavirus infection in a uterus neck cancer genesis. Modern
methods of diagnostics and prophylaxis

HUMAN PAPILLOMA VIRUS (HPV)


HPV is epitheliotropic and plays an important role in the development of CIN. HPV infected
cells (koilocytes) are characterized by enlarged cells with perinuclear halos. The nucleus is large,
Irregular and hyperchromatic. Depending on their oncogenic potential, HPV types are broadly

Grouped into two.


♦ High oncogenic risk—Types 16, 18, 31, 33, 35, 45, 56.
♦ Low oncogenic risk—Types 6, 11, 42, 43.

Pathogenesis of HPV Infection: HPV is epitheliotropic. Cervical epithelium → Infection


(Latent/Active with virus replication) → Oncogenic HPV DNA integration to human genome →
up-regulation of viral oncogenes → expression of E6 and E7 oncoproteins → Interference of
tumor suppressor genes (p53 and Rb) → host cell immortalization and HPV induced neoplastic
transformation.

Diagnosis
*Pap smear test – exfoliative cytology -The smear should contain cells from SCJ, TZ and the
endocervix.
Ayer’s spatula and an endocervical brush is used for the purpose. Cells are spread on a single
slide and
fixed immediately
*HPV DNA test -HPV DNA detection in cervical tissues may be a screening procedure as that of
Pap smear. Polymerase Chain reaction or southern blot or hybrid capture (HC) technique is
used for HPV DNA detection.
*Visual inspection with acetic acid -acetowhite lesions are considered for colposcopic
examination and/or biopsy
* Colposcopy
* Biopsy with or without colposcopy
Prophylaxis
*HPV vaccines
* To delay sexual exposure until the cervical epithelium, especially in the transformation zone,
has attained physiological maturity.
* To maintain a local hygiene and to treat vaginal infections.
* To use condom specially during early sexual life.
*To maintain penile hygiene as it may be the reservoir for high risk HPV.
* Reducing or quitting smoking reduces CIN.

68)Progressing extrauterine pregnancy ,clinic diagnostic, treatment


Ectopic pregnancy is implantation of fetus outside of uterus .eg fallopian tubes ,
ovaries,rectum,miometrium,bowel. Ectopic pregnancy can be progressed due to rupture of
tubes causing acute abdomen signs .
Clinical signs- sever acute abdominal pain
-tachycardia
- chest pain
- tenderness
- muscle rigidity
Diagnosis
Urine HCG - confirm pregnany
Tranvaginal ultrasound- visible ectopic pregnancy, if ruptured blood in peritoneum
Laproscopy- gold standard

Treatment
*Conservative ( if patient asymptomatic)
*Medical management
Methotrexate
*Surgical management
Rupture of ectopic pregnancy
Main two methods are
laparatomy - if patient heamodynamically unstable and present heamoperitonium
laparoscopy- patient heamodynamically stable
Salphyngiotomy- remove pregnancy only
Salphyngoectomy- remove tube together with pregnancy
If contralateral tube is macroscopically abnormal salpyngotomy is performed.but there is risk of
tubal narrowing in the future and recurrence of ectopic pregnancy.
If contralateral tube is normal macroscopically salpyngectomy is done ,

69. The prophylaxis of inflammatory diseases of female genitals.


• Practice safe sex - Use condoms every time you have sex, limit your number of partners
and ask about a potential partner's sexual history.
• Talk to your doctor about contraception - A contraceptive intrauterine device (IUD) may
increase your risk of PID temporarily for the first few weeks after insertion, but a barrier
method, such as a condom, reduces your risk.
• Use of a birth control pill alone offers no protection against acquiring STIs. But the pill may
offer some protection against the development of PID by causing your body to create
thicker cervical mucus, making it more difficult for bacteria to reach your uteru. It's still
important to use a condom every time you have sex, however.
• Get tested - Set up a regular screening schedule with your doctor, if you need to. Early
treatment of an STI gives you the best chance of avoiding pelvic inflammatory disease. PRE
• Request that your partner be tested. If you have pelvic inflammatory disease or an STI,
advise your partner to be tested and, if necessary, treated. This can prevent the spread of
STIs and possible recurrence of PID.
• Don't douche. Douching upsets the balance of bacteria in your vagina.
• Pay attention to hygiene habits. Wipe from front to back after urinating or having a bowel
movement to avoid introducing bacteria from your colon into the vagina.

70. The ascending gonorrhea. Clinic, diagnostics, treatment. Possible consequences for
reproductive function.
Based on the duration of the disease, fresh gonorrhea (from the moment of infection <2
months) and chronic gonorrhea (from the moment of infection >2 months)are
distinguished.

Classical manifestations of the acute form of gonorrhea in women:


• Purulent vaginal discharge
• Hyperemia
• Irritation
• edema and ulceration of mucous membranes;
• frequent and painful urination
• burnin
• itching
• lower abdominal pain.

Ascending gonorrhea
• Gonorrheal endometritis .
✓ liquid purulent-serous or sacral discharge from the genital tract
✓ menstrual disorders of the type of hyperpolymenorrhea can be
observed sometimes acyclic uterine bleeding occurs .
✓ With a delay of purulent contents in the uterine cavity, a pyometra clinic
develops.
• Gonorrheal salpingitis and salpingo-oophoritis .
✓ the formation of a hydrosalpinx is possible , and then a pyosalpinx , and in the
case of the transition of inflammation to the ovary, a pyovar , tubo-ovarian
abscess .
✓ Against the background of an extensive inflammatory process in the small
pelvis, a pronounced adhesion process is formed.
• Gonorrheal pelvioperitonitis .
✓ sharp pains in the lower abdomen with irradiation to the epigastrium and
mesogastrium, symptoms of muscle protection.
✓ The temperature is febrile values, vomiting, gas and stool retention
✓ Peritonitis rarely develops, since the rapid formation of adhesions delimits the
inflammatory process from the abdominal cavity

Diagnosis
• During a vaginal examination
a slightly enlarged, painful uterus, a conglomerate of fallopian tubes and ovaries welded
together, can be palpated.In order to confirm the diagnosis, material is taken from the vagina,
cervical canal, urethra, rectum, oral cavity, conjunctiva (depending on the localization of the
primary focus).
• Laboratory diagnostic tests include
microscopy of smears with a Gram stain
culture of secretions for gonococcus
examination of scrapings by PCR and PIF
Serological studies (RIF, ELISA, RSK) do not allow differentiating previously transferred and
current gonorrhea in women

Treatments
✓ With fresh gonorrhea in women with lesions of the lower urinary tract, a single dose or
administration of an antibiotic - ceftriaxone, azithromycin, ciprofloxacin, cefixime
The course of treatment for ascending gonorrhea or mixed infection is extended to 7-10 days
Local treatment includes washing the urethra with 0.5% solution of silver nitrate washing
vagina with antiseptics - solutions of potassium permanganate, chlorhexidine
treatment of the sexual partner.
✓ Chronic gonorrhea
Check for antibiotic sensitivity and increase duration of therapy
With complicated forms of gonorrhea in women (tubo-ovarian abscess, pyosalpinx, etc.),
surgical treatment is indicated
• Draining abscess
• Removal of affected tubes
• Remove adhesions

71. Diagnostic methods of hyperplastic processes of endometrium.

• Cytological examination - This method allows to determine the severity of proliferative


changes, but does not give a clear idea of its pathomorphological structure.

• Transvaginal ultrasound scanning - can evaluate the state of the endometrium, also the
myometrium, identify adenomyosis, myoma of the uterus. Also, ultrasound should be
performed to determine the size of the ovaries and evaluate their functions. Diagnosis
of endometrial hyperplasia in ultrasound is based on the detection of increased in the
anteroposterior size of the median maternal echo (M-echo) with increased acoustic
density.

• Hydro sonography - The ultrasonic picture of the endometrial polyps shows ovoid, less
often rounded inclusions in the structure of the M-echo and the uterine cavity of
increased echolocation

• hysteroscopy - It assess the condition of the walls of the uterus, identify adenomyosis,
submucous uterine fibroids and other forms of pathology. Atypical endometrial
hyperplasia does not have characteristic endoscopic criteria, the hysteroscopic pattern
resembles the usual glandular-cystic hyperplasia. In severe atypical hyperplasia,
glandular polypoid growths of faint yellowish or grayish color can be identified.

• Histological examination of scrapings of the mucous membrane of the uterus - the final
method for diagnosing hyperplastic endometrial processes

72. Apoplexy of ovaries. Causes, clinics, diagnostics, treatment


Symptom : sudden onset abdominal pain, occasional dizziness or syncope if develops
hemoperitoneum 
Sign abdominal tenderness, rebound tender from peritoneal irritation
PV : palpable mass
Diagnosis : pregnancy test,  CBC,  Ultrasound or culdocentesis (Hct ≤16% not a
hemoperitoneum)
Management : culdocentesis
1. Fresh blood - corpus luteum
2. Chocolate “old” blood - endometrioma
3. Oily sebaceous fluid - benign teratoma
4. Purulent fluid - PID or TOA  Patient who do not have hemoperitoneum can be observe in the
hospital, without surgical intervention

73. Types of hormonal contraception. Indications, contraindications, complications.

COMBINED ORAL CONTRACEPTIVES (PILLS)


The combined oral steroidal contraceptives is the most effective reversible method of
contraception. In the combination pill, the commonly used progestins are either levonorgestrel
or norethisterone or desogestrel and the estrogens are principally confined to either ethinyl-
estradiol or menstranol (3 methylether of ethinyl-estradiol).

Mode of action: The probable mechanism of contraception are:


Inhibition of ovulation — Both the hormones synergistically act on the hypothalamopituitary
axis. The release of gonadotropin releasing hormones from the hypothalamus is prevented
through a negative feedback mechanism. There is thus no peak release of FSH and LH from the
anterior pituitary. So follicular growth is either not initiated or if initiated, recruitment does not
occur.
Producing static endometrial hypoplasia— There is stromal edema, decidual reaction and
regression of the glands making endometrium nonreceptive to the embryo.
Alteration of the character of the cervical mucus (thick, viscid and scanty) so as to prevent
sperm penetration. Probably interferes with tubal motility and alters tubal transport. Thus,
even though accidental breakthrough ovulation occurs, the other mechanisms prevent
conception.

Estrogen inhibits FSH rise and prevents follicular growth. It is also useful for better cycle control
and to prevent breakthrough bleeding.
Progestin: Anovulatory effect is primarily by inhibiting LH surge. It is also helpful to counteract
the adverse effects of estrogen on the endometrium (endometrial hyperplasia and heavy
withdrawal bleeding). It is also responsible for changes in the cervical mucus (vide supra).
How to prescribe a pill: Instruction: New users should normally start their pill packet on day one
of their cycle. One tablet is to be taken daily preferably at bed time for consecutive 21 days. It is
continued for 21 days and then have a 7 days break, with this routine there is contraceptive
protection from the first pill. Next pack should be started on the eighth day, irrespective of
bleeding (same day of the week, the pill finished). Thus, a simple regime of “3 weeks on and 1
week off ” is to be followed. Packing of 28 tablets, there should be no break between packs.
Seven of the pills are dummies and contain either iron or vitamin preparations. However, a
woman can start the pill up to day 5 of the bleeding. In that case she is advised to use a condom
for the next 7 days. The pill should be started on the day after abortion. Following childbirth in
non-lactating woman, it is started after 3 weeks and in lactating woman it is to be withheld for
6 months.
Follow-Up: The patient should be examined after 3 months, then after 6 months and then
yearly. The patient above the age 35 should be checked more frequently. At each visit, any
adverse symptoms are to be noted. Examination of the breasts, weight and blood pressure
recording and pelvic examination including cervical cytology, are to be done and compared with
the previous records.
Missed pills: Normally there is return of pituitary and ovarian follicular activity during the pill-
free interval (PFI) of 7 days. Breakthrough ovulation may occur in about 20 percent cases during
the time. Lengthening of PFI due to omissions, malabsorption, or vomiting either at the start or
at the end of a packet, increases the risk of breakthrough ovulation and therefore pregnancy.
Management: When a woman forgets to take one pill (late up to 24 hours), she should take the
missed pill at once and continue the rest as schedule. There is nothing to worry. When she
misses two pills in the first week (days 1–7), she should take 2 pills on each of the next 2 days
and then continue the rest as schedule. Extra precaution has to be taken for next 7 days either
by using a condom or by avoiding sex.
If 2 pills are missed in the third week (days 15–21) or if more than two active pills are missed at
any time, another form of contraception should be used as back up for next 7 days as
mentioned above. She should start the next pack without a break. If she misses any of the 7
inactive pills (in a 28-day pack only) she should throw away the missed pills. She should take the
remaining pills one a day and start the new pack as usual.
The major complications are:
Depression: Low dose estrogen preparations are not associated with depression.
Hypertension (OGN): Current low dose COCS rarely cause significant hypertension. Pre-existing
hypertension is likely to be aggravated. Changes are seen only in systolic but not in diastolic
blood pressure. The effect on blood pressure is thought to involve the renin-angiotensin
system. There is marked increase in plasma angiotensinogen. The changes however reverse
back to normal 3–6 months after stoppage of pill.
Vascular complications (OGN): (a) Venous thromboembolism (VTE) — The overall risk is to the
extent of 3–4 times more than the non-users. Pre-existing hypertension, diabetes, obesity
thrombophilias (inherited or acquired) and elderly patient (over 35 especially with smoking
habits) are some of the important risk factors.
Cholestatic jaundice—Susceptibility is increased in women with previous history of idiopathic
recurrent jaundice in pregnancy or hepatitis.
Neoplasia (OGN) — Combined oral contraceptives (COCs) reduce the risk of epithelial ovarian
(50% ↓) and endometrial (50% ↓) carcinoma. This protective effect persists for 10–15 years
even after stopping the method following a use of 6 months to 1 year.

PROGESTIN ONLY CONTRACEPTION (POP/MINI PILL)


POP is devoid of any estrogen compound. It contains very low dose of a progestin in any one of
the following form — Levonorgestrel 75 µg, norethisterone 350 µg, desogestrel 75 µg,
lynestrenol 500 µg or norgestrel 30 µg. It has to be taken daily from the first day of the cycle.
Mechanism of action: It works mainly by making cervical mucus thick and viscous, thereby
prevents sperm penetration. Endometrium becomes atrophic, so blastocyst implantation is also
hindered. In about 2 percent of cases ovulation is inhibited and 50 percent women ovulate
normally.
How to prescribe mini pill: The first pill has to be taken on the first day of the cycle and then
continuously. It has to be taken regularly and at the same time of the day. There must be no
break between the packs. Delay in intake for more than 3 hours, the woman should have
missed pill immediately and the next one as schedule. Extra precaution has to be taken for next
2 days.
Contraindications: (i) Pregnancy (ii) unexplained vaginal bleeding (iii) recent breast cancer (iv)
arterial disease. (v) thromboembolic disease.

INJECTABLE PROGESTINS
The preparations commonly used are depomedroxy progesterone acetate (DMPA) and
norethisterone enanthate (NET-EN). Both are administered intramuscularly (deltoid or gluteus
muscle) within 5 days of the cycle. The injection should be deep, Z-tract technique and the site
not to be messaged. DMPA in a dose of 150 mg every three months (WHO 4 months) or 300 mg
every six months; NET-EN in a dose of 200 mg given at two-monthly intervals. Depo-Sub Q
provera 104, contains 104 mg of DMPA. It is given subcutaneously over the anterior thigh or
abdomen. It suppresses ovulation for 3 months as it is absorbed more slowly.
Mechanism of action: (1) Inhibition of ovulation — by suppressing the mid cycle LH peak (2)
cervical mucus becomes thick and viscid thereby prevents sperm penetration (3) Endometrium
is atrophic preventing blastocyst implantation.

Contraindications: Women with high risk factors for osteoporosis and the others are same as in
POP (

74. Hysteromyoma. Modern ideas of etiopathogenesis. Classification, clinic, diagnostics.


Hysteromyoma/ uterine leiomyoma/ fibroid are benign (noncancerous) proliferative tumors of
the smooth muscles of the uterus. Tumor is composed of smooth muscle and fibrous
connective tissue.
Etiology
• The etiology still remains unclear.
• The prevailing hypothesis is that, it arises from the neoplastic single smooth muscle cell of
the myometrium. The stimulus for initial neoplastic transformation is not known. The
following are implicated:
1. Chromosomal abnormality—particularly the chromosome six or seven (rearrangements,
deletions). Somatic mutations in myometrial cells may also be the cause for
uncontrolled cell proliferation.
2. Role of polypeptide growth factors—Epidermal growth factor (EGF), insulin-like growth
factor-1 (IGF-1) transforming growth factor (TGF), stimulate the growth of leiomyoma
either directly or via estrogen.
3. A positive family history is often present
Risk factors
Increased Risk ( +ve risk factors )
1. Age :- Middle-aged women are commonly diagnosed with fibroids
2. Nulliparity
3. Weight :- Obese women are at a higher risk for fibroids
4. Family history :- a positive family history
5. Race :- African-American women tend to have the highest frequency of fibroids,
although fibroids have been found in women of all ethnic backgrounds
6. Infections- PID, endometriosis
7. presence of IUD

Decreased Risk ( -ve risk factors )


1. tаbacco use
2. prior child birth
3. Oral contraceptive use
Pathogenesis
• It is predominantly an estrogen-dependent tumor.
• Myomas contains estrogen receptors in higher concentrations than in the surrounding
myometrium, but in lower concentrations than in endometrium. Data concerning
progesterone receptors in myoma are inconsistent.
• Myoma generally increase in size with estrogen therapy and during pregnancy but
decrease in size and even disappear following menopause.
• Rapid growth may also be due to degeneration or due to malignant change.
Classification
According to anatomy/ location
I. Typical
1. Subserosal - outside surface on the of the uterus (in the abdominal cavity)
2. Intramural/ Interstitial - within the muscular wall of the uterus (in the myometrium)
3. Submucous - bulging in to the uterine cavity (inside uterine cavity)
II. Atypical uterine myoma
1) intraligementous
2) intraperitoneal
3) cervical
4) pedunculated - attached to the uterus by a narrow pedicle containing blood vessels
а) pedunculated abdominal
в) pedunculated fundal
с) pedunculated vaginal
According to growth
1. Progressive
2. Stable
3. Regenerative
According to symptoms
1. Symptomatic
2. Asymptomatic

Clinical picture
Complains
► Submucous - dysmenorrhoea, sever pain
► Subserousal- without pain (only if large)
► Interstitial - pain in abdominal cavit
Symptoms
A. Common symptoms
1. Pressure effects - Pressure on bladder may result in suprapubic discomfort, frequent
urination, difficulty in urination. Pressure on rectosigmoid may result in low back pain.
2. Abnormal Uterine Bleeding; Menorrhagia (prolonged or heavy menstrual flow),
Secondary anemia
3. Pain- from twisted, pedunculated myomas or degenerating, hemorrhagic or infected
myomas, necrosis
B. Less common symptoms
1. Infertility- due to submucous myomas or with distortion of uterine cavity
2. Pregnancy complications
a. Recurrent Miscarriage
b. Premature labor
c. Fetal malpresentation
d. Labor complications
e. spontaneous abortion

► Majority are asymptomatic and are only suspected from pelvic examination
► Edema and varicosities of the lower extremities may result from large tumors
► Rapid growth particularly in peri-menopausal or postmenopausal may indicate sarcoma.
Diagnosis
Physical examination
• Abdominal exam - Uterus palpable above symphysis pubis
• Bimanual examination- Enlarged, mobile and irregular uterine contours
Laboratory:
• CBC- anemia present
• Hormonal disbalance
• Pregnancy test
Instrumental
• Ultrasonography- to distinguish ovarian mass, can determine location, size, structure. (but
cannot DD from polyps)
Ultrasound and Color Doppler (TVS) findings are:
(i) Uterine contour is enlarged and distorted.
(ii) Depending on the amount of connective tissue or smooth muscle proliferation,
fibroids are of different echogenecity-hypoechoic or hyperechoic.
(iii) Vascularization is at the periphery of the fibroid.
(iv) Central vascularization indicates degenerative changes.
• Hysteroscopy (and needle biopsy)- may help diagnose submucousal myomas, during this
procedure remove the pathological mass and after do biopsy ( can DD from polyps)
• Laparoscopy- after US if mass located outside the uterus laparoscopy is done. Used to DD
from ovarian fibroids. May be useful in complex cases and in ruling out other pelvic
pathology
• Magnetic resonance imaging (MRI)—is more accurate compared to ultrasound. It helps to
differentiate adenomyosis from fibroids.
• Imaging of the uterine tract - to exclude hydronephrosis

75. Gestational trophoblastic diseases


Persistent hydatidiform mole
Complete molar pregnancy
Clinical features
▪ Very high HCG mimics TSH, FSH , LH
▪ Cherry like clusters per vargina
▪ Nausea, vomiting
▪ Irritability, dizziness , photophobia
▪ Nervousness, anorexia , tremor
▪ Enlarge uterus than expected
Diagnosis
*Ultrasound
* Serum HCG levels
Treatment
▪ Immediate D&C under general anesthesia
▪ Follow up
▪ obtain quantitative HCG titre for 48 hrs
▪ serial quantitative HCG weekly untill levels are normal for 3 consective weeks , after HCG
levels normalized serial quantitative HCG monthly for 6 months
▪ Barrier contraceptives should use until HCG level gets normal, after can use hormonal
contraceptives.
Management
Hydatidiform mole has to evacuated.

Partial molar pregnancy


Clinical features
▪ Arise from dispermic fertilization, genome 69XXY, 69 XYY or 69XXX
▪ Characterized by focal hydropic villi and proliferation of cytotrophoblast
▪ There is often fetus through many abnormalities may present eg- hydrocephaly
▪ Oligohydramnious
*Nausea, vomiting
▪ Irritability, dizziness , photophobia
▪ Nervousness, anorexia , tremor
▪ Enlarge uterus than expected
Diagnosis
Ultra sound scan
▪ May reveal fetus with abnormalities or simply gestational sac.
▪ Low amniotic fluid
▪ Enlarged placenta with Swiss cheese apperence
Serum HCG levels

Treatment
▪ Immediate D&C under general anesthesia
▪ Follow up
▪ obtain quantitative HCG titre for 48 hrs
▪ serial quantitative HCG weekly untill levels are normal for 3 consective weeks , after HCG
levels normalized serial quantitative HCG monthly for 6 months
▪ Barrier contraceptives should use until HCG level gets normal, after can use hormonal
contraceptives.
Management
Hydatidiform mole has to evacuated.

Invasive mole
Clinical features
▪ HCG rise or plateau on follow up
▪ Abnormal uterine bleeding
▪Uterine wall may be perforated at multiple areas showing purple, fungating growth with
massive
Intraperitoneal hemorrhage. The neoplasm may invade the pelvic blood vessels and
metastasizes to vagina or distant sites

Diagnosis
• On laparotomy: (a) Perforation of the uterus through which purple fungating growth is
visible. (b) Hemoperitoneum.
*Histology— There is penetration of the uterus by the hyperplastic trophoblastic cells which
still
Retain villus structures. There is no evidence of Muscle necrosis.The materials for uterine
Curettage are often deceptive as the lesion may be deep inside the myometrium.
* Persistent high level of urinary or serum hCG.

Choriocarcinoma
Clinical features
*Persistent ill health.
* Irregular vaginal bleeding, at times brisk.
* Continued amenorrhea.
Other symptoms due to metastatic lesions are:
Lung:Cough, breathlessness, hemoptysis.
Vaginal: Irregular and at times brisk hemorrhage.
Cerebral: Headache, convulsion, paralysis or coma.
Liver: Epigastric pain, jaundice.
signs:
*Patient looks ill.
*Pallor of varying degrees.

Diagnosis
*Bimanual examination reveals subinvolution of the uterus. There may be a purplish red nodule
in the lower-third of the anterior vaginal wall. Unilateral or bilateral enlarged ovaries may be
palpable through lateral fornices.
*Metastatic brain lesion is suspected when the ratio of hcG in spinal fluid/in serum is more
than 1 : 60.
*Chest X-ray: X-ray shows ‘cannon ball’ shadow or ‘snow storm’ appearance due to numerous
tumor emboli.Pleural effusion may be present.
“Pelvic sonography: Sonography helps not only to localize the lesion but to differentiate GTN
from a normal pregnancy.
* Uterine curettage: Pretherapy D and C reduces the intrauterine tumor bulk.
Metastasis finding
Cerebral: The ratio of hCG levels in spinal fluid and serum is higher than 60.
• CT scan or MRI.
*Liver:CT scan; Ultrasonography.
*Chest:X-ray (metastasis); CT may show micro-metastases
Treatment for invasive mole and chorionic carcinoma

76. Maintaining of the


postoperative period for gynecology patients.
Possible complications, their prophylaxis
and treatment.

Vital Signs-
Much information can be obtained by close monitoring of the vital signs, including blood
pressure, pulse, and respiratory rate.
Later, after adequate analgesia and pulmonary function has been obtained, pulse rate
correlates better with intravascular volume status.
After discharge from the recovery room, vitals should be monitored every four hours until
stable and then every eight hours depending on the patient's progress.

Postoperative Activity-
Early ambulation is extremely important after surgery. In addition to improving diaphragmatic
excursion with its subsequent decrease in pulmonary atelectasis, it also prevents the
development of deep venous thrombosis. We normally require that our patients ambulate
within 12 hours of surgery. Thereafter, they are assisted to walk three to four times a day.

Nutrition-
Normal recovery from surgery may include transient loss of appetite and mild nausea. This
usually is secondary to the anesthetic agents and other perioperative medications used.
Symptoms can easily be managed by antiemetics such as Compazine 10 mg IM every four hours
as needed.
Patients with gastrointestinal dysmotility should be initially kept NPO for 48 hours or until
symptoms of nausea resolve. For those who are: severely symptomatic with continued vomiting
and distention, a nasogastric tube should be inserted and placed on suction. As gastric motility
returns, noted by decreased NG output and resolution of nausea, the tube can be removed and
the patient advanced to a clear liquid diet. Prior to oral advancement, the patient should
receive at least 100 grams of glucose in the form of 2 liters of 5% dextrose per day; this will
minimize protein catabolism during this starvation period.

Fluids and Electrolyte-Management


Postoperative fluid management is dependent on:
Current deficits
Maintenance requirements
Abnormal losses
The status of the patient's current conditions should first be determined. Utilizing information
from the patient's history such as preoperative vomiting, bowel distention, oral intake,
intraoperative hemorrhage, extravascular fluid accumulation (third space), and previous fluid
replacement, clues to the patient's fluid deficits can be determined, Physical examination, vital
signs, recent weight change, and record of fluid balance can also help to determine the status
of the intravascular volume and total body water. If uncertainty exists regarding the patient's
actual fluid status, invasive monitoring can be used to measure central venous pressure or left
ventricular filling pressure, (preload), using the Swan-Ganz catheter.

Pain Control-
Liberal use of postoperative analgesics is essential for recovery. Adequate pain control allows
for early ambulation, improved pulmonary toilet, and decreased overall stress. The most
effective regimen for pain control requires small frequent dosing, preferably via the
intravenous route.
With the normal progression of recovery, patients can usually be switched to an oral narcotic
within two to four days. For those who are difficult to wean from IV narcotics intramuscular
injection can be used
An alternative form of pain control is gaining in popularity. This involves placement of an
epidural catheter by an experienced anesthesiologist. The catheter is subsequently infused with
a preservative-free narcotic agent that bathes the epidural space and provides quality pain
control

Wound Care-
Wound care is based on understanding of the biological principles of healing. With the
attainment of epithelial continuity, the operative dressing can be removed after 24–48 hours.
However, if wound drainage is noted upon inspection, a sterile dressing must be replaced until
the drainage ceases and epithelialization is assured.
In healthy individuals with an abdominal incision. sutures or staples can generally be removed
on the fifth postoperative day followed by placement of adhesive strips. This allows for maximal
cosmetic benefit while providing adequate support for wound stability. The timing of the
removal can be adjusted in either direction depending on the importance of each opposing
factor. Thus, individuals who are nutritionally compromised, in whom cosmesis is of less
importance, sutures may be allowed to stay for a longer period of time.
Complications
Shock -
Respiratory compromise -
Postoperative fever - appropriate broad-spectrum antibiotics.
Oliguria - intravenous Lasix, hemodialysis
Thromboembolic Disease-anticoagulation therapy. Heparin is the drug of choice,compression
stockings
Massive, life-threatening pulmonary emboli may require embolectomy

Electrolyte and Acid-Base Abnormalities-


Urinary Retention-Treatment of patients with overdistention dysfunction requires placement of
an indwelling Foley catheter for 24–48 hours, thus allowing recovery of the injured
musculature.

77.Extrauterine pregnancy interrupted by a rupture of the fallopian tube

An ectopic pregnancy is a medical emergency in which a fertilized egg implants itself outside
the uterus. Usually, an ectopic pregnancy is situated in one of the fallopian tubes. As it grows, it
can cause the tube to tear or burst. This results in dangerous internal bleeding.

An ectopic pregnancy is not able to develop into a healthy pregnancy or baby, and the mother
must be treated to avoid risks and complications, such as rupture, that can be life-threatening.

Symptoms

If you have an ectopic pregnancy, you may experience the same symptoms as in any early
pregnancy at first, such as nausea, tiredness, and breast tenderness. Additional signs that the
pregnancy is ectopic most often develop six to eight weeks after your last normal menstrual
period but can occur throughout the first trimester. These symptoms include:

• Pain during intercourse


• Irregular vaginal bleeding or spotting
• Cramping or pain on one side, or in the lower abdomen
• Palpitations

But 50% is asymptomatic

When an ectopic pregnancy causes a rupture, there are additional symptoms.

• Sudden, severe abdominal or pelvic pain


• Dizziness or fainting
• Pain in the lower back
• Pain in the shoulders (due to leakage of blood into the abdomen affecting the
diaphragm)

69/78. The prophylaxis of inflammatory disease of female genital


• Protected sex, wear condom
• Do not have multiple sexual partners
• Regular screening of patient and her partner
• If STD, treatment of both the patient and the partner
• Educate the patient, about the disease, method of transmission and complication
• Good personal hygiene and genital hygiene
• Rinse vulva with soap and water and dry with soap
• Do not share materials like towels, blankets and under wears
• Wear cotton underwear
• Change underwear after use

79. Same as 72

79. Ovarian apoplexy. Causes, clinics, diagnosis and treatment.

Ovarian apoplexy is hemorrhage into the ovarian parenchyma due to follicle rupture in the
ovulatory period or luteal rupture attended by damage to ovarian tissue and bleeding into the
abdominal cavity. Most often ovarian apoplexy occurs at the age of 16 – 30.

Etiology. The main etiological factors are inflammation of the ovary, hormonal disturbances
(increased LH secretion), injuries, overwork, etc. It may be without any visible reason, even during
the sleep.

Clinical picture. The most typical symptom is acute pain in the iliac area spreading throughout the
abdomen and irradiating to the rectum. Pain is accompanied by dizziness, faintness, cold sweat,
and nausea and vomiting.

Severity of the symptomatology is determined by the presence or absence of intra-abdominal


bleeding and its intensity (thus, two types of the disease are distinguished: anemic and algesic).
The internal bleeding is related to skin paleness, tachycardia, and low blood pressure. The
abdomen is bloated, only weakly participating in respiration, and is painful on palpation.
Gynecological examination may elucidate overhanging of the vaginal fornices, tenderness upon
cervical displacement, the uterus of a normal size, and a tender and enlarged ovary.

Treatment. If there are no signs of significant bleeding and the patient feels well, conservative
treatment is possible (bed rest, cold application onto the low abdomen, hemostatic and anti-
inflammatory therapy). In cases of intra-abdominal bleeding (anemic form of apoplexy)
laparotomy and ovarian resection in the presence of hematoma or suturing of bleeding site are
performed.

80. Hormone producing tumors of ovaries. Clinic, diagnostics, treatment.


Ovarian cancer is the fourth most common cause of cancer deaths in women exceeded only by
breast, colon and lung malignancies. It is more common amongst nulliparous.
Malignant cancers of ovaries classified into;
1. Hormone producing tumors
a) Estrogen producing tumors - Granulosa cell tumors, Thecoma
b) Androgen producing tumors - Sertoli-leydig cell tumor
2. Non-hormone producing tumors (most common, early diagnosis difficult because
asymptomatic)
Sertoli-leydig cell tumor (androblastoma, arrhenoblastoma)
Sertoli-Leydig cell tumor is very rare and accounts for less than 0.5% of all ovarian tumors.
They probably arise from the male directed cell rests in the hilum of the ovary, from granulosa
cells or from teratomas. The tumor produces predominantly androgens (80%) and, in some
cases estrogen. The tumors are small, usually unilateral (90%) and solid in consistency. Cut
surface shows yellowish tinge with areas of hemorrhage and necrosis. Microscopic picture
often resembles to various testicular cells such as Sertoli and Leydig cells.
Granulosa cell tumors
Granulosa cell tumors constitute 2% of all ovarian neoplasms. The tumor originates from the
‘rests’ of primitive granulosa cells unused in folliculogenesis. It is the commonest ovarian
stromal tumor. It is bilateral only in 2 percent of cases and is a slow growing tumor. The size
varies, so also the consistency — may be solid or cystic. Cut section is characteristically yellow
or orange due to its lipid content.
As the tumor produces estrogen, there may be associated endometrial hyperplasia (50%).
Unopposed estrogenic stimulation leads to development of endometrial carcinoma in about
5–10 percent of cases.
Thecoma
Thecoma is predominantly a lesion of postmenopausal age. It may occur as a distinct entity
or mixed with granulosa cell tumor. External appearance looks like a fibroma. Cut surface
shows islands of yellow tissue separated by grey fibrous septa. Microscopic picture reveals
cells like that of cortical stroma with areas of granulosa cells.
Due to excess estrogen production, there is endometrial hyperplasia and often associated
with endometrial carcinoma. It is responsible for postmenopausal bleeding. Rarely it may
cause ascites or Meig’s syndrome.

Clinical features:
Androgen producing tumors
• Early symptoms- Abnormal menstrual cycle, amenorrhea
• The androgens produced by the tumor first lead to defeminization — atrophy of the
breasts and uterus and amenorrhea.
• Followed by masculinization (50%) - male type of distribution of hair, hoarseness of
voice, breast atrophy, hirsutism, baldness and clitoral enlargement.
• Serum testosterone level is elevated.
Estrogen producing tumors
• It produces effects caused by hyperestrinism which differs with ages.
➢ Prior to puberty:-Precocious puberty
➢ Childbearing period:- Abnormal uterine bleeding, Abnormal menstrual cycle,
increased duration of bleeding and severe vaginal bleeding during menstruation
➢ Postmenopausal:- Bleeding
May have positive effect on health- skin and vaginal mucosa
become more elastic, decrease hair loss, has irregular menstruation

Non-specific signs of tumor mass;


• Feeling of abdominal distension and vague discomfort.
• Features of dyspepsia such as flatulence and eructations.
• Loss of appetite with a sense of bloating after meals.
• In pre-existing tumor.
− Appearance of dull aching pain and tenderness over one area.
− Rapid enlargement of the tumor.

Gradually, more pronounced symptoms appear. These are:


• Abdominal swelling which may be rapid ( ascites in III stage)
• Dull abdominal pain.
• Sudden loss of weight.
• Respiratory distress — may be mechanical due to ascites or due to pleural effusion
• Dilation of paraumbilical subcutaneous veins visible in eyes (in III and IV stage)

Signs: The following are the findings in an established case of ovarian malignancy, in later stages.
General Examination reveals
• Cachexia and pallor of varying degree.
• Jaundice may be evident in late cases.
• Left supraclavicular lymph gland (Virchow’s) may be enlarged
• Edema leg or vulva is characteristic of malignant and not of benign neoplasm.

Per abdomen
• Liver may be enlarged, firm and nodular.
• A mass is felt in the hypogastrium; too often it may be bilateral. It has got the following
features:
− Feel — solid or heterogenous.
− Mobility — mobile or restricted.
− Tenderness — usually present.
− Surfaces — irregular.
− Margins — well-defined but the lower pole is usually not reached.
− Percussion — usually dull over the tumor; may be resonant due to overlying intestinal
adhesions.

Per vaginum
• The uterus may be separated from the mass felt per abdomen.
• Nodules may be felt through the posterior fornix. If it is more than 1 cm, the diagnosis of
malignancy is almost certain

Diagnostics

➢ Clinical.
➢ Ancillary aids.
➢ Operative findings.
➢ Histologic confirmation.

Laboratory
• Increased hormone level- estrogen, androgens
• Increase concentration of tumor markers- Elevation of serum CA 125 beyond 35 U/mL
may be suggestive
But can occur in other pathologies- Carcinomas of the breast, lung, colon and
endometrium, Endometriosis, Pelvic inflammatory disease, Peritonitis
• Detection of malignant cells from the ascetic fluid collected by abdominal paracentesis
or cul-de sac aspiration is a positive proof of abdominal malignancies

US examination- To determine size (normally small; may be only 1cm), substance inside the
tumor
Relative malignancy signs;
• Irregular, thick capsule
• Papillary structure of capsule
• Severe aortic blood flow
• Many cavities inside the cyst
• Presence of liquid in small pelvis (even can detect small amount)

Laparoscopy - cannot confirmation malignancy. It can just detect a neoplasm.


Hysteroscopy- not effective
Histological examination- Absolute method of diagnosis. This not only confirms the diagnosis but
also identifies the type and grade of malignancy.

Pre-operative examination
• Examination of GIT- fibrogastroscopy, colonoscopy, CT, MRI
• In case of estrogen producing tumor- mammography, hysteroscopy (to examine
endometrium

Treatment

Surgical Tx
A. Early stage disease (stage ia, g1 , g2 ):
a. Young woman → Unilateral oophorectomy (fertility sparing surgery) → Routine follow
up and monitoring → Completion of family → Removal of the uterus and the other ovary.
b. Elderly woman → Hysterectomy and bilateral Salpingo-oophorectomy.
c. In Stage Ia, G3 disease and others stage I diseases: Staging Laparotomy → Hysterectomy
and bilateral Salpingo-oophorectomy. Chemotherapy is considered for most patients.
B. advanced stage disease: in metastasis
Exploratory Laparotomy → Cytoreductive or debulking surgery. This includes : Total abdominal
hysterectomy bilateral salpingo-oophorectomy, complete omentectomy, retroperitoneal lymph
node sampling and resection of any metastatic tumor.

Adjuvant chemotherapy
A. In stage Ia (grade I) epithelial carcinoma → No adjuvant chemotherapy.
B. In all other stage I disease → Adjuvant chemotherapy with carboplatin and paclitaxel for six
cycles.
C. advanced stage disease.
• Chemotherapy: Chemotherapy is used widely following surgery to improve the result in
terms of survival. Drugs are given for five or six cycles at 3-4 weekly interval.
• Combination chemotherapy: Paclitaxel (175 mg/m2 ) and carboplatin (400 mg/m2 ) are
commonly used.

Radiotherapy:

Prevention
(i)Genetic screening for BRCA 1 and BRCA 2 for women with high risk for ovarian and breast
cancer.
(ii) Annual mammographic screening for women with strong family history of breast cancer.
(iii) Periodic screening for other malignancies (colonoscopy, endometrial biopsy)
(iv) Combined oral contraceptive pills as a preventive measure
(v) Prophylactic oophorectomy along with hysterectomy specially in ‘high risk’ women is a
preventive measure against ovarian malignancy
(vi) Normal pregnancy with long term breast feeding
(vii) Ab of PID and somatic diseases.

81. Malformations of female genitals. Clinical importance. Diagnostics. Correction of


malformations.
Anomalies of external genitals
Perineal or vestibular anus
Ectopic ureter- uncontrollable wetness,
correction-Partial nephrectomy and ureterectomy
Vaginal abnormality
Narrow introitus- Dyspareunia may be the first complaint, Treatment is effective
by manual stretching under general anesthesia
Hymen abnormality- imperforate hymen, chief complaints are periodic lower abdominal
pain, which may be continuous, primary amenorrhea and urinary symptoms, such as frequency,
dysuria or even retention of urine.
Abdominal examination reveals a suprapubic swelling, which may be uterine or full bladder.
Vulval inspection reveals a tense bulging membrane of bluish coloration ectal examination
reveals the bulged vagina.
Ultrasonography can make the diagnosis of hematometra and hematocolpos
incision is made in the hymen, Spontaneous escape of dark tarry colored blood is allowed
Partial agenesis- Primary amenorrhoea, hematometra, hematocolpos, cyclic lower abdominal
pain and presence of lower abdominal mass. treatment is hysterectomy.
Complete agenesis- primary amenorrhea and dyspareunia. Treatment is graduated
vaginal dilators, vaginoplasty)

Uterine anomalies
Class I: Müllerian agenesis/Hypoplasia—segmental,
Class II: Unicornuate uterus,
Class III: Didelphys uterus,
Class IV: Bicornuate uterus,
Class V: Septate uterus,
Class VI: Arcuate uterus, and
Class VII: Diethylstilbestrol (DES)-related abnormality.

Clinical feautures
nfertility and dyspareunia, Dysmenorrhea, (menorrhagia, crypto- menorrhea), Midtrimester
abortion, Cervical incompetenc, Preterm labor, IUGR, IUD
Diagnosis
hysterography, hysteroscopy ,laparoscopy , ultrasonography (vaginal probe) and magnetic
resonance imaging (MRI) reveals septate vagina and two cervices.
Treatment -
Rudimentary horn should be excised to reduce the risk of ectopic pregnancy Unification
operation (bicornuate/septate uterus)
Hysteroscopic metroplasty.

ABnorMALItIES oF tHE FALLoPIAn tubes- tubes may be unduly elongated; may have accessory
ostia or diverticula. tube may be absent on one side.
Anomalies of the ovaries- streak gonads or gonadal dysgenesis, Accessory ovary.

82) Hormone producing tumors of ovaries. Clinic, diagnostics, treatment

Masculinizing tumors-
sertoli-leydig cell tumors (androblastoma)

Feminizing tumors-
granulosa cell tumors
thecoma

sertoli-leydig cell tumors


The tumor produces predominantly androgens
(80%) and, in some cases estrogen.

Clinical symptoms
atrophy of the breasts and uterus and amenorrhea followed by masculinization. male type of
distribution of hair, hoarseness of voice, hirsutism, baldness and clitoral enlargement.

diagnostic
Serum testosterone level is elevated
clinical signs and History of patient
MRI , CT, laparascopy

treatment
surgical removal of the tumor
Unilateral oophorectomy for younger
age group is optimum. For older patients total hysterectomy
with bilateral salpingo-oophorectomy is ideal. chemotherapy (VAC or VBP) is needed for
recurrent disease

granulosa cell tumors


tumor produces estrogen. It occurs in all ages

clinical symptoms
hyperestrinism
Precocious puberty
Abnormal uterine bleeding
Postmenopausal Bleeding
acute abdomen
endometrial hyperplasia

Diagnostic
high levels of serum estrogen, inhibin
histological examination of biopsy(“Coffee bean nuclei and Call-Exner bodies in granulosa cells)
clinical signs and History of patient
MRI , CT, laparascopy

Treatment
Laparotomy and surgical staging is done .Unilateral salpingo-oophorectomy is the optimum
treatment for children or women in the reproductive age.Metastatic disease and recurrences
have been treated with chemotherapeutic regimens.

Thecomas

clinical pic
hyperestrinism
endometrial hyperplasia and often associated with endometrial carcinoma
Abnormal uterine bleeding
Postmenopausal Bleeding

Diagnostic
high levels of serum estrogen,
histological examination of biopsys (cells like that of cortical stroma with areas of granulosa
cells)
clinical signs and History of patient
MRI , CT, laparascopy

Treatment
surgical removal — total hysterectomy with bilateral salpingo-oophorectomy.

gynandroblastoma(Mixed)
This is a very rare type of tumor. It contains both granulosa
cell (estrogenic) or Sertoli-Leydig cell (androgenic) types.
Usually, it has got a benign course. Surgical removal is the
optimum treatment.
83. Gonorrhea. Methods of provocation. Treatment. Posttreatment control.
N. gonorrhoeae-gram negative diplococcic
Methods of provocation

gonorrhea is transmitted through sexual contact with the penis, vagina, mouth, or anus of an
infected partner. Ejaculation does not have to occur for gonorrhea to be transmitted or
acquired. Gonorrhea can also be spread perinatally from mother to baby during childbirth.

People who have had gonorrhea and received treatment may be reinfected if they have sexual
contact with a person infected with gonorrhea.

Gonorrhea: Diagnosis
Clinical exam
Cervical culture
Polymerase chain reaction (PCR)
or ligase chain reaction (LCR)
Gram stain– polymorphonucleocytes with gram negative intracellular diplococci
Treatment
Intramuscular Ceftriaxone
For pregnant women only:
Do not treat with Quinolones or Tetracyclines
Evaluate and treat all sexual partners

Posttreatment control-
Because re-infection is common, men and women with gonorrhea should be retested three
months after treatment of the initial infection, regardless of whether they believe that their sex
partners were successfully treated.

84.Viral infections, sexually transmitted. Modern diagnostic methods. The infections influence
on female reproductive function, their prophylaxis.
i. Herpes simplex virus- HSV:
Clinical Presentation
Primary Infection
Prodrome phase:
Tingling/itching of skin
Appearance of painful vesicles in clusters on an erythematous base
Vesicles ulcerate then crust over and heal within 7-14 days
Viral shedding continues for up to 2-3 weeks

Recurrent Disease
After primary infection,virus migrates to sacral ganglion and lies dormant
Reactivation occurs due to various triggers
Reoccurrence is usually milder and shorter in duration

The infections influence on female reproductive function


Chancroid is the next most common cause of sexually, genital ulcers

Laboratory findings.
Syphilis must first be ruled out.
Clinical diagnosis is more reliable than smears or culture because of the difficulty of isolating
this microorganism.

HSV: Diagnosis
Clinical presentation
Viral culture
Tzanck smear/Giemsa Smear

Prophylaxis-
patient education, use of barrier protection, and chronic suppressive therapy.
Acyclovir prophylaxis
ii. HIV-
The infections influence on female reproductive function

may enter menopause younger or have more severe hot flashes than women who do not have
HIV. Researchers also think the drop in the female hormone estrogen after menopause may
affect women’s CD4 counts.

• Problems related to sexually transmitted diseases (STDs)—Some STDS, like genital


herpes, pelvic inflammatory disease (PID), and chancroid can happen more often and be
more severe and/or harder to treat in women living with HIV than in HIV-negative women.
STD screening and treatment is important for the health of a woman living with HIV and
because having HIV and other STDs may increase the risk of HIV transmission to sexual
partners.

• Vaginal yeast infections—In women living with HIV, vaginal yeast infections can occur more
frequently and be harder to treat. Recurring vaginal yeast infections (those that happen at
least four times a year) can happen more often in women with advanced HIV or AIDS.

• Bacterial vaginosis (BV)—BV is a condition caused by changes in the amount of certain


types of bacteria found in the vagina. BV is more common in women living with HIV and
may be harder to treat.
• Menstrual cycle problems—may experience missed periods, lighter or heavier bleeding, or
more severe premenstrual syndrome.

• Cervical Cancer and HIV


Women with HIV have a higher risk of cervical cancer. It is important that they be screened
regularly for this disease.

Prophylaxis-

Get tested for HIV.


Choose less risky sexual behaviors.
Use condoms every time you have sex.
Limit your number of sexual partners
Get tested and treated for STDs. Insist that your partners get tested and treated, too. Having an
STD can increase your risk of getting HIV or spreading it to others.
Talk to your health care provider about pre-exposure prophylaxis (PrEP). PrEP is an HIV
prevention option for people who do not have HIV but who are at risk of getting HIV. PrEP
involves taking a specific HIV medicine every day to reduce the risk of getting HIV through sex
or injection drug use. For more information, read the HIVinfo fact sheet on Pre-Exposure
Prophylaxis (PrEP).
Do not inject drugs. But if you do, use only sterile drug injection equipment and water, and
never share your equipment with others.

Human Papiloma Virus HPV-

Diagnosis-
PAP SMEAR
DNA TEST

The infections influence on female reproductive function

it can cause health problems like genital warts and cancer.

HPV can cause cervical and other cancers including cancer of the vulva, vagina or anus. It can
also cause cancer in the back of the throat, including the base of the tongue and tonsils
(called oropharyngeal cancer).

Cancer often takes years, even decades, to develop after a person gets HPV.
Prophylaxis-

Get vaccinated. The HPV vaccine is safe and effective. It can protect against diseases (including
cancers) caused by HPV when given in the recommended age groups

Get screened for cervical cancer. Routine screening for women aged 21 to 65 years old can
prevent cervical cancer.

If you are sexually active-

Use latex condoms the right way every time you have sex. This can lower your chances of
getting HPV. But HPV can infect areas not covered by a condom – so condoms may not fully
protect against getting HPV;

Be in a mutually monogamous relationship – or have sex only with someone who only has sex
with you.

HEPATITIS B
The most important and usual serological methods to detect hepatitis viruses include; enzyme
immunoassay (EIA), radio-immunoassay (RIA) immuno-chromatographic assay (ICA), and
immuno-chemiluminescence

HBsAg (Hepatitis B surface antigen) - A "positive" or "reactive" HBsAg test result means that
the person is infected with hepatitis B. This test can detect the actual presence of the hepatitis
B virus (called the “surface antigen”

The infections influence on female reproductive function


tubal infertility.
probably increases the risk of pelvic infection in female partner through impaired immune
response to sexually transmitted infections,
with consequent tubal damage and infertility.

Prophylaxis-
The mainstay of postexposure prophylaxis (PEP) is hepatitis B vaccine, but, in certain
circumstances, hepatitis B immune globulin is recommended in addition to vaccine for added
protection.

85. Acyclic uttering bleeding


METRORRHAGIA
Definition: Metrorrhagia is defined as irregular, acyclic bleeding from the uterus. Amount of
bleeding is variable. While metrorrhagia strictly concerns uterine bleeding but in clinical
practice, the bleeding from any part of the genital tract is included under the heading. Then
again, irregular bleeding in the form of contact bleeding ) or intermenstrual bleeding in an
otherwise normal cycle is also included in metrorrhagia. In fact, it is mostly related to surface
lesion in the uterus

CAuSES Of ACyClIC blEEDING

DUb—usually during adolescence, following childbirth and abortion and preceding


menopause Submucous fibroid Uterine polyp Carcinoma cervix and endometrial
carcinoma

CAuSES Of INTERMENSTRuAl blEEDING


Apart from the causes of contact bleeding, other causes are:
• Urethral caruncle „ ovular bleeding
• breakthrough bleeding in pill use
• IUCD in utero „ Decubitus ulcer

Menometrorrhagia is the term applied when the bleeding is so irregular and excessive that the
menses (periods) cannot be identified at all.

OlIGOMENORRHEA
Definition: Menstrual bleeding occurring more than 35 days apart and which remains constant
at that frequency is called oligomenorrhea. Causes are mentioned below

COMMON CAuSES Of OlIGOMENORRHEA


„ Age-related—during adolescence and preceeding menopause
• Weight-related—obesity
• Stress and exercise related
• endocrine disorders—PCoS (commonest), hyperprolactinemia, hyperthyroidism )
• Androgen producing tumors—ovarian, adrenal
• Tubercular endometritis—late cases „ Drugs:
• Phenothiazines • Cimetidine • Methyldo

DySfuNCTIONAl uTERINE blEEDING (Dub)


Definition- DUB is defined as a state of abnormal uterine bleeding without any clinically
detectable organic, systemic, and iatrogenic cause (Pelvic pathology, e.g. tumor, inflammation
or pregnancy is excluded).
Heavy menstrual bleeding (HMB) is defined as a bleeding that interferes with woman's physical,
emotional, social and maternal quality of life.
86) Premalignant lesions of endometrium: atypical hyperplasia, adenomatous polyp.
Etiopathogenesis. Clinical features. Diagnosis.

Premalignant lesions of endometrium:


Etiology:
Endometrial hyperplasia develops in women of 40–50 years. Amongst numerous factors,
unopposed estrogen appears to be the primary factor. Pre-menopausal persistent anovulation is
almost a constant factor. In the postmenopausal women with obesity, peripheral conversion of
androgens into estrogen is a risk factor.
Long-term estrogen stimulation in condition of polycystic ovarian syndrome or feminizing ovarian
tumor may predispose to endometrial cancer

Risk Factors:
Unopposed estrogen stimulation
• Delayed menopause
• PCOS
• Nulliparity
• Previous radiation therapy
• Family history of endometrial carcinoma, carcinoma of breast, ovary or colon
• Tamoxifen therapy
• Diabetes
• Obesity
• Hypertension

Pathogenesis: Estrogen stimulation of the endometrium, without the controlling effects of a


progestin or progesterone, leads to endometrial hyperplasia and, eventually, endometrial
cancer. This abnormal growth may be due to estrogen stimulation or genetic mutations leading
to uncontrolled proliferation.

Clinical Features: There is no classic symptom of premalignant lesions. But the constant feature
is abnormal perimenopausal uterine bleeding. Patients may present: menorrhagia, metrorrhagia,
postmenopausal bleeding.
Other signs: abnormal Vaginal discharge, pap smear showing glandular abnormalities

• Abnormal uterine bleeding (most common presenting symptom):

o Postmenopausal bleeding

o Abnormal menstrual cycles in premenopausal individuals

• Vaginal discharge (clear or white) in postmenopausal women


• Abdominal or pelvic pain/pressure
• Dyspareunia (pain during intercourse)

• Urinary symptoms:

o Dysuria

o Difficulty urinating

• Unintentional weight loss

Diagnosis: ultimate diagnosis is by uterine curettage and histology.


• Accidental diagnosis is made during investigation of infertility, DUB, PCOS or excised specimen
of removed uterus.
• Diagnosis by screening procedures extended to ‘at risk’ women are not as effective like that of
CIN.
Vaginal pool smear, endometrial aspiration (pipelle endometrial sampling), endometrial biopsy,
ultrasound (check thickness of endometrium and uterine bleeding) are the different methods
available for screening.
Ultimate diagnosis is by uterine curettage and histology: showing type of hyperplasia: simple
(with or without atypia) or Complex (adenomatous with or without atypia)

Histology:
Atypical hyperplasia histology: The endometrial glands have cytologic atypia. The gland outlines
are of complex hyperplasia in type. The nuclei of the glands show enlargement, irregular size and
shape, hyperchromasia and coarse chromatin.

Adenomatous polyp: is adenomatous hyperplasia with cellular atypia (i.e., nuclear enlargement,
hyperchromasia, increased nucleocytoplasmic ratio).
• Atypical hyperplasia is further subtyped as mild, moderate, and severe. This type of hyperplasia
has a great potential for malignant progression

87. Premenopausal syndrome


Reasons of emergence
Few years prior to menopause, along with depletion ofthe ovarian follicles, the follicles become
resistant to pituitary gonadotropins. As a result, effective folliculogenesis is impaired with
diminished estradiol production.
After menopause, the stromal cells of the ovary continue to produce androgens because of
increase in LH. The main androgens are androstenedione and testosterone. Though the
secretion of androgens from postmenopausal ovary are more, their peripheral levels are
reduced due to conversion of Androgens to estrone in adipose tissue. The cumulative effect is a
decrease in estrogen
Clinics
*Vasomotor symptoms -hot flush
*Urogenital atrophy
* Osteoporosis and fracture
*Cardiovascular disease
*Cerebrovascular disease
* Psychological changes
*Skin and Hair
* Sexual dysfunction
* Dementia and cognitive decline.

Assessment of disease severity


1.Cessation of menstruation for consecutive 12
months during climacteric.
2. Appearance of menopausal symptoms ‘hot flush’
and ‘night sweats’.
3. Vaginal cytology – showing maturation index of
at least 10/85/5 (Features of low estrogen).
4. Serum estradiol : < 20 pg/mL.
5. Serum FSH and LH:>40 mlU/mL (three values at weeks interval required).
Principles of treatment

Nonhormonal Treatment
* Lifestyle modification includes: Physical activity (weight bearing), reducing high coffee intake,
smoking and excessive alcohol. There should be adequate calcium intake (300 mL of milk),
Reducing medications that causes bone loss (corticosteroids)
*Nutritious diet—balanced with calcium and protein is helpful
*Supplementary calcium—daily intake of 1–1.5 g can reduce osteoporosis and fracture
*Exercise—weight bearing exercises, walking, Jogging
*Vitamin D—supplementation of vitamin D3(1500–2000 IU/day) along with calcium can reduce
osteoporosis and fractures. Exposure to Sunlight enhances synthesis of cholecalciferol (vitamin
D3
) in the skin
* Cessation of smoking and alcohol
* Bisphosphonates prevent osteoclastic bone resorption
*Florides
*Calcitonin (50-100 IU daily) SC
*Selective estrogen receptor modulators
*Thiazides reduce urinary calcium excretion
* Carbapentine
* Clonidine
* Proxitene
* Soy protein
* Hormone replacement therapy - Estrogen—conjugated equine estrogen 0.3 mg or 0.625 mg is
given daily for woman who had hysterectomy.
•Estrogen and cyclic progestin: For a woman with intact uterus estrogen is given continuously
for 25
days and progestin is added for last 12–14 days.
•Continuous estrogen and progestin therapy
•Continued combined therapy can prevent endometrial hyperplasia. There may be irregular
bleeding.

88. Endometritis, etiology,clinics,diagnostics, treatment


Etiology- polymicrobial disease arising from ascending infection from organisms in normal
vaginal flora
Eg- ureaplasma urealyticum
- gardinella vaginalis
-group B streptococcus
Clinical-
- fever
-lower abdominal pain
-foul smell lochia
-abdormal vaginal bleeding
-dysparunia
-disuria
Diagnostic
- culture
-endometrial biopsy
-D and C
-hysterescopy

Treatment
-antibiotic therapy ( iv cefoxitine, iv doxycycline,iv cephalosporin)
-treat sexual partner for std

89. Gonorrhea of the lower part of genitals. Clinic, diagnostics, treatment.


Symptoms:
• Urinary symptoms such as dysuria (25%)
• Excessive irritant vaginal discharge (50%)
• Acute unilateral pain and swelling over the labia due to involvement of Bartholin’s gland
• There may be rectal discomfort due to associated proctitis from genital contamination y
• Others: Pharyngeal infection, intermenstrual bleeding.
Signs :
• Labia may be swollen and look inflamed
• The vaginal discharge is mucopurulent
• The external urethral meatus and the openings of the Bartholin’s ducts look congested.
On squeezing the urethra and giving pressure on the Bartholin’s glands, purulent
exudate escapes out through the openings. Bartholin’s gland may be palpably enlarged,
tender with fluctuation, suggestive of formation of abscess.
• Speculum examination reveals congested ectocervix with increased mucopurulent
cervical secretions escaping out through the external os

DIAGNOSIS :
• Nucleic acid amplication testing (NAAT) of urine or endocervical discharge is done. First
void morning urine sample (preferred) or at least one hour since the last void sample
should be tested. NAAT is very sensitive and specific (95%).
• In the acute phase, secretions from the urethra, Bartholin’s gland, and endocervix are
collected for Gram stain and culture.
• A presumptive diagnosis is made following detection of Gram-negative intracellular
diplococci on staining.
• Culture of the discharge in Thayer
• Martin medium further confirms the diagnosis. Drug sensitivity test is also to be
performed
TREATMENT
Preventive
• Adequate therapy for gonococcal infection and meticulous follow up are to be done till
the patient is declared cured.
• To treat adequately the male sexual partner simultaneously.
• To avoid multiple sex partners.
• To use condom till both the sexual partners are free from disease.

Curative:
• The specific treatment for gonorrhea is single dose regimen of any one of the following
drugs
1. Ceftriaxone — 125 mg IM
2. Ciprofloxacin — 500 mg PO
3. Ofloxacin — 400 mg PO
4. Cefixime — 400 mg PO
5. Levofloxacin — 250 mg PO
• patient with gonorrhea must be suspected of having syphilis or chlamydial infection. As
such, treatment should cover all the three.
FOLLOW UP :
• Cultures should be made 7 days after the therapy. Repeat cultures are made at monthly
intervals following menses for three months. If the reports are persistently negative,
the patient is declared cured.

90. Syndrome after total ovariotomy.


Ovarian remnant syndrome
The syndrome is defined as the persistence of the ovarian function even after an apparently
bilateral oophorectomy.
Oophorectomy becomes technically difficult during hysterectomy in cases with extensive
endometriosis or pelvic inflammatory disease.
The presenting complaints are chronic pelvic pain (cyclic), deep dyspareunia and persistence of
symptoms of endometriosis.
Pain is due to the remnant of ovarian cortical tissue, left behind (retroperitoneally) unintendedly
following a difficult oophorectomy. The pain arises from the ovaries
(preserved during hysterectomy) due to tension within the growing follicle or due to periovarian
adhesions.

Confirmation is done by serum FSH levels in premenopausal range.


Laparoscopic visualization of the remnant ovarian tissue is difficult because of adhesions.
Vaginal ultrasound and MRI are helpful to the diagnosis.
Ovarian suppression as mentioned above may be used for diagnosis.

Cure is effective by surgical removal- laparotomy


Careful dissection is needed as it is adjacent to the ureter in the retroperitoneal space.
91. Pelvic peritonitis. Clinical features. Diagnosis. Treatment.

Clinical Features: The infection is restricted mostly (80%) to childbearing period (20–40 years).
There may be past history of tubercular affection of the lungs or lymph glands. Genital
tuberculosis occurs in 10–20 per cent of patients who have pulmonary tuberculosis in
adolescence. A family history of contact may be available. Onset is mostly insidious. A flare up of
the infection may occur acutely either spontaneously or following diagnostic endometrial
curettage or hysterosalpingography.
Symptoms vary considerably with the severity and stage of the disease. At one extreme, there is
neither any symptom nor any palpable pelvic pathology.
Sometimes symptoms like weakness, low grade fever, anorexia, anemia or night sweats may be
present. The lesion is accidentally diagnosed during investigation for infertility or dysfunctional
uterine bleeding. These cases are often designated as “silent tuberculosis”.

In others, the clinical manifestations appear. These are:


Infertility: It may be primary or secondary andis present in about 70–80 per cent cases of pelvic
tuberculosis. The causes are predominantly due to tubal blockage, adhesions in the endometrial
cavity (uterine synechiae) or associated
ovulatory dysfunction.
Menstrual abnormality: In about 50 per cent, the menstrual function is normal.

The menstrual abnormalities include:


Menorrhagia or irregular bleeding is probably due to ovarian involvement, pelvic congestion or
endometrial proliferative lesion. It is the early manifestation. These patients fail to respond with
hormone therapy. Endometrial tuberculosis is a rare cause of puberty menorrhagia and
postmenopausal bleeding.
Amenorrhea or oligomenorrhea: Secondary amenorrhea is more common and may be the only
presenting symptom that makes the patient seek
medical advice. it is a late manifestation.
Others: Chronic pelvic pain is present in about 20–30 percent cases. It is often associated with
tuboovarian mass and may be precipitated by tubal patency test.
Secondary infection may aggravate the chronic pain to an acute one.
Vaginal discharge—cervical or vaginal tuberculosis may be associated with postcoital bleeding
or blood-stained discharge.
Constitutional symptoms such as loss of weight, malaise, anorexia, pyrexia and anaemia are
present in the acute phase of the disease

Signs: Health status: The general health usually remains unaffected. There may be constitutional
symptoms like weakness, low grade fever, anorexia, anemia and night sweats. There may be
evidences of active or healed extra-genital tubercular lesion.

Per abdomen: Abdominal findings may be negative or


rarely one may find an irregular tender mass in lower
abdomen arising out of the pelvis. Abdomen may feel
doughy due to matted intestines.
Tubercular ascites when encysted mimics an ovarian cyst.

Per vaginam: The pelvic findings may be negative in 50 per cent cases. Vulval or vaginal ulcer
presents with undermined edges. There may be thickening of the tubes which are felt through
the lateral fornices or nodules, felt through posterior fornix. At times, there is bilateral pelvic
mass of varying sizes quite ease.

Diagnosis:
The aims of investigations are:
To identify the primary lesion.
To confirm the genital lesion.
Blood: The leucocyte count and ESR values may be raised. Periodic examination is of value to
evaluate the progress of the lesion.
Mantoux test: Positive test with high dilution is suggestive that the patient is sensitized to
tuberculoprotein. A negative test excludes tuberculosis.
Chest X-ray is taken for evidence of healed or active
pulmonary lesion.
Diagnostic uterine curettage: This is to be done during the week preceding menstruation. The
tubercles are likely to come to the surface during this period. The material should be sent to the
laboratory in two portions.
a. One part in formol-saline for histopathological examination to detect the giant cell system.
Histology could detect tuberculosis in about 10 per cent of cases only. False-positive histology
may be due to the presence of chronic lesions (like talc or
catgut granuloma) or sarcoidosis. False-negative result is due to improper timing of uterine
curettage or due to less incidence of uterine infection.
b. One part in normal saline for:
i. Culture in Löwenstein-Jensen media.
ii. Identification of the acid-fast bacilli by Ziehl-
Neelsen’s stain (AFB-Microscopy).
iii. Nucleic acid amplification.
iv. Guineapig inoculation.
A positive culture is suggestive while a positive guineapig inoculation test is diagnostic.
Bacteriological test, if positive, should be able to type the
bacilli and report on their drug sensitivity.
Nucleic acid amplification techniques with Polymerase Chain Reaction (PCR),
can identify M. tuberculosis from endometrium or menstrual blood (clinical specimens
Rigid (lead-pipe) tubes with nodulations at places.
‘Tobacco pouch’ appearance with blocked fimbrial end
Beaded appearance of the tube with variable filling density
Distal tube obstruction.
Coiling of the tubes or calcified shadow at places.
Bilateral cornual block
Tubal diverticula and/or fluffiness of tubal outline.
Uterine cavity—irregular outline, honeycomb appearance or presence of uterine synechiae.
Imaging: Abdominal and pelvic ultrasound, CT or
MRI is helpful where a mass and/or ascites is present.
Laparoscopy: identification of tubercles in the pelvic organs or characteristic segmented
nodular appearance of the tubes. Biopsy may be taken from peritoneal tubercles for histology.
Aspiration of fluid is done for culture. This may be accidentally discovered during diagnostic
laparoscopy for infertility work up or for chronic pelvic pain.

Treatment:
general: In the presence of active pulmonary tuberculosis, hospital admission is preferred.
Otherwise, pelvic tuberculosis per se need not require hospitalization or bed rest except in
acute exacerbation. To improve the body resistance, due attention is to be paid as regards diet
and to correct anemia. Until the infection is controlled, the husband should use condom during
intercourse to prevent possibility of contracting urogenital tuberculosis.

chemotherapy: Antitubercular chemotherapy is the treatment of first choice.


Drugs:
Isoniazid: 15 mg/kg, 3 times a week for 6 months.
Rifampicin: 600 mg, 3 times a week for 6 months.
Pyrazinamide: 30 mg/kg, 3 times a week for first 2 months only.
Ethambutol: 30 mg/kg, 3 times a week for first 2 months only

Initial phase: Four drugs are used for 2 months to reduce the bacterial population and to
prevent emergence of drug-resistance.
Continuation phase: Treatment is continued for a period of further 4 months with isoniazid and
rifampicin. This standard regimen may be used during pregnancy and lactation. After about a
year of treatment, diagnostic endometrial curettage is to be done. If the histological and/or
bacteriological examination becomes positive, the treatment must be continued further. If
these are negative, the endometrium is examined at interval of 6 months

Surgical:
The ideal surgery should be total hysterectomy with bilateral salpingo-oophorectomy.
In young women at least one ovary, if found apparently healthy, should be preserved.
Isolated excision of tubo-ovarian mass, drainage of pyometra or repair of fistula may be done in
selected cases.

92. Benign diseases of uterus neck. Modern methods of diagnostics.


1.Cervical ectopy (erosion) - Cervical ectopy is a condition where the squamous epithelium of
the ectocervix is replaced by columnar epithelium, which is continuous with the endocervix.It is
not an ulcer.
- Can be congenital or acquired.
- Congenital - At birth, in about one-third of cases, the columnar epithelium of the endocervix
extends beyond the external os. This condition persists only for a few days until the level of
estrogen derived from the mother falls. Thus, the congenital ectopy heals spontaneously.
- Acquired

1. Hormonal: The squamocolumnar junction is not static and its movement, either inwards or
outwards is dependent on estrogen. When the estrogen level is high, it moves out so that the
columnar epithelium extends onto the vaginal portion of the cervix replacing the squamous
epithelium. This state is observed during pregnancy and amongst ‘pill users’. The
squamocolumnar junction returns back to its normal position after 3 months following
delivery and little earlier following withdrawal of ‘pill’.
2. Infection: The role of infection as the primary cause of ectopy has been discarded. However,
chronic cervicitis may be associated or else the infection may supervene on an ectopy
because of the delicate columnar epithelium which is more vulnerable to trauma and
infection.
Clinical features - Symptoms: The lesion may be asymptomatic.However, the following
symptoms may be present.Vaginal discharge—The discharge may be
excessively mucoid. It may be mucopurulent, offensive and irritant in presence of infection; may
be even blood-stained due to premenstrual congestion.
Contact bleeding especially during pregnancy and ‘pill use’ either following coitus or defecation
may be associated.
Associated cervicitis may produce backache,pelvic pain and at times, infertility.
Signs: Internal examination reveals,
Per speculum—There is a bright red area surrounding and extending beyond the external os in
the ectocervix. The outer edge is clearly demarcated. The lesion may be smooth or having small
papillary folds. It is neither tender nor bleeds
to touch. On rubbing with a gauze piece, there may be multiple oozing spots (sharp bleeding in
isolated spots in carcinoma).
The feel is soft and granular giving rise to a grating sensation.
2.CERVICAL CYSTS
Nabothian Cysts : These are usually multiple. They are formed due to blocking of the cervical
gland mouths usually as a result of healing of ectopy (epidermidization). The pent up secretion
produces cysts of varying sizes from microscopic to pea. The presence of the cysts furthest from
the external os indicates the extent of transformation zone. The lining epithelium is columnar.
The treatment is directed towards chronic cervicitis.
Endometriotic Cysts: These are situated in the portio vaginalis part of the cervix. The cyst is small
and reddish and of less than 1 cm in diameter.
It is more explained by the implantation theory.The implantation of the endometrium occurs
during delivery or surgery. The lining epithelium shows endometrial glands and stroma.
Symptoms include intermenstrual or postcoital bleeding, deep dyspareunia and dysmenorrhea.
Speculum examination reveals a small reddish cyst.
The treatment is destruction by cauterization and rarely by excision.
Mesonephric Cysts: These are usually situated on the outerside of the cervical stroma. They
seldom exceed 2.5 cm. These are lined by cuboidal
epithelium. They are asymptomatic. The existence of the cyst is discovered on speculum
examination and confirmation by excision biopsy.
3. EVERSION (ECTROPION)
In chronic cervicitis, there is marked thickening of the cervical mucosa with underlying tissue
edema. These thickened tissues tend to push out through the external os along the direction of
least resistance. The entity is most marked where the cervix has already been lacerated. In such
conditions, the longitudinal muscle
fibers are free to act unopposed. As a result, the lips of the cervix curl upwards and outwards to
expose the red looking endocervix so as to be confused with
ectopy.
4.CERVICAL TEAR
Varying degrees of cervical tear is invariable during vaginal delivery. One or both the sides may
be torn or the tear may be irregular (stellate type). If there is no superimposed infection and the
tear is small, the torn surfaces may appose leaving behind only a small notch. However, if
infection supervenes, eversion occurs confusing the diagnosis of ectopy.
Non-obstetric causes of cervical lacerations are during operative procedures of dilatation of the
cervix. Postmenopausal atrophy or chronic cervicitis predisposes to tear.
5.ELONGATION OF THE CERVIX
The normal length of the cervix is about 2.5 cm. The vaginal and the supravaginal parts are of
equal length.
The elongation may affect either part of the cervix.
Causes: Elongation of the supravaginal part is commonly associated with the uterine prolapse.
Vaginal part is always elongated congenitally. Chronic cervicitis may produce some hypertrophy
and makes the cervix bulky.
Symptoms
There is no specific symptom for supravaginal elongation. However, congenital elongation of the
vaginal part may present the following:
+Sensation of something coming down
+Dyspareunia
+Infertility
PELVIC EXAMINATION: Supravaginal elongation is
featured by:
Associated uterine prolapse.
Fornix—shallow.
Vaginal cervix—normal length.
Uterine body—normal in size.
Uterocervical canal—increased in length evidenced by introduction of an uterine sound.This
indirectly proves that the increase is in the
supravaginal part.
Congenital elongation is featured by :
Fornix—deep.
Vaginal cervix—elongated.
Uterine body—normal in size.
Uterocervical canal—increased in length,evidenced by uterine sound.
Treatment:
Supravaginal elongation
As it is associated with uterine prolapse, its treatment protocol will be the same as that for
prolapse.
Congenital elongation
The excess length of the cervix is amputated (cervical amputation). In presence of congenital
prolapse, some form of cervicopexy has to be done.
Modern methods of diagnosis of benign conditions of cervix
1. Screening tests - All sexually active women should be screened starting from the age of 21
years or after 3 years of vaginal sex with no upper age limits.
Screening should be yearly till the age of 30.Thereafter, it should be done at an interval of every
2–3 years after three consecutive yearly negative smears (ACOG 2009). The high risk group
should be screened with HPV DNA testing combined with cytology . The negative predictive value
of one negative HPV DNA test and two negative cytology tests are almost 100%. When both the
tests are negative, the screening interval may be increased to 6 years.
- Conventional cytology/PAP smear - Pap testing after total hysterectomy, done for benign
lesion is not recommended.
- Liquid based cytology
- (also can perform visual inspection with acetic acid , visual inspection with Lugol’s iodine –
iodine react with glycogen in normal smooth muscle cells, so cervix will become
brown.Pathological cells have small amount/haven’t glycogen, so no colouration.Non
colouration can be also occurred in chronic conditions in vagina,post menapaused,
hypoestrogenemia)
- HPV/DNA tests
2. EXAMINATION OF CERVICAL MUCUS
• Bacteriological study: Cusco’s bivalve speculum is introduced without lubricant. With the
help of a sterile cotton swab, the cervical canal is swabbed. The material is either sent for
culture or spread over a microscopic slide for Gram staining.
• Hormonal status: The physical, chemical and cellular components of the cervical
secretion are dependent on hormones—estrogen and progesterone. estrogen increases
the water and electrolyte content with decrease in protein. As such, the mucus becomes
copious, clear and thin. Progesterone, on the other hand, decreases the water and
electrolytes but increases the protein. As a result, the mucus becomes scanty, thick and
tenacious.
3. Colposcopy and biopsy
Procedure: The patient is placed in lithotomy position. The cervix is visualized using a Cusco’s
speculum . Colposcopic examination of the cervix and vagina is done using low power
magnification (6-16 fold). Cervix is then cleared of any mucus discharge using a swab soaked with
normal saline. Green filter and high magnification can be used now. Next, the cervix is wiped
gently with 3 percent acetic acid and examination repeated.
Acetic acid causes coagulation of nuclear protein which is high in CIN. This prevents transmission
of light through the epithelium, which is visible as white (acetowhite) areas.
**Abnormal cytology is an indication of colposcopic evaluation and directed biopsy. If colposcopy
is not available, biopsy is to be taken from the unstained areas following application of Schiller’s
or Lugol’s iodine. In the presence of infection, repeat cytology has to be done after the infection
is controlled.

93. Medical abortions. Methods of abortion performance and their assessment.


• Oral mifepristone (Mifeprex) and oral misoprostol (Cytotec). This is the most common
type of medical abortion. These medications are usually taken within seven weeks of the
first day of your last period.
• Oral mifepristone and vaginal, buccal or sublingual misoprostol. This type of medical
abortion uses the same medications as the previous method, but with a slowly dissolving
misoprostol tablet placed in your vagina (vaginal route), in your mouth between your teeth
and cheek (buccal route), or under your tongue (sublingual route).

• Methotrexate and vaginal misoprostol. Methotrexate (Otrexup, Rasuvo, others) is rarely


used for elective, unwanted pregnancies, although it's still used for pregnancies outside of
the uterus (ectopic pregnancies). This type of medical abortion must be done within seven
weeks of the first day of your last period, and it can take up to a month for methotrexate to
complete the abortion. Methotrexate is given as a shot or vaginally and the misoprostol is
later used at home.
• Vaginal misoprostol alone. Vaginal misoprostol alone can be effective when used before
nine weeks of gestation of the embryo. But vaginal misoprostol alone is less effective than
other types of medical abortion.

Medical abortion refers to the administration of medications to end a pregnancy.

94. Premenstrual syndrome. Еtiopathogenesis. Classification. Clinical features. Diagnosis.


Treatment.
• Premenstrual syndrome (PMS) is a psychoneuroendocrine disorder of unknown
etiology, often noticed just prior to menstruation.
• There is cyclic appearance of a large number of symptoms during the last 7–10 days of
the menstrual cycle.
It should fulfil the following criteria (ACOG) :
1. Not related to any organic lesion.
2. Regularly occurs during the luteal phase of each ovulatory menstrual cycle.
3. Symptoms must be severe enough to disturb the life style of the woman or she requires
medical help.
4. Symptom-free period during rest of the cycle.
When these symptoms disrupt daily functioning they are grouped under the name
premenstrual dysphoric disorder (PMDD).

Clinical features:
• PMS is more common in women aged 30–45.
• It may be related to childbirth or a disturbing life event.
• There are no abnormal pelvic findings excepting features of pelvic congestion.
TREATMENT
Life style modification and congnitive behavior therapy are important steps.
GENERAL
Nonpharmacological:
(a) Assurance, Yoga, Stress management, Diet manipulation.
(b) Avoidance of salt, caffeine and alcohol specially in second half of cycle improves the
symptoms.
Nonhormonal :
(a) Tranquilizers or antidepressant drugs, may be of help logically.
(b) Pyridoxine – 100 mg twice daily is helpful by correcting tryptophan metabolism specially
following ‘pill’ associated depression.
(c) Diuretics in the second half of the cycle – Frusemide 20 mg daily for consecutive 5 days a week
reduces fluid retention.
(d) Anxiolytic agents are found to be helpful to women having persistent anxiety. Alprazolam 0.25
mg, BID) is given during the luteal phase of the cycle.
(e) Selective Serotonin Reuptake Inhibitors (SSRI) and Noradrenaline Reuptake Inhibitors (SNRI)
are found to be very effective.
Other drugs used are: Sertaline (50 mg/day) and Venlafaxine.
Hormones:
Any one of the following drugs is to be prescribed:
Oral contraceptive pills: The idea is to suppress ovulation and to maintain an uniform hormonal
milieu. The therapy is to be continued for 3–6 cycles. Newer OCPs contain progestin
drospirenone. It has antimineralocorticoid and antiandrogenic properties. Drospirenone
containing OCPs are found to have better control of symptoms.
Progesterone is not effective in treating PMS. Levonorgestrel intrauterine system (IUS) had been
used to suppress ovarian cycle.
Spironolactone: It is a potassium sparing diuretic. It has anti-mineralocorticoid and anti-
androgenic effects. It is given in the luteal phase (25–200 mg/day). It improves the symptoms of
PMDD.
Bromocriptine: 2.5 mg daily or twice daily may be helpful, at least to relieve the breast
complaints.
Suppression of ovarian cycle:
Suppression of the endogenous ovarian cycle can be achieved by:
Danazol 200 mg daily is to be adjusted so as to produce amenorrhea. Barrier method of
contraception should be advised during the treatment.
GnRH analogues — The gonadal steroids are suppressed by administration of GnRH agonist for 6
months (medical oophorectomy). GnRH analogues in PMS are used: (i) To assess the role of
ovarian steroids in the aetiology of PMS. (ii) This can also predict whether bilateral oophorectomy
would be of any help or not. The preparations and doses used are as given
– Goserelin (Zoladex): 3.6 mg is given subcutaneously at every 4 weeks.
– Leuprorelin acetate (Prostap):3.75 mg is given by SC or IM at every 4 weeks.
– Triptorelin (Decapeptyl) : 3 mg is given IM every 4 weeks.
Oophorectomy
In established cases of primary PMS with recurrence of symptoms and approaching to
menopause, hysterectomy with bilateral oophorectomy is a last resort.

95.Endometrial contraception. Indications, contraindications, complications.


• Intrauterine contraceptive devices (IUCD)
Hormone containing IUD either releasing progesterone (progestasert) or levonorgestrel (LNG-
IUS – Mirena)
Non hormonal – eg;Copper T 380A, Cu T 200
-The introduction of IUD is an OPD procedure without anesthesia taking full aseptic precautions.
‘No touch’insertion technique is preferred.
-The replacement time for CuT 200 B is 4 years, multiload 250—3 years, CuT 380A—10 years,
multiload 375—5 years and LNG–IUS is 5 years.
Indications for IUD
• Parous female at risk of pregnancy and for spacing or limiting pregnancy
• LNG – IUCD As a treatment for gynecological conditions –eg: endometrial
hyperplasia,Adenomyosis.Also suitable for women with menorrhagia and mild
dysmenorrhea.
• Women who desire long term reversible contraception
• Women with the following medical conditions for which an IUD may be optimal method:
• Diabetes
• Breast cancers – copper IUD (LNG IUCD contraindicated for current breast cancers)
• VTE – LNG – IUCD
Contraindications for Insertion of IUCD:
(1) Presence o\f pelvic infection current or within 3 months
(2) Undiagnosed genital tract bleeding
(3) Suspected pregnancy
(4) Distortion of the shape of the uterine cavity as in fibroid or congenital
uterine-malformation
(5) Severe dysmenorrhea
(6) Past history of ectopic pregnancy
(7) Within 6 weeks following cesarean section
(8) STIs —Current or within 3 months
(9) Trophoblastic disease
(10) Significant immunosuppression.
Additionally for CuTare: (11) Wilson disease and (12) Copper
allergy.
For LNG-IUS are: (13) Hepatic tumor or hepatocellular disease (active)(14) Current breast cancer
and (15) Severe arterial disease.
Complications of IUD
• Immediate:
-Cramp like pain — It is transient but at times, severe and usually lasts for 1/2
to 1 hour. It is relieved by analgesic or antispasmodic drugs.
-Syncopal attack—Pain and syncopal attack are more often found in nulliparous or when the
device is large enough to distend the uterine cavity.
-Partial or complete perforation — It is due to faulty technique of insertion but liable to be met
with in lactational period when the uterus remains small and soft.
• Remote:
-Pain — The pain is more or less proportionate to the degree of myometrial distension. A
proper size of the device may minimize the pain.
- Abnormal menstrual bleeding—The excessive bleeding involves increased menstrual blood
loss, prolongation of duration of period and intermenstrual bleeding. The patient may
become anemic and is of concern in one who is already
anemic. Iron supplement is advocated. Tranexamic acid may be given for short-term relief.
Menstrual loss is much less with the use of third generation IUDs.
- Pelvic infection (PID)—The risk of developing PID is 2–10 times greater amongst IUD users.
The risk is more in the first 3 weeks. Infection with chlamydia and rarely with actinomyces
are seen.Newer IUDs reduce the risk.
- Perforation of the uterus—The incidence of uterine perforation is about 1 in 1000
insertions.Most perforations occur at the time of insertion but the migration may also occur
following initial partial perforation with subsequent myometrial contraction.It is however
less common when the device is introduced by the withdrawal technique.

96. Ultrasonic test for diagnostics of gynecologic diseases.

Gynecologic ultrasonography refers to the application of medical ultrasonography to the


female pelvic organs (specifically the uterus, the ovaries, and the fallopian tubes) as well as the
bladder, the adnexa, and the recto-uterine pouch. The procedure may lead to other medically
relevant findings in the pelvis.
Gynecologic sonography is used extensively:

to assess pelvic organs,


to diagnose acute appendicitis
to diagnose and manage gynecologic problems including endometriosis, leiomyoma,
adenomyosis, ovarian cysts and lesions,
to identify adnexal masses, including ectopic pregnancy,
to diagnose gynecologic cancer
in infertility treatments to track the response of ovarian follicles to fertility medication (i.e.
Pergonal). However, it often underestimates the true ovarian volume.

Through transvaginal sonography ovarian cysts can be aspirated. This technique is also used in
transvaginal oocyte retrieval to obtain human eggs (oocytes) through sonographic directed
transvaginal puncture of ovarian follicles in IVF.

Gynecologic ultrasonography is sometimes overused when it is used to screen for ovarian


cancer in women who are not at risk for this cancer.There is consensus that women with only
average risk for ovarian cancer should not be screened with this procedure for cancer
Sonohysterography is a specialized procedure by which fluid, usually sterile saline (then called
saline infusion sonography or SIS), is instilled into the uterine cavity, and gynecologic
sonography performed at the same time. A review in 2015 came to the conclusion that SIS is
highly sensitive in the detection of intrauterine abnormalities in subfertile women, comparable
to hysteroscopy. SIS is highly sensitive and specific test in the diagnosis of uterine polyps,
submucous uterine fibroids, uterine anomalies and intrauterine adhesions (as part of
Asherman's syndrome), and can be used as a screening tool for subfertile women prior to IVF
treatment.

97.Cysts of ovaries. Classification, clinic, diagnostics, options of treatment.


CLASSIFICATION
Functional cyst – growing during menstrual cycle due to hormonal imbalance. These cyst must
not be operated. They resolve spontaneously or after drugg treatment.
Organic cyst – they can complicate or degenerate.Should be removed and analysed.There are
benign cysts and cysts at the limit of the malignancy called borderline.
Ovarian cysts also can be classified as,
1.Benign ovarian cysts
- Functional – follicular cyst, corpus luteal cyst, theca luteal cyst
- Inflammatory – tubo ovarian abscess,endometrioma
- Germ cell – benign teratoma (dermoid cyst)
- Epithelial – serous cystadenoma,mucinous cystadenoma,brenner tumour
- Sex cord stromal – fibroma , thecoma
2.Malignant ovarian cyst
- Epithelial – serous cystadenocarcinoma
- Germ cell – choriocarcinoma
- Sex cord stromal – gynandroblastroma
Clinical features
- Ovarian cysts are usually asymptomatic, they often are incidental findings during
ultrasound performed for other reasons.
- Lower abdominal pain – pain can be sharp,intermittent, sudden onset,severe
- A sudden onset severe abdominal pain may be suggest a cyst rupture with
intraperitoneal bleeding..
- Non specific symptoms – nausea, vomiting
- Vaginal spotting may occur secondary to decreased estrogen level and to hormonal
imbalance.
- Premenarchal and postmenapausal females usually haven’t functional cysts – both have
increased incidences of malignancies.
- Torsion is more common in women younger than 30 years – strenuous activities,such as
exercises or sexual intercourse may precede rupture of cyst.
- Inflammatory ovarian cysts are usually associated with PID
Physical examination
Vital signs are usually within reference range.
⚪ A low-grade fever may be observed.

⚪ Tachycardia may be observed because of pain or hypovolemia in case of bleeding (occasionally


orthostatic).
⚪ Patients with cysts complicated with inflammation, necrosis, and bacterial infection or those
with hemorrhagic complications may present in florid septic or hypovolemic shock.
⚪ Abdominal tenderness usually is elicited in the lower quadrants.

⚪ Tenderness ranges from the usual mild-to-moderate tenderness (mainly with cystic rupture)
to overt peritonitis (from cystic content rupture or intraperitoneal hemorrhagic, infectious, or
purulent processes).
A pelvic mass may be palpated.
⚪ Occasionally, the initial presentation is one of septicemia, peritonitis, and/or shock.

⚪ Cervical motion tenderness may be elicited with an ovarian cyst.

⚪ Adnexal tenderness is usually one-sided, but it may be bilateral.

⚪ Adnexal masses may be localized and tender.

⚪ Any adnexal masses in the postmenopausal period are rare and suspicious. A thorough search
to exclude a malignancy or a benign
tumor is mandated.
⚪ A rectal examination may reveal localized pain or aid in the palpation of a mass lesion.
Laboratory and instrumental diagnosis
- FBC/LFT/RFT
- USS/TVS/TAS
- Tumor markers – Ca 125,Ca 19.9,,CEA
- Pregnancy test
- Inflammatory markers – CRP,ESR, WBC
- CXR
- CT
- MRI
Treatment
These cyst must not be operated. They resolve spontaneously or after drugg treatment.
The goal of therapy is to treat the complications resulting from torsion, infection,hemorrhage,
and rupture.
- COCP - Used to help down regulate hypothalamic-pituitary stimulation to the ovaries,
which decreases the stimulus for the formation of cysts.
- Ethinyl estradiol and progesterone reduce the secretion of LH and FSH from the pituitary
by decreasing amount of gonadotropin-releasing hormones.
- Progesterone – causes the secretory transformation of endometrium and reduce the
secretion of LH and FSH.
- Analgesics/sedatives - Pain relief is always a paramount concern, but it must be
remedied with agents chosen for the given clinical situation.
- Non steroid antyinflammatory drugs usually are used
- Fentanyl (Duragesic) -- Potent narcotic analgesic with much shorter half-life than
morphine sulfate. DOC for conscious sedation analgesia. Excellent choice for pain
management and sedation with short duration (30-60 min) and easy to titrate. Easily and
quickly reversed by naloxone.
- Hydrocodone bitartrate and acetaminophen (Vicodin) -Drug combination indicated for
moderate to severe pain.Should be given only on discharge to a patient with abdominal
pain with a known cause.
- anti-inflammatory agents - For relief of mild to moderate pain.Inhibits inflammatory
reactions and pain by decreasing activity of cyclo-oxygenase(COX), which results in a
decrease of prostaglandin synthesis.
- Ketorolac (Toradol) – Inhibits prostaglandin synthesis by decreasing the activity of the
COX, which results in decreased formation of prostaglandin precursors.
Surgical treatment – organic cysts must operate
Definitive surgery
In young patients
-Ovarian cystectomy leaving behind the healthy ovarian tissue is the operation of choice.
-Ovariotomy (salpingo-oophorectomy) is reserved for a big tumor that has destroyed almost all
the ovarian tissues or for a gangrenous cyst.
-If both the ovaries are involved, ovarian cystectomy should be done at least in one ovary.
-Retention of the uterus for possible ART may be considered when bilateral ovariotomy has to
be done.
- In parous women around 40 years
- Total hysterectomy with bilateral salpingo-oophorectomy is to be done.
In between these two extremes of age
Individualization is to be done as regards the nature of surgery. Due consideration is to be given
about the reproductive and menstrual function.
In all cases, the entire tumor is to be sent for histological examination. If a part is to be sent, a
small piece from the comparatively solid or thick capsule is to be selected.

98. Indications for planned and emergency expeditious treatment of a hysteromyoma.


Types of operations.
Indications for myomectomy- planned
Persistent uterine bleeding despite medical therapy.
excessive pain or pressure symptoms.
size >2 weeks, woman desirous to have a baby.
Unexplained infertility with distortion of the uterine cavity.
recurrent pregnancy wastage due to fibroid.
rapidly growing myoma during follow-up.
™subserous pedunculated fibroid

inDIcatIOns OF emeRgency suRgeRy In a FIBROID


torsion of a subserous pedunculated fibroid.
Massive intraperitoneal hemorrhage following rupture of veins over subserous fibroid.
Uncontrolled infected fibroid.
Uncontrolled bleeding fibroid

Types of operations

Myomectomy
(may be done by)
Laparotomy
Laparoscopy
Hysteroscopy

Embolotherapy
■ Myolysis
■ Hysterectomy

99. Cervical intraepithelial dysplasia (CIN). Etiology. Role of viral infection in the
development of CIN.
etiology
(i) Infection with high-risk HPV,
(ii) Multiple types of HPV
(iii) Persistence of infection,
(iv)Age > 30 years,
(v) Smoking, and
(vi) Compromised host immunodefense

Role of viral infection


hPV Infection→ (ImmunologicFactors ) →Persistent HPV Infection→ Cellular
Dysregulation → High-Grade CIN → Invasive Cancer→ Co- carcinogens→ cellular dysregulation
low grade squamous intraepithelial lesions include CIN 1 and changes of HPV

Two mechanisms are involved in the process of replacement of endocer-vical columnar


epithelium by squamous epithelium.
♦ By squamous metaplasia of the subcolumnar

reserve cells (Fig. 22.2).


♦ Squamous epidermidization by ingrowth of the squamous epithelium of the ectocervix under
the columnar epithelium.

Initially, the squamous cells are immature but ultimately become mature and indistinguishable
to the adjacent squamous epithelium.This metaplastic process is very active at the time of
menarche and during and after first pregnancy.

These periods are of high estrogenic phase which lowers the vaginal pH. The acid pH probably is
an important trigger for the metaplastic process. This metaplastic cells have got the potentiality
to undergo atypical transformation by trauma or infection.The prolonged effect of carcinogens
can produce continuous changes in the immature cells which may lead to malignancy.

100. Refer Q113 (same)

101. Uterine bleedings at apremenopauzal stage. Specifics of diagnostics and treatment.


The uterine bleeding at the premenopausal stage is also known as anovulatory uterine
bleeding.
It is associated with the anovulation caused by impaired or unestablished functional
relationships in the hypothalamic – pituitary – ovarian axis.
The other main causes are,
• impaired secretion of hormones as a result of diseases and psychic traumas
• hypovitaminoses
• intoxications
• impaired function of the thyroid, adrenals, and other endocrine glands.
In the premenopausal women the age-related changes take place in the hypothalamic centers,
which regulate the secretion of gonadotropic hormones by the pituitary gland; there are
alterations in the cyclic character of secretion of these hormones and the corresponding
ovarian hormones. Primary changes in the ovaries and uterus can also provoke the disturbances
of the neurohormonal interrelationships.
Diagnostics
general and gynecologic examination
• bleedings that follow the suppression of menses
• monophase basal body temperature
• high or low karyopyknotic index.
• Histological study of the endometrial scrape (preferably on the first days of bleedings)
reveals different types of hyperplasia, which may be coupled with endometritis
• Anovulatory uterine bleedings should be differentiated from many disease forms that
are attended with bleedings (spontaneous abortions, interrupted fallopian pregnancy, tumors
of the uterus, hormone- producing tumors of the ovaries, etc.)
Treatment
• Followed by the arresting of bleeding, prevention of its recurrence and regulation of
ovarian function.
• Stages of treatment depend on the patient’s age and her general condition. Sometimes
uterine bleedings can be so abundant that emergency care is required, and the
physician should take energetic steps to arrest the bleeding.
The doctor’s tactics largely depends on the patient’s age:
1. at juvenile age (puberty) medical treatment is appropriative, while surgical (D&C) used
as a last resort, or in severe case (Hb< 90g/L;
2. at reproductive and premenopausal agers D&C with histology – mandatory ( to exclude
malignancy).
Juvenile bleeding
Iron therapy
• iron deficiency should be assessed and treated.
• The target dose is in the range of 150 mg/day to 200 mg/day of elemental iron in 1 to 3
divided doses/day.
• To aid in absorption, iron supplements should be taken on an empty stomach with a
glass of orange juice.
• The duration of replacement should extend at least 3 months beyond normalization of
hemoglobin to allow for replenishment of iron stores
Hormonal therapy
• Because the underlying problem is bleeding from a hyperproliferative endometrium that
has outgrown its estrogen supply, the primary therapeutic goal is reestablishment of estrogen
supply in the form of high-dose oral or parenteral estrogen
• This is achieved through combine oral contraceptive pills
• Monophasic pills containing at least 30—35 μg of ethinyl estradiol should be used
initially and can be given once daily in the adolescent who is not actively bleeding
• Those who bleed quickly (changing pads or swabs every few hours) may require 4 times
a day until the bleeding stops (usually within 2-3 days).
• Then one tablet three times a day for 3 days, then one tablet twice a day for 2 days,
then one tablet once a day for 21 days, then 7 days without taking the tablets. with these 7
days a new menstruation will begin, and from the 1st day of the new flow a new set of the
same COCP tablets should begin with the usual scheme: 1 tablet once a day on the mouth
during sleep for 21 days.
• Consumption of COCP should be administered within 3-6 months.
• In cases of ineffective conservative treatment, when there is a threat to the life of the
patient, the surgical curettage should be used for arresting of bleeding.
At reproductive and menopausal age
• Treatment of bleedings in the child-bearing age and, particularly, the premenopausal
bleedings should be started with curettage of the uterine mucosa for arresting of bleeding and
histological examination of the endometrial scrape, which is followed by the arrest of bleeding.
• The day, when D&C was done would register as the 1st day of the cycle and COCP
should be administered by scheme 1 tablet per mouth once a day at bed time within 21 days,
etc
• As to conservative treatment, hormonal drugs are prescribed to arrest bleeding
(hormonal haemostasis) and for the regulation of the menstrual cycle.
• The COCP should be administered for 6 months
• In case of acute, profuse bleeding, 3 steps of treatment are offered.
1. To stop bleeding: Premarin IV 25 mg every 4 hours until bleeding stops.
2. Exogenous progesterone to create secretory changes in the endometrium and prepare for
shading off: Medroxyprogesterone acetate 10 mg of orally once a day for 10 days, followed by
withdrawal bleeding 3-5 days after completion of the drug.
3. COCP on schedule, 3-6 months
• Coagulants, hemostatic agents, stimulants of uterine- contractility (Ergot-dugs) should
be used as additional drugs

102. Extrauterine pregnancy interrupted by a rupture of the fallopian tube.


• Tubal rupture is predominantly common in isthmic and interstitial implantation. As the
isthmic portion is narrow and the wall is less distensible, the wall may be easily eroded by the
chorionic villi.
• Isthmic rupture usually occurs at 6–8 weeks, the ampullary one at 8–12 weeks and the
interstitial one at about 4 months.
• Depending upon the site of rupture, it is known as:
(1) Intraperitoneal rupture: This type of rupture is common. The rent is situated on the roof or
sides of the tube. The bleeding is intraperitoneal.
(2) Extra-peritoneal rupture (intraligamentary): This is rare and occurs when the rent lies on the
floor of the tube where the broad ligament is attached. It is commonly met in isthmic
implantation.
Symptoms in patients with a ruptured uterine tube are distinguished by ,
I. severe internal bleeding
II. shock
III. acute anemia.
• The history is characterized by the same factors of risk of ectopic pregnancy and delay
of menstruation.
• The clinical picture of tubal pregnancy, which terminates by the type of rupture, is
characterized by a sudden attack of a sharp pain in the lower abdomen or in the iliac area with
irradiation into the rectum, hip, shoulder-blade.
• The patients soon become adynamic and find it difficult to answer the questions.
• Any signs of internal bleeding appear, for example, severe paleness, cyanosis of the lips,
and cold sweat. The pulse is frequent, blood pressure may be low.
• The abdomen is drastically bloated, the patient spares it during breathing. Percussion
and palpation of the abdomen are tender. Percussion elicits dull sounds in sloping places.
• Palpation elicits pronounced tenderness of the abdomen, the Shchotkin - Blumberg
symptom is positive.
• Gynecological examination reveals painful cervical displacements, the overhanging of
the posterior and lateral fornices.
• Palpation of the posterior fornix is very painful ("Douglas's cry"). The uterine body is
usually enlarged, softened and tender.
• To one side of the adnexal area one can palpate dough-like mass with a vague outline,
occasionally pulsation may be felt.
Treatment
Tubal pregnancy is treated surgically by removing the pregnant tube.
The patient is operated on immediately after the diagnosis has been established, even if the
condition of the patient is only satisfactory, because the hemorrhage into the abdominal cavity
may recur at any moment
If the clinical picture is grave, the patient is operated on urgently, with transfusion of blood;
cardiac drugs and other means to control anemia and shock are used.
In general the ectopic pregnancy must be removed by one of the following techniques:
➢ Salpingectomy. If the tube is badly damaged, the correct treatment is removal of the
affected tube. If implantation has occurred in the interstitial portion of the tube, then it
may be necessary to resect part of the uterine horn in addition to removing the tube.
➢ Salpingotomy. If the ectopic pregnancy is contained within the tube, it may be possible
to conserve the tube by removing ofthe pregnancy and reconstitution of the tube. This
is particularly important when the contralateral tube has been lost. However, there is an
increased risk of recurrent ectopic pregnancy in the damaged tube.
➢ Tubal compression. Occasionally when the tubal pregnancy occurs in the fimbrial end of
the tube it may be possible to squeeze out the ovum without opening the tube, but it is
essential to be certain that adequate hemostasis is obtained.

103. Genital prolapse: causes, types and degrees of prolapse, preventive, conservative and
surgical methods of treatment.
Pelvic organ prolapse happens when the muscles and tissues supporting the pelvic organs (the
uterus, bladder, or rectum) become weak or loose. This allows one or more of the pelvic organs
to drop or press into or out of the vagina.
Causes
• Overweight or obesity
• Repeated heavy lifting
• Aging. Pelvic floor disorders are more common in older women. About 37% of women
with pelvic floor disorders are 60 to 79 years of age, and about half are 80 or older
• Hormonal changes during menopause. Loss of the female hormone estrogen during and
after menopause can raise your risk for pelvic organ prolapse. Researchers are not sure
exactly why this happens.
Congenital factors
• Bladder exstrophy
• Nulliparous
• Collagen defects ( type IV Ehlers- Danlos syndrome , marfan syndrome )
• Race ( white people have more risk )
• Anatomically – congenital short vagina
Childbirth factors
• Successive vaginal deliveries
• Straining during 1st stage of labor
• Forceps before full cervical dilatation
• Prolonged 2nd stage of labor
• Trauma
Types
• Uterine -When the pelvic muscles and ligaments stretch beyond the ability to
support the uterus, a uterine prolapse is likely. This causes the uterus to descend out
of the vagina.
• Vaginal prolapse -This is a condition where the upper walls of the vagina lose their
normal shape, resulting in a collapse into the vaginal canal or lower.
• Bladder prolapse (anterior or cystocele prolapse) – happens when the supportive
tissue of the bladder and vaginal wall weakens and bulges. The result is the bladder
lowering into the vagina. Many of the same issues that cause a uterine prolapse can
also cause this condition.
• Rectocele prolapse ( A posterior vaginal ) - prolapse occurs when the thin wall of
tissue that separates the rectum and vagina weakens, causing the vaginal wall to
bulge and descend.
• Enterocele prolapse - This condition happens when the lower intestine descends
into the lower pelvic cavity and pushes at the top of the vagina.
Degrees of prolapse
• First-degree prolapse: The organs have only slipped down a little.
• Second-degree prolapse: The organs have slipped down to the level of the vaginal
opening.
• Third-degree prolapse: The vagina or womb has dropped down so much that up to 1 cm
of it is bulging out of the vaginal opening.
• Fourth-degree prolapse: More than 1 cm of the vagina or womb is bulging out of the
vaginal opening.

Preventive treatment
• Pelvic muscle exercises (Kegels) - strengthen or retrain the nerves and muscles of
the pelvic floor. Regular daily exercising of the pelvic muscles can be helpful.
Although pelvic floor exercises may not correct the prolapse, they may help
control symptoms and limit the worsening of prolapse.
• Maintain a Normal Weight - Overweight women are at a significantly increased
risk of developing prolapse.
• Cessation of Smoking -Smoking increases your risk of genital prolapse
• Choose High Fiber and Drink Plenty of Fluids - A diet with plenty of fiber and
fluids helps to maintain regular bowel function. Constipation increases your risk
for POP.
• Strain During Bowel Movements - Chronic straining and constipation increase
your chance of developing prolapse. This is especially true for posterior vaginal
wall prolapse.
• Regular checkup of the diseases -Chronic cough increases abdominal and pelvic
pressure—see a doctor to discuss treatment options. Persistent constipation also
requires further evaluation and treatment.
Conservative treatment
• Topical hormonal treatments improve patients' comfort
• Pessaries - cube-shaped pessaries have to be changed daily, permanent ring
pessaries require to be changed by a doctor every 3 months.
Surgical treatment
It depends on the type of genital prolapse that the patient have,
for severe symptomatic pelvic organ prolapse for patients who failed or refused a trial of
pessary management is surgery.
• anterior colporrhaphy-is to plicate the vaginal muscularis fascia overlying the
bladder (pubocervical fascia) to diminish the bladder and anterior vaginal
protrusion.
• Posterior vaginal repair (posterior colporrhaphy) is performed to repair the posterior
vaginal defect, usually a rectocele.
• abdominal sacral colpopexy and total abdominal hysterectomy- done with the high
uterosacral ligament suspension. allow fixation of the upper vagina or the uterus to
the sacrum, with the help of grafts and sutures through the anterior sacral ligament
(presacral fascia) at the level of the sacral promontory or at S1-S2.and done for
apical vaginal prolapse and uterine prolapse
• High uterosacral ligament suspension - suspend the prolapsed vaginal vault
bilaterally to the uterosacral ligaments
• Iliococcygeus suspension -suspending the vaginal vault to the fascia of the
iliococcygeus muscle in patients with attenuated uterosacral ligaments
• Endopelvic fascia repair -aims to suspend the prolapsed vaginal vault to the
endopelvic fascia
• Sacrospinous ligament fixation
104.gestational trophoblastic disease.(persistent hydatiform mole,invasive
mole,choriocarcinoma)clinical picture,diagnosis,treatment.

Clinical picture

• Period of amonorrhea following vaginal bleeding


• Passage of vessicles in uterus
• Hyperemesis gravidarum
• Uterus size>POA
• Increase thyroxine level
• Pre eclampsia
• Abdominal distenstion due to theca luteal ovarian cyst
• Choriocarcinoma -persistant ill health,continued ammonorhea,lungs-
cough,dyspnoea ,haemoptysis ,vaginal-irregular bleeding,cerebral-
headache,convulsion ,paralysis ,coma,liver-juandice

Diagnosis

• General examination-passage of vessicles from uterus.uterus size >POA


• Serum hcg-twice the amount of median value
• Transvaginal USS-complete mole-snow storm appearence,theca luteal cyst of ovaries
• Hystopathological -to differentiate complete and partial,malignant leissions
• Karyotype-to differentiate complete and partial mole.
• Chest x ray -to exclude lung metastasis.
• Liver function test- exclude liver metastasis .
Treatment

• Treatment of choice is suction evacuation .-high risk for bleeding(prepare blood for
cross match.
• Complete mole-suction evacuation
• Partial mole-sucction evacuation /medical evacuation .
• After treatment check hcg level monthly for 6months.
• For choriocarcinoma
• Stage -1
• Law risk-single agent of chemotherapy -methotreaxate
• High risk-combination therapy(methotrexate ,actinomycin B and etoposide.
• stage-2
• Law risk-single agent chemotherapy -methotrexate
High risk-combination therapy(methotrexate ,actinomycin B and etoposide.
• stage -3
• Law risk-single agent chemotherapy +hysteroectomy
• High risk-combination therapy +hysteroectomy
• Stage -4
• Combinations therapy+surgery(hepatic resection,cranioectomy)+radiation
• After treatment follow up with HCG level,avoid pregnancy for 12months since after
chemotherapy completion.

105. Endoscopic methods of diagnosis and treatment for gynecological types of diseases
Endoscopy has become an essential armamentarium in the diagnostic evaluation of gynecologic
lesions as well as for operative procedures. Gynecological endoscopy

Includes the procedures


• Laparoscopy • Hysteroscopy • Salpingoscopy
• Falloposcopy • Cystoscopy • Culdoscopy
• Sigmoidoscopy and proctoscopy
LAPAROSCOPY
Laparoscopy is a technique of visualization of peritoneal cavity by means of a fiberoptic
endoscope introduced through the abdominal wall. Prior Pneumoperitoneum is achieved by
introduction of
Carbon dioxide or air. For diagnostic purposes, either local or general anesthesia may be used.
Indications
Diagnostic
* Infertility work up
– Peritubal adhesions.
– Chromopertubation
– Minimal endometriosis
– Ovulation stigma of the ovary
– Before reversal of sterilization operation
* Chronic pelvic pain
* Nature of a pelvic mass: Fibroid, ovarian cyst
To diagnose an acute pelvic lesion
– Ectopic pregnancy
– Acute appendicitis
– Acute salpingitis—diagnosis and collection of pus for culture.
*Follow-up of pelvic surgery (second look)
– Tuboplasty
– Ovarian malignancy
– Evaluation of therapy in endometriosis.
*Investigation protocol of amenorrhea
* Diagnosis of suspected Müllerian abnormalities
*Uterine perforation.
HYSTEROSCOPY
Hysteroscopy is an operative procedure whereby the endometrial cavity can be visualized with
the aid of fiberoptic telescope. The uterine distension Is achieved by carbon dioxide, normal
saline or
Glycine. The instrument is to pass transcervically, Usually without dilatation of the cervix or
local anesthesia. However, for operative hysteroscopy, either Paracervical block or general
anesthesia is required.
Diagnostic hysteroscopy should be performed in The postmenstrual period for better view
without
Bleeding. The chance of conception disturbance is absent
Diagnostic
*Unresponsive irregular uterine bleeding to exclude Uterine polyp, submucous fibroid or
products of
Conception
* Congenital uterine septum in recurrent abortion
*Missing threads of IUD
* Intrauterine adhesions (uterine synechiae)
*To visualize transformation zone with colpomi-
Crohysteroscopy when colposcopic finding is Unsatisfactory.
Operative—Transcervical resection of endometrium (TCRE) Or laser ablation of endometrium
(LAE) is done As an alternative to hysterectomy for dysfunctional Uterine bleeding
SALPINGOSCOPY: In salpingoscopy, a firm telescope Is inserted through the abdominal ostium
of the
Uterine tube so that the tubal mucosa can be visualized By distending the lumen with saline
infusion. The Telescope is to be introduced through laparoscope.
Salpingoscopy allows study of the physiology and Anatomy of the tubal epithelium and permits
more
Accurate selection of patients for IVF rather than The tubal surgery.
Falloposcopy—study the entire length of tubal Lumen with the help of a fine and flexible
fiberoptic
Device. It is performed through the uterine cavity, using A hysteroscope. It helps direct
visualization of tubal Ostia, mucosal pattern, intratubal polyps, or debris.
CYSTOSCOPY: The main use of cystoscopy in Gynecology is to evaluate cervical cancer prior to
Staging and, to investigate the urinary symptoms Including hematuria, incontinence and
fistulae.
CULDOSCOPY: Culdoscopy is an optical instrument Designed to visualize the pelvic structures
through an
Incision in the pouch of Douglas. Its use has almost Been replaced by laparoscopy.
PROCTOSCOPY AND SIGMOIDOSCOPY: For Rectal involvement of genital malignancy, a digital
Examination or at best proctoscopy is usually adequate.
EXAMINATION UNDER ANESTHESIA (EUA): EUA Is indicated where bimanual examination
cannot
Be conducted properly either because of extreme Tenderness or inadequate relaxation of
abdominopelvic Muscles or non-cooperative patient. It should be done Routinely in all cases of
uterine malignancy for clinical Staging. It is extended freely to examine virgins or in Cases with
pediatric gynecological problems.
106.Premenopauzal syndrome. Reasons of emergence, clinic, assessment of disease severity,
principles of treatment.
Reasons of emergence
Perimenopause is the transition phase before menopause, which can last from 2 to 10 years.
During this time, ovulation becomes irregular and the production of estrogen and progesterone
decreases. Several processes occur within the body, including:

• Infrequent release of eggs by the ovaries


• Reduced production of estrogen and other hormones
• Decreased fertility
• Shorter menstrual cycles, fewer ovulations and more cycle irregularity
• Reduced levels of hormones, including estrogen, progesterone, androgen
(androstenedione) and testosterone

Clinical features
• Hot flashes
• Night sweating
• Breast tenderness
• Worse premenstrual syndrome
• Lower sex drive
• Fatigue
• Joint and muscles aches
• Difficulty concentration
• Irregular periods
• Vaginal dryness, discomfort during sex
• Urine leakage when coughing or sneezing
• Frequent urination
• Mood changes
• Trouble sleeping
Assessment of disease severity
By reviewing a woman's medical history, her menstrual history, and her signs and symptoms

Principles of treatment

• Explain to the patient that the symptoms that she is experiencing are normal to that of
women undergoing the process of menopause.
• Lifestyle modification (Eating a healthy diet which includes whole grains, fruits and
vegetables, get enough calcium, getting regular exercise, quite smoking and alcohol,
remain healthy body weight etc )
• Hormone Replacement Therapy (low dose combined oral oestrogens and progestins)
• Sedatives (For symptomatic treatment of irritability and insomnia)
• Herbal medicines (Symptomatic management)

107. chronic salpingo-oophoritis. Clinical options of disease course, specifics of treatment,


forecast for female reproductive function.

Chronic salpingitis

pathology

Hydrosalpinx.

Pyosalpinx.

Chronic interstitial salpingitis.

Salpingitis isthmica nodosa.

Clinical Features:

abdominal tenderness

rebound tenderness (±).

Cervical and uterine motion tenderness.

adnexal tenderness.

temperature (> 38°C).

leucocytosis (> 10,000/mm3).


Purulent material from peritoneal cavity by laparoscopy or by culdocentesis

Pelvic abscess or tubo-ovarian mass on bimanual examination or on sonography.

* Diagnosis-Ultrasound and Color Doppler (TVS)- Sausage-shaped complex cystic structure with
reduced resistance index (RI) in the adnexal region is suggestive of the diagnosis

Treatment :

Antibiotics.

For acute salphingitis :

Outpatient therapy: (i) Ofloxacin 400 mg PO twice daily for 14 days plus metronidazole 500 mg
PO twice daily for 14 days.

Inpatient therapy (Temp >39°C, toxic look, lower abdominal guarding, and rebound
tenderness). Clindamycin 900 mg IV 8 hourly, plus gentamicin 2 mg/kg IV, then 1.5 mg/kg IV
every 8 hours are given. This is followed by doxycycline 100 mg twice daily orally for 14 days.

IV fluids to correct dehydration and nasogastric suction in the presence of abdominal


distension or ileus are maintained.

Laparotomy is done if there is clinical suggestion of abscess rupture.

Surgery,

Surgical drainage if collection of pus or fluid

salphingectomy

chronic oophoritis

Clinical features of chronic oophoritis:

The chief complaint is chronic deep seated pelvic pain, may be localized to one side. There is
deep dyspareunia

Pelvic examination reveals the uterus fixed to an indurated and tender mass. The uterus is also
drawn to the affected side because of scarring. Movement of the cervix produces pain.
Ultrasonograhy can localize the abscess with its site and extent.

Treatment :
Deep pelvic short wave diathermy may be tried to relieve pain and dyspareunia. Too often, all
the measures fail, hysterectomy decision may have to be considered even at an early age
specially in women whose family is completed.

forecast for female reproductive function.

if treatment is delayed or if the infection is left untreated entirely salpingitis can cause
blockages, adhesions, or scarring in the fallopian tubes. This can lead to infertility.

(complications)

The following may happen :

(i) Formation of tuboovarian cyst

(ii) Torsion

(iii) Infection from the gut

(iv) Rupture.

Pyosalphinx- intense inflammatory reaction with secretion of pus

Chronic intestinal salpingitis

108. Sex cord stromal tumor of ovary. Clinical features


Examples are
Granulosa cell tumors.
x Thecomas, fibromas.
x Sertoli-Leydig cell tumors (androblastoma).
x Gynandroblastoma (mixed)

clinical features

Granulose cell tumor


Prior to puberty: Precocious puberty
x Childbearing period: Abnormal uterine bleeding.
x Postmenopausal: Bleeding
Acute abdominal bleeding

Thecoma
due to excess estrogen production, there is endo-metrial hyperplasia and often associated with
endo- metrial carcinoma. It is responsible for postmenopausal bleeding.

sertoli-leydig cell tumor


defeminization — atrophy of the breasts and uterus and amenorrhea followed by
masculinization
This is evidenced by male type of distribution of hair, hoarseness of voice, breast atrophy,
hirsutism, baldness and clitoral enlargement. Serum testosterone level is elevated

Gynandroblastoma
This is a very rare type of tumor. It contains both granulosa cell (estrogenic) or Sertoli-Leydig
cell (androgenic) types. Usually it has got a benign course.

109.Medications for treatment of uterine bleeding (hormonal, haemostatic, uterotonic drugs).

Hormonal medications
• Birth control pill
• Conjugated estrogen 25 – 40 mg IV
• Progestins pills (medroxyprogesterone acetate, norethindrone ), 1/more times daily for
2-3 weeks
• Intrauterine contraceptive device ( IUD )

Hemostatic drugs
• Tranexamic acid
• Non-steroidal anti-inflammatory drugs (ibuprofen 600-800mg 6-8h, naproxen 250-
500mg 12h)
• Danazol
• GnRH analog
• Antifibrinolytic

Uterotonic drugs
• Oxytocins
• Prostaglandins
• Ergot alkaloids.

Gven intramuscularly (IM), intravenously (IV), and as a tablet, gel, or suppository

110.Spontaneous abortions. Causes, clinic and principles of treatment, in stages.


spontaneous abortion = miscarriage
➢ Threatened miscarriage
➢ Inevitable miscarriage
➢ Complete miscarriage
➢ Incomplete miscarriage
➢ Missed miscarriage
➢ Septic miscarriage

Causes
✓ Genetic (majority 50%) (Autosomal trisomy, Polyploidy, Monosomy X (45, X),
Structural chromosomal rearrangements, Other chromosomal abnormalities like
mosaic, double trisomy, etc)

✓ Endocrine and metabolic (Luteal Phase Defect (LPD), Deficient progesterone


secretion from corpus luteum or poor endometrial response to progesterone, Thyroid
abnormalities: Overt hypothyroidism or hyperthyroidism, Poorly controlled Diabetes
mellitus )

✓ Anatomical abnormalities (Cervico–uterine factors : Cervical incompetence,


Congenital malformation of the uterus ( (i) reduced intra-uterine volume, (ii) reduced
expansile property of the uterus, (iii) reduced placental vascularity when implanted
on the septum and (iv) increased uterine irritability and contractility.), Uterine fibroid,
Intrauterine adhesions (synechiae))

✓ Infection (Viral: Rubella, cytomegalovirus, variola, vaccinia or HIV; Parasitic:


Toxoplasma, malaria ; Bacterial: Ureaplasma, chlamydia, brucella)

✓ Immunological (Antiphospholipid antibody syndrome (APAS), Immune factors


(Successful pregnancy is the result of predominantly Th2 cytokine response.Women
with recurrent miscarriage have more Th 1 response), Autoimmunity)
✓ Thrombophilias

✓ Environmental (Cigarette smoking, Alcohol consumption, Contraceptive agents (IUD),


Drugs, chemicals, noxious agents (anesthetic gases, arsenic, aniline, lead,
formaldehyde), Miscellaneous

✓ Others (Maternal medical illness (Cyanotic heart disease, hemoglobinopathies),


Premature rupture of the membrane, Paternal factors: Sperm chromosomal anomaly

✓ Unexplained (40-60%)

Threatened miscarriage (It is a clinical entity where the process of miscarriage has started
but has not progressed to a state from which recovery is impossible)
Clinical features
- Bleeding per vaginam (slight and may be brownish or bright red in color)
- Pain (Bleeding is usually painless but there may be mild backache or dull pain in lower
abdomen.)
Treatment
- Rest: The patient should be in bed for few days until bleeding stops. limit
activities for at least 2 weeks and avoid heavy work.
- Drugs: Relief of pain —>diazepam 5 mg tablet twice daily

Inevitable miscarriage (It is the clinical type of abortion where the changes have progressed to a
state from where continuation of pregnancy is impossible)
Clinical features
- Increased vaginal bleeding.
- Aggravation of pain in the lower abdomen
- Internal examination reveals dilated internal os of the cervix
Treatment
- Excessive bleeding controlled by administering Methergine 0.2 mg if the cervix is dilated
and the size of the uterus is less than 12 weeks.
- The blood loss is corrected by intravenous (IV) fluid therapy and blood transfusion.
Active Treatment:
- Before 12 weeks:
(1) Dilatation and evacuation followed by curettage of the uterine cavity by blunt curette
using analgesia or under general anesthesia.
(2) Alternatively, suction evacuation followed by curettage is done.

- After 12 weeks:
(1) The uterine contraction is accelerated by oxytocin drip (10 units in 500 mL of normal
saline) 40–60 drops per minute.
If the fetus is expelled and the placenta is retained —> removed by ovum forceps
If the placenta is not separated —> digital separation followed by its evacuation
(under general anesthesia)

Complete miscarriage (When the products of conception are expelled)


Clinical features
- Abdominal pain.
- Vaginal bleeding becomes trace or absent.
- Internal examination reveals: Uterus is smaller than the period of amenorrhea and a little
firmer, Cervical os is closed, Bleeding is trace
- Examination of the expelled fleshy mass is found complete.
- Ultrasonography (TVS): reveals empty uterine cavity.
Treatment
Transvaginal sonography - to see that uterine cavity is empty,
otherwise evacuation of uterine curettage should be done.
Incomplete miscarriage (When the entire products of conception are not expelled, instead a
part of it is left inside the uterine cavity)
Clinical features
- Continuation of pain in lower abdomen.
- Persistence of vaginal bleeding
- Internal examination reveals: uterus smaller than the period of amenorrhea, patulous
cervical os often admitting tip of the finger , varying amount of bleeding. - on examination,
the expelled mass is found incomplete
- Ultrasonography: reveals echogenic material (products of conception) within the cavity
Treatment
- Evacuation of the retained products of conception
- Tablet misoprostol 200 µg is used vaginally every 4 hours

Missed miscarriage (When the fetus is dead and retained inside the uterus for a variable
period)
Clinical features
- Persistence of brownish vaginal discharge.
- Subsidence of pregnancy symptoms
- Retrogression of breast changes.
- Cessation of uterine growth which in fact becomes smaller in size.
- Non-audibility of the fetal heart sound
- Cervix feels firm.
- Immunological test for pregnancy becomes negative.
- Realtime ultrasonography reveals an empty sac early in the pregnancy or the absence of
fetal cardiac motion and fetal movements.
Treatment
Uterus is less than 12 weeks:
(i) Expectant management—Many women expel the conceptus spontaneously
(ii) Medical management: Prostaglandin E1 (misoprostol) 800 mg vaginally in the
posterior fornix is given
(iii) Suction evacuation or dilatation and evacuation is done

Septic miscarriage (Any abortion associated with clinical evidences of infection of the uterus
and its contents)
Clinical features
The woman looks sick and anxious
High temperature
Chills and rigors (suggest-bacteremia)
Persistent tachycardia
Hypothermia (endotoxic shock) < 36°C
Abdominal or chest pain
Tachypnea
Impaired mental state
Diarrhea and/or vomiting
Renal angle tenderness
Pelvic examination: Offensive, purulent vaginal discharge, uterine tenderness, boggy feel in
the POD (pelvic abscess)
Treatment
Principles of management are:
(a) To control sepsis.
(b) To remove the source of infection.
(c) To give supportive therapy to bring back the normal homeostatic and cellular metabolism.
(d) To assess the response of treatment.
(1) Antibiotics
(2) Prophylactic antigas gangrene serum and antitetanus serumintramuscularly are given if
there is a history of interference.
(3) Analgesics and sedatives
111. Endometrial cancer. Factors of risk, clinical options, diagnostics, modern methods of
treatment. Prophylaxis.
Risk factors
Early menarche
Late menopause
Nulliparity
Obesity
Chronic anovulation/polycystic ovarian disease
Exogenous unopposed estrogen
Estrogen secreting tumors
History of breast and ovarian cancer, history of hypertension and diabetes mellitus
Tamoxifen therapy
Radiation menopause

Clinical features

Patient profile:
Obesity, hypertension or diabetes

symptoms

* Postmenopausal bleeding , which may be slight, irregular or continuous. may be excessive.

* In premenopausal women, there may be irregular and excessive bleeding.

* At times, there is watery and offensive discharge due to pyometra.

* Pain is not uncommon. It may be colicky due to uterine contractions in an attempt to expel
the polypoidal growth.

* Few patients (< 5%) remain asymptomatic.

Signs: as mentioned in patient profile. There may be varying degrees of pallor.

Pelvic examination: Speculum examination reveals the cervix looking healthy and the blood or
purulent offensive discharge escapes out of the external os.

Bimanual examination reveals—The uterus is either atrophic, normal or may be enlarged due to
spread of the tumor, associated fibroid or pyometra.
The uterus is usually mobile unless in late stage, when it becomes fixed.

Rectal examination corroborates the bimanual findings.

Regional lymph nodes and breasts are examined carefully

Diagnosis

The following guidelines are prescribed:

* postmenopausal bleeding

* History and clinical examination are to be recorded.

* Endometrial biopsy – using a Sharman curette or a soft, flexible, plastic suction cannula
(pipelle) has been done with reliability (90%). This is done as an outpatient procedure. Histology
is the definitive diagnosis.

* Papanicolaou smear is not a reliable diagnostic test. Only in 30% cases.

*Ultrasound and color Doppler (TVS):

Endometrial thickness > 4 mm.


Hyperechoic endometrium with irregular outline.
Increased vascularity with low vascular resistance.
Intrauterine fluid. However, it cannot replace definitive biopsy.

*Hysteroscopy—It helps in direct visualization of endometrium and to take target biopsy.

* Fractional curettage—It is not only the definite method of diagnosis but can detect the extent
of growth. This is done under anesthesia with utmost gentleness to prevent perforation of the
uterus. If pyometra is detected, the procedure is withheld for about 1 week to avoid
perforation and systemic infection.

The orderly steps for fractional curettage:

-Endocervical curettage (ECC).

- To pass an uterine sound to note the length of the uterocervical canal.

-Dilatation of the internal os.


-Uterine curettage at the fundus and lower part of the body. The endometrial tissue is usually
profuse and often dark color.

- Finally, a polyp forceps is introduced in case any endometrial polyp has escaped the curette.

The specimens should be placed in separate containers, labelled submitted for histological
examination.

* Computed Tomography (CT) to detect lymph node metastases

*Magnetic Resonance Imaging (MRI) can detect myometrial invasion

*Positron Emission Tomography

Modern methods of treatment


Preventive/prophylaxis : Primary prevention includes:
* Strict weight control
* restrict the use of estrogen after menopause in nonhysterectomized women.
* Education as regard the significance of irregular bleeding per vaginum in perimenopausal and
postmenopausal period.

Secondary prevention includes:

* Screening of ‘high risk’ women

Methods

* The cytologic specimens are obtained by either endometrial aspiration or endometrial lavage.

If suspicious cells are detected, histological specimen is obtained by uterine curettage.

*The presence of abnormal endometrial cells in vaginal pool cytology requires a diagnostic
curettage.

* Judicial hysterectomy in premalignant lesions of the corpus

Curative : surgery, radiotherapy, chemotherapy, combined therapy

In stage 1- surgery is main

Surgical procedures
- Incision longitudinal midline or paramedian

-Peritoneal washings are taken for cytology.

- Thorough exploration of liver, diaphragm, omentum, pelvic organs, pelvic and paraaortic
lymph nodes, is done.

-Suturing the cervix and fimbrial end of the fallopian tubes to prevent tumor cells spillage
during operation are not essential.

-Total abdominal hysterectomy (extrafascial) with bilateral salpingo-oophorectomy


.- Presence of big tumor (>2 cm), cervical extension, poor tumor differentiation and myometrial
invasion (>1/2 thickness) as determined by frozen section biopsy, indicates pelvic and
paraaortic lymph node sampling.
- Lymph node sampling of the following areas is done : (i) Common iliac (ii) External iliac (iii)
Internal iliac (iv) Obturator and (v) Paraaortic.
-Vaginal hysterectomy may be done (stage I, with well-differentiated tumor)
-Laparoscopic hysterectomy

In stage 2
A. Radical hysterectomy bilateral salpingooophorectomy with pelvic and para-aortic
lymphadenectomy
B. Combined radiation and surgery: Radiation (external and intracavitary) followed in 6 weeks
by total abdominal hysterectomy and bilateral salpingo-oophorectomy.
or C. Initial surgery (modified radical hysterectomy) followed by external and intravaginal
radiation

radiotherapy

Contraindications of radiotherapy: Presence of a pelvic mass, pelvic kidney, pyometra, pelvic


abscess

Combined therapy- (surgery and radiation)

For Stage 3, 4

Adjuvant chemotherapy followed by pelvic radiation.


Combination chemotherapy is commonly used.
Drugs comprise: adriamycin, cisplatin and cyclophosphamide.

External pelvic and intracavitary radiation followed by extended hysterectomy

Chemotherapy
used in advanced and recurrent cases or in metastatic lesions.
-progesterone
-tamoxifen- is a non steroidal anti estrogen
-cytotoxic drugs- adriamycin, cisplatin, carboplatin, paclitaxel and cyclophosphamide

For recurrent disease

Radiation therapy
Exenterative surgery
Hormonal therapy and chemotherapy

112. Epithelial tumor of ovary. Classification. Clinical features. Diagnosis. Differential


diagnosis. Complications. Treatment.

Classification:
1) Serous: 1) serous cystadenoma (benign)
2) serous cystadenocarcinoma(malignant)
3) Intermediate
2) Mucinous: 1) mucinous cystadenoma (benign)
2) mucinous cystadenocarcinoma (malignant)
3) intermediate
3) Endometrioid: benign/ intermediate/malignant
4) Brenner
5) Clear cell: benign/ intermediate/ mlignant
6) Undifferentiated

Morphologically
● Cystic – Cystadenomas
● Solid/cystic – Cystadenofibromas
● Solid - adenofibromas

Clinical Features:
Symptoms: In its early stage, ovarian carcinoma is a notoriously silent disease (asymptomatic).
The presenting complaints are usually of short duration and insidious in onset. Symptoms are
not specific.
Feeling of abdominal distension and vague discomfort.
Features of dyspepsia such as flatulence and eructations
Loss of appetite with a sense of bloating after meals.
In pre-existing tumor.
− Appearance of dull aching pain and tenderness over one area.
− Rapid enlargement of the tumor.
Gradually, more pronounced symptoms appear. These
are:
Abdominal swelling which may be rapid.
Dull abdominal pain.
Sudden loss of weight.
Respiratory distress — may be mechanical due to ascites or due to pleural effusion.
Menstrual abnormality is conspicuously absent except in functioning ovarian tumors

Signs: The following are the findings in an established case of ovarian malignancy
general Examination reveals:
Cachexia and pallor of varying degree.
Jaundice may be evident in late cases.
Left supraclavicular lymph gland (Virchow’s) may be enlarged
Edema leg or vulva is characteristic of malignant and not of benign neoplasm.

Per abdomen
Liver may be enlarged, firm and nodular.
A mass is felt in the hypogastrium; too often it may be bilateral. It has got the following
features:
− Feel — solid or heterogenous.
− Mobility — mobile or restricted.
− Tenderness — usually present.
− Surfaces — irregular.
− Margins — well-defined but the lower pole is usually not reached.
− Percussion — usually dull over the tumor; may be resonant due to overlying intestinal
adhesions.

Per vaginum
The uterus may be separated from the mass felt per abdomen.
Nodules may be felt through the posterior fornix. If it is more than 1 cm, the diagnosis of
malignancy is almost certain

Treatment:
PrEvENtivE
Primary prevention: Because of dearth about the knowledge of epidemiology of ovarian
cancer, the
primary prevention cannot be clearly formulated.
However, the preventive measures are:
(i) Genetic screening for BRCA 1 and BRCA 2 for women with high risk for ovarian and breast
cancer.
(ii) Annual mammographic screening for women with strong family history of breast cancer.
(iii) Periodic screening for other malignancies (colonoscopy, endometrial biopsy) for women
with Lynch II syndrome
(iv) Combined oral contraceptive pills as a preventive measure is recommended to a woman
specially belonging to Lynch type II families.
(v) Prophylactic oophorectomy along with
hysterectomy specially in ‘high risk’ women is a preventive measure against ovarian
malignancy.

guidelines for Management of an Enlarged ovary


An ovarian enlargement of > 8 cm during childbearing period deserves careful follow up.
In postmenopausal women, any ovarian enlargement should be assessed by serum CA-125 and
transvaginal sonography.
Cysts that are simple, unilocular, < 8 cm in diameter with normal serum CA-125 — can be
managed conservatively. Women should be under follow up with ultrasound scan and serum
CA-125 at an interval of 4 months.
Early laparotomy is indicated in following cases:
− The ovary enlarges progressively beyond 8cm
while under observation.
− Any symptomatic ovarian tumor regardless
of size.

Secondary prevention (screening):


Clinical
Tumor markers
Ultrasound imaging
Genetic testing

113. Menopausal hormone therapy. Basic principles of a substitution hormone therapy


appointment. Indications and contraindications for the appointment of menopausal hormone
therapy preparations

The HRT is indicated in menopausal women to


overcome the short-term and long-term consequences
of estrogen deficiency

The principal hormone used in HRT is estrogen. woman with an intact uterus, only estrogen
therapy leads to endometrial
hyperplasia and even endometrial carcinoma. Addition of progestins for last 12–14 days each
month can prevent
this problem. Commonly used estrogens are conjugated estrogen (0.625–1.25 mg/day) or
micronized estradiol . Progestins used are medroxyprogesterone acetate, micronized
progesterone
Considering the risks, hormone therapy should be used with the lowest effective dose and for a
short period of time

Oral estrogen regime- woman who had hysterectomy.


Estrogen and cyclic progestin-For a woman with intact uterus
Continuous estrogen and progestin therapy- can prevent endometrial hyperplasia.

Indication of Hormone Replacement Therapy


Relief of menopausal symptoms
Prevention of osteoporosis
To maintain the quality of life in menopausal years.
Premature ovarian failure
Gonadal dysgenesis
Surgical or radiation menopause

contraindications

undiagnosed genital tract bleeding


estrogen dependent neoplasm in the body
History of venous thromboembolism
active liver disease
Gallbladder disease

114. Infertile marriage. Causes. Methods of married couples inspection

Infertility is defined as a failure to conceive within one or more years of


regular unprotected coitus.
Primary infertility -patients who have never conceived.
Secondary infertility -previous pregnancy but failure to conceive subsequently

causes

males infertility
Defective spermatogenesis
Obstruction of the efferent duct system
Failure to deposit sperm high in the vagina
Errors in the seminal fluid.
Hypothalamic-pituitary disorders
Undescended testes
drugs (antihypertensive)
genetic mutations
infections (gonorrhoea)
immunological factors

females infertility
Ovarian factors
Anovulation or oligo-ovulation
Decreased ovarian reserve
Luteal phase defect
Luteinized unruptured follicle

Tubal and peritoneal factors


Pelvic infections
Previous tubal surgery or sterilization.
Salpingitis isthmica nodosa
Tubal endometriosis
Polyps or mucous debris within the tubal lumen,
tubal spasm

Uterine factors
uterine hypoplasia, inadequate secretory
endometrium, fibroid uterus, endometritis
congenital malformation of uterus

Cervical factor
anatomical and physiological defects

Vaginal factors
Atresia of vagina , transverse vaginal
septum, septate vagina, or narrow introitus
Vaginitis and purulent discharge may at times be
implicated

Male
History
full physical examination especially reproductive system
Routine investigation includes urine and blood examination
Semen analysis (norm concentration of sperms 20 million/ml)
Serum FSH, LH, testosterone, prolactin, and TSH
qualitative and quantitative examination of semen
Transrectal ultrasound
Immunological tests

Female
History- (surgical, Menstrual, obstetric) contraception, Sexual problems
examination -General examination, Systemic examination, Gynaecological examination ,
Speculum examination

In the presence of major fault in male, there is very little scope to proceed for
investigation for the female partner. when a major defect is detected in
female such as müllerian agenesis or intersex, infertility investigations should be suspended.
However, correctable abnormality should be rectified first prior to investigation
try to use non invasive or minimal invasive methods

115. Principles and methods of surgical treatment of “acute abdomen” in gynecology.


Rehabilitation after surgery.
The patients are most often critically ill. Intensive resuscitative or supportive measures are to be
taken.
Hospitalized immediately.
Operated immediately after admission
- Maximum time for operation is 2 hrs
- Optimal interval is 30 min
- Operated within 10 min after admission in some cases (pt. in shock/supor condition)
Definitive Treatment for definitive diagnosis
- Immediate laparotomy – hemoperitoneum, rupture of tubo-ovarian abscess, twisted ovarian
cyst, acute surgical condition.
- Institution of medical therapy – UTI, PID, gastroenteritis, hyperstimulation syndrome.
First, diagnostic laparoscopy is done to visualize pelvic pathology.
Rehabilitation –
- Monitor the patient for normal vital signs, oxygen saturation, depth of consciousness, cardiac
rhythm, skin temperature, colour and urine output.
- Opioid analgesic - to relieve pain and anxiety
- Managing the surgical site: observed for bleeding, type and integrity of dressing and drains.
- Assessing and managing gastrointestinal function: − Nausea and vomiting are common after
anaesthesia. Check of peristalsis movement.
- Assessing and managing voluntary voiding
- Early ambulation reduces the incidence of post-operative complication as, atelectasis,
pneumonia, gastrointestinal discomfort and circulatory problem.
116. Postmenopausal bleeding. Reasons, clinic, differential diagnostics, fundamentals of
treatment.
Reasons –
- Senile endometritis
- Genital malignancy – (Carcinoma of the cervix, endometrium, vagina, vulva and Fallopian
tube / Sarcoma uterus / Granulosa cell tumor of the ovary)
- Dysfunctional uterine bleeding.
- Decubitus ulcer.
- Urethral caruncle, polyp, prolapse mucosa or carcinoma.
- Retained and forgotten foreign body such as pessary or IUCD.
++ (Atrophic endometrium, Endometrial hyperplasia, Uterine polyp, Tubercular endometritis,
Cervical erosion and polyp, Senile vaginitis, Withdrawal bleeding following estrogen intake)
Clinic –
General examination
Obesity and hypertension are often related to endometrial carcinoma.
Enlarged groin or supraclavicular lymph nodes may be palpated. Metastatic nodules in the
anterior vaginal wall may be present.
Per abdomen: A lump in the lower abdomen may be due to pyometra or uterine sarcoma or
adnexal mass.
Bimanual examination
- Uterus may be normal, atrophic or enlarged due to pyometra or sarcoma.
- Adnexal mass (infective or ovarian) may be palpable.
Differential diagnosis –
- Atrophic vaginitis
- Endometrial/ Cervical polyp
- Endometrial hyperplasia
- Endometrial carcinoma
- Hormonal effect
Exogenous estrogens – hormone therapy
Endogenous estrogens – acute stress, estrogen secreting ovarian tumor
- Cervical carcinoma
Treatment –
If the cause is found; treatment is directed to it.
If no cause is detected; only minimal bleeding once or twice, careful observation.
In cases of recurrences or continued bleeding; whatever may be the amount, it is better to
proceed for laparotomy and to perform hysterectomy with bilateral salpingo-oophorectomy.

117. Gonorrhea of the lower part of genitals. Clinic, diagnostics, treatment.


Clinic –
Symptoms
- Urinary symptoms such as dysuria
- Excessive irritant vaginal discharge
- Acute unilateral pain and swelling over the labia due to involvement of Bartholin’s gland
- There may be rectal discomfort due to associated proctitis from genital contamination
- Others: Pharyngeal infection, intermenstrual bleeding.
Signs
- Labia - swollen and inflamed
- vaginal discharge - mucopurulent
- The external urethral meatus and the openings of the Bartholin’s ducts - congested. On
squeezing the urethra and giving pressure on the Bartholin’s glands, purulent exudate
escapes out through the openings. Bartholin’s gland may be palpably enlarged, tender with
fluctuation, suggestive of formation of abscess.
- Speculum examination reveals congested ectocervix with increased mucopurulent cervical
secretions escaping out through the external os
Diagnostics –
- Nucleic acid amplication testing (NAAT) of urine or endocervical discharge. First void morning
urine sample (preferred) or at least one hour since the last void sample should be tested.
- In the acute phase, secretions from the urethra, Bartholin’s gland, and endocervix are
collected for Gram stain and culture.
- Culture of the discharge in Thayer Martin medium further confirms the diagnosis.
- Drug sensitivity test is also to be performed
Treatment –
Preventive
- Adequate therapy for gonococcal infection and meticulous follow up are to be done till the
patient is declared cured.
- To treat adequately the male sexual partner simultaneously.
- To avoid multiple sex partners.
- To use condom till both the sexual partners are free from disease.
Curative:
- single dose regimen
Ceftriaxone — 125 mg IM
Ciprofloxacin — 500 mg PO
Ofloxacin — 400 mg PO
Cefixime — 400 mg PO
Levofloxacin — 250 mg PO
- Patient with gonorrhea must be suspected of having syphilis or chlamydial infection. As such,
treatment should cover all the three.

118. Functional cysts of ovaries. Modern methods of diagnostics and treatment.


Diagnostics –
Related to temporary hormonal disorders.
Rarely becomes complicated.
Sometimes confused with neoplastic cyst but can be distinguished by the following features:
- Usually 6–8 cm in diameter.
- Usually asymptomatic.
- Spontaneous regression usually following correction of the functional disturbances to which
it is related.
- Unilocular.
- Contains clear fluid inside unless hemorrhage occurs.
- Lining epithelium corresponds to the functional epithelium of the unit from which it arises
These cysts are small and re-examination after 12 weeks solves the diagnosis in most cases. The
follicular or corpus luteum cyst usually regresses, while neoplastic cyst usually increases in size.
Laparoscopy is of help.
Treatment –
Whenever a cyst persists or grow, it should be removed by laparoscopy/laparotomy.

119. Medical abortions. Methods of abortion performance and their assessment.


Methods –
In first trimester
- Mifepristone
- Mifepristone and misoprostol (PGE1)
- Methotrexate and misoprostol
- Tamoxifen and misoprostol
In second trimester - Prostaglandins and their analogs or Oxytocin
- Misoprostol (PGE1 analog):
- Mifepristone and prostaglandins
- Gemeprost (PGE1 analog)
- Dinoprostone (PGE2 analog)
- Prostaglandin F2α (PGF2α)
Assessment –
In first trimester
- 200 mg of mifepristone orally is given on day 1.
- On day 3, misoprostol (PGE1) 400 µg orally or 800 µg vaginally is given.
- Patient remains in the clinic for 4 hours during which expulsion of the conceptus often
occurs.
- Patient is reexamined after 10–14 days.
- Complete abortion is observed in most.
In second trimester
- Misoprostol (PGE1 analog): mean induction—abortion interval is 11–12 hours
- Mifepristone and prostaglandins: median induction delivery interval is 6.5 hours
- Gemeprost (PGE1 analog): mean induction-abortion interval was 14–18 hours.
- Dinoprostone (PGE2 analog): mean induction to abortion interval is 17 hours
- Prostaglandin F2α (PGF2α): mean induction to abortion interval is in 36 hours

You might also like